commit bb326fbd0ab5f9ff3dfb5341a28b023fb5eca2c1 Author: Michael Loveys Date: Wed Sep 8 10:33:48 2021 -0500 inital commit diff --git a/Assignment 10.2/main.py b/Assignment 10.2/main.py new file mode 100644 index 0000000..c088118 --- /dev/null +++ b/Assignment 10.2/main.py @@ -0,0 +1,17 @@ +name = input("Enter file:") +if len(name) < 1: + name = "mbox-short.txt" +handle = open(name) + +emailhour = dict() + +for line in handle: + line = line.strip().split() + + if len(line) > 0: + if line[0] == "From": + hour = line[5].split(":")[0] + emailhour[hour] = emailhour.get(hour, 0) + 1 + +for hour,numberofemails in sorted(emailhour.items()): + print(hour,numberofemails) \ No newline at end of file diff --git a/Assignment 10.2/readme.md b/Assignment 10.2/readme.md new file mode 100644 index 0000000..896af30 --- /dev/null +++ b/Assignment 10.2/readme.md @@ -0,0 +1,22 @@ + +10.2 Write a program to read through the mbox-short.txt and figure out the distribution by hour of the day for each of the messages. You can pull the hour out from the 'From ' line by finding the time and then splitting the string a second time using a colon. + +```From stephen.marquard@uct.ac.za Sat Jan 5 09:14:16 2008``` + +Once you have accumulated the counts for each hour, print out the counts, sorted by hour as shown below. + +Desired Output +``` +04 3 +06 1 +07 1 +09 2 +10 3 +11 6 +14 1 +15 2 +16 4 +17 2 +18 1 +19 1 +``` \ No newline at end of file diff --git a/Assignment 2.2/main.py b/Assignment 2.2/main.py new file mode 100644 index 0000000..6fa42a0 --- /dev/null +++ b/Assignment 2.2/main.py @@ -0,0 +1,2 @@ +name = input("Enter your name") +print("Hello", name) \ No newline at end of file diff --git a/Assignment 2.2/readme.md b/Assignment 2.2/readme.md new file mode 100644 index 0000000..5e7f17b --- /dev/null +++ b/Assignment 2.2/readme.md @@ -0,0 +1,3 @@ +Write a program that uses input to prompt a user for their name and then welcomes them. Note that input will pop up a dialog box. Enter **Sarah** in the pop-up box when you are prompted so your output will match the desired output. + +Desired Output: ``Hello Sarah`` \ No newline at end of file diff --git a/Assignment 2.3/main.py b/Assignment 2.3/main.py new file mode 100644 index 0000000..eb4b592 --- /dev/null +++ b/Assignment 2.3/main.py @@ -0,0 +1,6 @@ +hours = input("Hours Worked: ") +payrate = input("Hourly Pay Rate: ") + +pay = float(hours) * float(payrate) + +print("Pay:", pay) \ No newline at end of file diff --git a/Assignment 2.3/readme.md b/Assignment 2.3/readme.md new file mode 100644 index 0000000..ef57047 --- /dev/null +++ b/Assignment 2.3/readme.md @@ -0,0 +1 @@ +Write a program to prompt the user for hours and rate per hour using input to compute gross pay. Use 35 hours and a rate of 2.75 per hour to test the program (the pay should be 96.25). You should use input to read a string and float() to convert the string to a number. Do not worry about error checking or bad user data. diff --git a/Assignment 3.1/main.py b/Assignment 3.1/main.py new file mode 100644 index 0000000..0a06aa2 --- /dev/null +++ b/Assignment 3.1/main.py @@ -0,0 +1,15 @@ +hours_worked = input("Enter Hours Worked: ") +hours_worked = float(hours_worked) + +hourly_rate = input("Enter Hourly Rate: ") +hourly_rate = float(hourly_rate) + +total_pay = 0; + +if hours_worked > 40: + overtime_hours = hours_worked - 40 + total_pay = (hourly_rate * 40.0) + (overtime_hours * (hourly_rate * 1.5)) +elif hours_worked < 40: + total_pay = (hourly_rate * hours_worked) + +print(total_pay) \ No newline at end of file diff --git a/Assignment 3.1/readme.md b/Assignment 3.1/readme.md new file mode 100644 index 0000000..af59112 --- /dev/null +++ b/Assignment 3.1/readme.md @@ -0,0 +1 @@ +Write a program to prompt the user for hours and rate per hour using input to compute gross pay. Pay the hourly rate for the hours up to 40 and 1.5 times the hourly rate for all hours worked above 40 hours. Use **45 hours** and a **rate of 10.50 per hour** to test the program (the pay should be 498.75). You should use input to read a string and float() to convert the string to a number. Do not worry about error checking the user input - assume the user types numbers properly. \ No newline at end of file diff --git a/Assignment 3.3/main.py b/Assignment 3.3/main.py new file mode 100644 index 0000000..484e21f --- /dev/null +++ b/Assignment 3.3/main.py @@ -0,0 +1,17 @@ +score = input("Enter Score: ") + +try: + score = float(score) +except: + score = 0 + +if score >= 0.9: + print("A") +elif score >= 0.8: + print("B") +elif score >= 0.7: + print("C") +elif score >= 0.6: + print("D") +else: + print("F") \ No newline at end of file diff --git a/Assignment 3.3/readme.md b/Assignment 3.3/readme.md new file mode 100644 index 0000000..fa6b3a3 --- /dev/null +++ b/Assignment 3.3/readme.md @@ -0,0 +1,12 @@ +Write a program to prompt for a score between 0.0 and 1.0. If the score is out of range, print an error. If the score is between 0.0 and 1.0, print a grade using the following table: + +Score Grade +``` +>= 0.9 A + +>= 0.8 B +>= 0.7 C +>= 0.6 D +< 0.6 F +If the user enters a value out of range, print a suitable error message and exit. For the test, enter a score of 0.85. +``` \ No newline at end of file diff --git a/Assignment 4.6/main.py b/Assignment 4.6/main.py new file mode 100644 index 0000000..c3740af --- /dev/null +++ b/Assignment 4.6/main.py @@ -0,0 +1,25 @@ +def computepay (hours, rate): + try: + hours = float(hours) + rate = float(rate) + except: + print("Invalid hours or rate received") + return 0; + + totalpay = 0 + + if hours > 40: + # Standard Pay for first 40 hours + totalpay = (40 * rate) + + # Total Pay is added to Overtime Pay for everything over the first 40 at 1.5x the rate + totalpay = totalpay + ((hours - 40) * (rate * 1.50)) + else: + totalpay = hours * rate; + + return totalpay + +workedhours = input("Enter Hours Worked: ") +hourlyrate = input("Enter Hourly Rate: ") + +print("Pay", computepay(workedhours,hourlyrate)) \ No newline at end of file diff --git a/Assignment 4.6/readme.md b/Assignment 4.6/readme.md new file mode 100644 index 0000000..4e273b8 --- /dev/null +++ b/Assignment 4.6/readme.md @@ -0,0 +1,8 @@ +Write a program to prompt the user for hours and rate per hour using input to compute gross pay. + +Pay should be the normal rate for hours up to 40 and time-and-a-half for the hourly rate for all hours worked above 40 hours. + +Put the logic to do the computation of pay in a function called computepay() and use the function to do the computation. The function should return a value. +Use 45 hours and a rate of 10.50 per hour to test the program (the pay should be 498.75). + +You should use input to read a string and float() to convert the string to a number. Do not worry about error checking the user input unless you want to - you can assume the user types numbers properly. Do not name your variable sum or use the sum() function. \ No newline at end of file diff --git a/Assignment 5.2/main.py b/Assignment 5.2/main.py new file mode 100644 index 0000000..7ec00bf --- /dev/null +++ b/Assignment 5.2/main.py @@ -0,0 +1,32 @@ +smallest = None +largest = None + +# Start Loop until done is received and both smallest and largest have values +while True: + number = input("Please enter a number: ") + + try: + number = int(number) + except: + + if number == "done": + if smallest is None or largest is None: + print("Not Enough Information to determine smallest and largest") + else: + print("Maximum is", largest) + print("Minimum is", smallest) + break; + else: + print("Invalid input") + continue + + if smallest is None: + smallest = number + elif number < smallest: + smallest = number + + if largest is None: + largest = number + elif number > largest: + largest = number + \ No newline at end of file diff --git a/Assignment 5.2/readme.md b/Assignment 5.2/readme.md new file mode 100644 index 0000000..85aeda0 --- /dev/null +++ b/Assignment 5.2/readme.md @@ -0,0 +1,8 @@ +Write a program that repeatedly prompts a user for integer numbers until the user enters 'done'. Once 'done' is entered, print out the largest and smallest of the numbers. If the user enters anything other than a valid number catch it with a try/except and put out an appropriate message and ignore the number. Enter 7, 2, bob, 10, and 4 and match the output below. + +Desired Output: +``` +Invalid input +Maximum is 10 +Minimum is 2 +``` \ No newline at end of file diff --git a/Assignment 6.5/main.py b/Assignment 6.5/main.py new file mode 100644 index 0000000..912927c --- /dev/null +++ b/Assignment 6.5/main.py @@ -0,0 +1,7 @@ +text = "X-DSPAM-Confidence: 0.8475" + +confidence = text.find("0") + +confidence = float(text[confidence:]) + +print(confidence) \ No newline at end of file diff --git a/Assignment 6.5/readme.md b/Assignment 6.5/readme.md new file mode 100644 index 0000000..e69de29 diff --git a/Assignment 7.1/main.py b/Assignment 7.1/main.py new file mode 100644 index 0000000..dabb423 --- /dev/null +++ b/Assignment 7.1/main.py @@ -0,0 +1,8 @@ +# If accepting input to the file name: +#filename = input("Please enter the file name to read: ") + +filename = "words.txt" +filehandle = open (filename, 'r') + +for line in filehandle: + print(line.rstrip().upper()) \ No newline at end of file diff --git a/Assignment 7.1/readme.md b/Assignment 7.1/readme.md new file mode 100644 index 0000000..974baa3 --- /dev/null +++ b/Assignment 7.1/readme.md @@ -0,0 +1 @@ +7.1 Write a program that prompts for a file name, then opens that file and reads through the file, and print the contents of the file in upper case. Use the file words.txt to produce the output below. \ No newline at end of file diff --git a/Assignment 7.1/words.txt b/Assignment 7.1/words.txt new file mode 100644 index 0000000..a2f4fe0 --- /dev/null +++ b/Assignment 7.1/words.txt @@ -0,0 +1,24 @@ +Writing programs or programming is a very creative +and rewarding activity You can write programs for +many reasons ranging from making your living to solving +a difficult data analysis problem to having fun to helping +someone else solve a problem This book assumes that +{\em everyone} needs to know how to program and that once +you know how to program, you will figure out what you want +to do with your newfound skills + +We are surrounded in our daily lives with computers ranging +from laptops to cell phones We can think of these computers +as our personal assistants who can take care of many things +on our behalf The hardware in our current-day computers +is essentially built to continuously ask us the question +What would you like me to do next + +Our computers are fast and have vasts amounts of memory and +could be very helpful to us if we only knew the language to +speak to explain to the computer what we would like it to +do next If we knew this language we could tell the +computer to do tasks on our behalf that were reptitive +Interestingly, the kinds of things computers can do best +are often the kinds of things that we humans find boring +and mind-numbing diff --git a/Assignment 7.2/main.py b/Assignment 7.2/main.py new file mode 100644 index 0000000..bb0a9d4 --- /dev/null +++ b/Assignment 7.2/main.py @@ -0,0 +1,18 @@ +# Use the file name mbox-short.txt as the file name + +#fn = input("File Name: ") +fn = "mbox-short.txt" +fh = open(fn) + +count = 0 +confidence = 0 + +for line in fh: + if line.startswith("X-DSPAM-Confidence:"): + score = line.find("0") + score = float(line[score:]) + + count = count + 1 + confidence = confidence + score + +print ("Average spam confidence:", confidence / count) \ No newline at end of file diff --git a/Assignment 7.2/readme.md b/Assignment 7.2/readme.md new file mode 100644 index 0000000..279441e --- /dev/null +++ b/Assignment 7.2/readme.md @@ -0,0 +1,12 @@ +7.2 Write a program that prompts for a file name, then opens that file and reads through the file, looking for lines of the form: + +``` +X-DSPAM-Confidence: 0.8475 +``` + +Count these lines and extract the floating point values from each of the lines and compute the average of those values and produce an output as shown below. Do not use the sum() function or a variable named sum in your solution. + +Desired Output: +``` +Average spam confidence: 0.7507185185185187 +``` \ No newline at end of file diff --git a/Assignment 8.4/main.py b/Assignment 8.4/main.py new file mode 100644 index 0000000..0a4a6af --- /dev/null +++ b/Assignment 8.4/main.py @@ -0,0 +1,12 @@ +fh = open("romeo.txt") + +words = [] + +for line in fh: + for word in line.rstrip().split(): + if not word in words: + words.append(word) + +words.sort() + +print(words) \ No newline at end of file diff --git a/Assignment 8.4/readme.md b/Assignment 8.4/readme.md new file mode 100644 index 0000000..f539315 --- /dev/null +++ b/Assignment 8.4/readme.md @@ -0,0 +1,6 @@ +Open the file romeo.txt and read it line by line. For each line, split the line into a list of words using the split() method. The program should build a list of words. For each word on each line check to see if the word is already in the list and if not append it to the list. When the program completes, sort and print the resulting words in alphabetical order. + +Desired Output: +``` +['Arise', 'But', 'It', 'Juliet', 'Who', 'already', 'and', 'breaks', 'east', 'envious', 'fair', 'grief', 'is', 'kill', 'light', 'moon', 'pale', 'sick', 'soft', 'sun', 'the', 'through', 'what', 'window', 'with', 'yonder'] +``` \ No newline at end of file diff --git a/Assignment 8.4/romeo.txt b/Assignment 8.4/romeo.txt new file mode 100644 index 0000000..ebbf317 --- /dev/null +++ b/Assignment 8.4/romeo.txt @@ -0,0 +1,4 @@ +But soft what light through yonder window breaks +It is the east and Juliet is the sun +Arise fair sun and kill the envious moon +Who is already sick and pale with grief diff --git a/Assignment 8.5/main.py b/Assignment 8.5/main.py new file mode 100644 index 0000000..84648d5 --- /dev/null +++ b/Assignment 8.5/main.py @@ -0,0 +1,23 @@ +fname = input("Enter file name: ") +if len(fname) < 1: + fname = "mbox-short.txt" + +fh = open(fname) +count = 0 + +for line in fh: + line = line.split() + + # Check Length, Count from and print email + if len(line) > 0: + if line[0] == "From": + count = count + 1 + print(line[1]) + + # Iterate over words in the line and if from print email +# for word in line: +# if word == "From": +# count = count + 1 +# print(line[1]) + +print("There were", count, "lines in the file with From as the first word") diff --git a/Assignment 8.5/mbox-short.txt b/Assignment 8.5/mbox-short.txt new file mode 100644 index 0000000..684d15e --- /dev/null +++ b/Assignment 8.5/mbox-short.txt @@ -0,0 +1,1910 @@ +From stephen.marquard@uct.ac.za Sat Jan 5 09:14:16 2008 +Return-Path: +Received: from murder (mail.umich.edu [141.211.14.90]) + by frankenstein.mail.umich.edu (Cyrus v2.3.8) with LMTPA; + Sat, 05 Jan 2008 09:14:16 -0500 +X-Sieve: CMU Sieve 2.3 +Received: from murder ([unix socket]) + by mail.umich.edu (Cyrus v2.2.12) with LMTPA; + Sat, 05 Jan 2008 09:14:16 -0500 +Received: from holes.mr.itd.umich.edu (holes.mr.itd.umich.edu [141.211.14.79]) + by flawless.mail.umich.edu () with ESMTP id m05EEFR1013674; + Sat, 5 Jan 2008 09:14:15 -0500 +Received: FROM paploo.uhi.ac.uk (app1.prod.collab.uhi.ac.uk [194.35.219.184]) + BY holes.mr.itd.umich.edu ID 477F90B0.2DB2F.12494 ; + 5 Jan 2008 09:14:10 -0500 +Received: from paploo.uhi.ac.uk (localhost [127.0.0.1]) + by paploo.uhi.ac.uk (Postfix) with ESMTP id 5F919BC2F2; + Sat, 5 Jan 2008 14:10:05 +0000 (GMT) +Message-ID: <200801051412.m05ECIaH010327@nakamura.uits.iupui.edu> +Mime-Version: 1.0 +Content-Transfer-Encoding: 7bit +Received: from prod.collab.uhi.ac.uk ([194.35.219.182]) + by paploo.uhi.ac.uk (JAMES SMTP Server 2.1.3) with SMTP ID 899 + for ; + Sat, 5 Jan 2008 14:09:50 +0000 (GMT) +Received: from nakamura.uits.iupui.edu (nakamura.uits.iupui.edu [134.68.220.122]) + by shmi.uhi.ac.uk (Postfix) with ESMTP id A215243002 + for ; Sat, 5 Jan 2008 14:13:33 +0000 (GMT) +Received: from nakamura.uits.iupui.edu (localhost [127.0.0.1]) + by nakamura.uits.iupui.edu (8.12.11.20060308/8.12.11) with ESMTP id m05ECJVp010329 + for ; Sat, 5 Jan 2008 09:12:19 -0500 +Received: (from apache@localhost) + by nakamura.uits.iupui.edu (8.12.11.20060308/8.12.11/Submit) id m05ECIaH010327 + for source@collab.sakaiproject.org; Sat, 5 Jan 2008 09:12:18 -0500 +Date: Sat, 5 Jan 2008 09:12:18 -0500 +X-Authentication-Warning: nakamura.uits.iupui.edu: apache set sender to stephen.marquard@uct.ac.za using -f +To: source@collab.sakaiproject.org +From: stephen.marquard@uct.ac.za +Subject: [sakai] svn commit: r39772 - content/branches/sakai_2-5-x/content-impl/impl/src/java/org/sakaiproject/content/impl +X-Content-Type-Outer-Envelope: text/plain; charset=UTF-8 +X-Content-Type-Message-Body: text/plain; charset=UTF-8 +Content-Type: text/plain; charset=UTF-8 +X-DSPAM-Result: Innocent +X-DSPAM-Processed: Sat Jan 5 09:14:16 2008 +X-DSPAM-Confidence: 0.8475 +X-DSPAM-Probability: 0.0000 + +Details: http://source.sakaiproject.org/viewsvn/?view=rev&rev=39772 + +Author: stephen.marquard@uct.ac.za +Date: 2008-01-05 09:12:07 -0500 (Sat, 05 Jan 2008) +New Revision: 39772 + +Modified: +content/branches/sakai_2-5-x/content-impl/impl/src/java/org/sakaiproject/content/impl/ContentServiceSqlOracle.java +content/branches/sakai_2-5-x/content-impl/impl/src/java/org/sakaiproject/content/impl/DbContentService.java +Log: +SAK-12501 merge to 2-5-x: r39622, r39624:5, r39632:3 (resolve conflict from differing linebreaks for r39622) + +---------------------- +This automatic notification message was sent by Sakai Collab (https://collab.sakaiproject.org/portal) from the Source site. +You can modify how you receive notifications at My Workspace > Preferences. + + + +From louis@media.berkeley.edu Fri Jan 4 18:10:48 2008 +Return-Path: +Received: from murder (mail.umich.edu [141.211.14.97]) + by frankenstein.mail.umich.edu (Cyrus v2.3.8) with LMTPA; + Fri, 04 Jan 2008 18:10:48 -0500 +X-Sieve: CMU Sieve 2.3 +Received: from murder ([unix socket]) + by mail.umich.edu (Cyrus v2.2.12) with LMTPA; + Fri, 04 Jan 2008 18:10:48 -0500 +Received: from icestorm.mr.itd.umich.edu (icestorm.mr.itd.umich.edu [141.211.93.149]) + by sleepers.mail.umich.edu () with ESMTP id m04NAbGa029441; + Fri, 4 Jan 2008 18:10:37 -0500 +Received: FROM paploo.uhi.ac.uk (app1.prod.collab.uhi.ac.uk [194.35.219.184]) + BY icestorm.mr.itd.umich.edu ID 477EBCE3.161BB.4320 ; + 4 Jan 2008 18:10:31 -0500 +Received: from paploo.uhi.ac.uk (localhost [127.0.0.1]) + by paploo.uhi.ac.uk (Postfix) with ESMTP id 07969BB706; + Fri, 4 Jan 2008 23:10:33 +0000 (GMT) +Message-ID: <200801042308.m04N8v6O008125@nakamura.uits.iupui.edu> +Mime-Version: 1.0 +Content-Transfer-Encoding: 7bit +Received: from prod.collab.uhi.ac.uk ([194.35.219.182]) + by paploo.uhi.ac.uk (JAMES SMTP Server 2.1.3) with SMTP ID 710 + for ; + Fri, 4 Jan 2008 23:10:10 +0000 (GMT) +Received: from nakamura.uits.iupui.edu (nakamura.uits.iupui.edu [134.68.220.122]) + by shmi.uhi.ac.uk (Postfix) with ESMTP id 4BA2F42F57 + for ; Fri, 4 Jan 2008 23:10:10 +0000 (GMT) +Received: from nakamura.uits.iupui.edu (localhost [127.0.0.1]) + by nakamura.uits.iupui.edu (8.12.11.20060308/8.12.11) with ESMTP id m04N8vHG008127 + for ; Fri, 4 Jan 2008 18:08:57 -0500 +Received: (from apache@localhost) + by nakamura.uits.iupui.edu (8.12.11.20060308/8.12.11/Submit) id m04N8v6O008125 + for source@collab.sakaiproject.org; Fri, 4 Jan 2008 18:08:57 -0500 +Date: Fri, 4 Jan 2008 18:08:57 -0500 +X-Authentication-Warning: nakamura.uits.iupui.edu: apache set sender to louis@media.berkeley.edu using -f +To: source@collab.sakaiproject.org +From: louis@media.berkeley.edu +Subject: [sakai] svn commit: r39771 - in bspace/site-manage/sakai_2-4-x/site-manage-tool/tool/src: bundle java/org/sakaiproject/site/tool +X-Content-Type-Outer-Envelope: text/plain; charset=UTF-8 +X-Content-Type-Message-Body: text/plain; charset=UTF-8 +Content-Type: text/plain; charset=UTF-8 +X-DSPAM-Result: Innocent +X-DSPAM-Processed: Fri Jan 4 18:10:48 2008 +X-DSPAM-Confidence: 0.6178 +X-DSPAM-Probability: 0.0000 + +Details: http://source.sakaiproject.org/viewsvn/?view=rev&rev=39771 + +Author: louis@media.berkeley.edu +Date: 2008-01-04 18:08:50 -0500 (Fri, 04 Jan 2008) +New Revision: 39771 + +Modified: +bspace/site-manage/sakai_2-4-x/site-manage-tool/tool/src/bundle/sitesetupgeneric.properties +bspace/site-manage/sakai_2-4-x/site-manage-tool/tool/src/java/org/sakaiproject/site/tool/SiteAction.java +Log: +BSP-1415 New (Guest) user Notification + +---------------------- +This automatic notification message was sent by Sakai Collab (https://collab.sakaiproject.org/portal) from the Source site. +You can modify how you receive notifications at My Workspace > Preferences. + + + +From zqian@umich.edu Fri Jan 4 16:10:39 2008 +Return-Path: +Received: from murder (mail.umich.edu [141.211.14.25]) + by frankenstein.mail.umich.edu (Cyrus v2.3.8) with LMTPA; + Fri, 04 Jan 2008 16:10:39 -0500 +X-Sieve: CMU Sieve 2.3 +Received: from murder ([unix socket]) + by mail.umich.edu (Cyrus v2.2.12) with LMTPA; + Fri, 04 Jan 2008 16:10:39 -0500 +Received: from ghostbusters.mr.itd.umich.edu (ghostbusters.mr.itd.umich.edu [141.211.93.144]) + by panther.mail.umich.edu () with ESMTP id m04LAcZw014275; + Fri, 4 Jan 2008 16:10:38 -0500 +Received: FROM paploo.uhi.ac.uk (app1.prod.collab.uhi.ac.uk [194.35.219.184]) + BY ghostbusters.mr.itd.umich.edu ID 477EA0C6.A0214.25480 ; + 4 Jan 2008 16:10:33 -0500 +Received: from paploo.uhi.ac.uk (localhost [127.0.0.1]) + by paploo.uhi.ac.uk (Postfix) with ESMTP id C48CDBB490; + Fri, 4 Jan 2008 21:10:31 +0000 (GMT) +Message-ID: <200801042109.m04L92hb007923@nakamura.uits.iupui.edu> +Mime-Version: 1.0 +Content-Transfer-Encoding: 7bit +Received: from prod.collab.uhi.ac.uk ([194.35.219.182]) + by paploo.uhi.ac.uk (JAMES SMTP Server 2.1.3) with SMTP ID 906 + for ; + Fri, 4 Jan 2008 21:10:18 +0000 (GMT) +Received: from nakamura.uits.iupui.edu (nakamura.uits.iupui.edu [134.68.220.122]) + by shmi.uhi.ac.uk (Postfix) with ESMTP id 7D13042F71 + for ; Fri, 4 Jan 2008 21:10:14 +0000 (GMT) +Received: from nakamura.uits.iupui.edu (localhost [127.0.0.1]) + by nakamura.uits.iupui.edu (8.12.11.20060308/8.12.11) with ESMTP id m04L927E007925 + for ; Fri, 4 Jan 2008 16:09:02 -0500 +Received: (from apache@localhost) + by nakamura.uits.iupui.edu (8.12.11.20060308/8.12.11/Submit) id m04L92hb007923 + for source@collab.sakaiproject.org; Fri, 4 Jan 2008 16:09:02 -0500 +Date: Fri, 4 Jan 2008 16:09:02 -0500 +X-Authentication-Warning: nakamura.uits.iupui.edu: apache set sender to zqian@umich.edu using -f +To: source@collab.sakaiproject.org +From: zqian@umich.edu +Subject: [sakai] svn commit: r39770 - site-manage/branches/sakai_2-5-x/site-manage-tool/tool/src/webapp/vm/sitesetup +X-Content-Type-Outer-Envelope: text/plain; charset=UTF-8 +X-Content-Type-Message-Body: text/plain; charset=UTF-8 +Content-Type: text/plain; charset=UTF-8 +X-DSPAM-Result: Innocent +X-DSPAM-Processed: Fri Jan 4 16:10:39 2008 +X-DSPAM-Confidence: 0.6961 +X-DSPAM-Probability: 0.0000 + +Details: http://source.sakaiproject.org/viewsvn/?view=rev&rev=39770 + +Author: zqian@umich.edu +Date: 2008-01-04 16:09:01 -0500 (Fri, 04 Jan 2008) +New Revision: 39770 + +Modified: +site-manage/branches/sakai_2-5-x/site-manage-tool/tool/src/webapp/vm/sitesetup/chef_site-siteInfo-list.vm +Log: +merge fix to SAK-9996 into 2-5-x branch: svn merge -r 39687:39688 https://source.sakaiproject.org/svn/site-manage/trunk/ + +---------------------- +This automatic notification message was sent by Sakai Collab (https://collab.sakaiproject.org/portal) from the Source site. +You can modify how you receive notifications at My Workspace > Preferences. + + + +From rjlowe@iupui.edu Fri Jan 4 15:46:24 2008 +Return-Path: +Received: from murder (mail.umich.edu [141.211.14.25]) + by frankenstein.mail.umich.edu (Cyrus v2.3.8) with LMTPA; + Fri, 04 Jan 2008 15:46:24 -0500 +X-Sieve: CMU Sieve 2.3 +Received: from murder ([unix socket]) + by mail.umich.edu (Cyrus v2.2.12) with LMTPA; + Fri, 04 Jan 2008 15:46:24 -0500 +Received: from dreamcatcher.mr.itd.umich.edu (dreamcatcher.mr.itd.umich.edu [141.211.14.43]) + by panther.mail.umich.edu () with ESMTP id m04KkNbx032077; + Fri, 4 Jan 2008 15:46:23 -0500 +Received: FROM paploo.uhi.ac.uk (app1.prod.collab.uhi.ac.uk [194.35.219.184]) + BY dreamcatcher.mr.itd.umich.edu ID 477E9B13.2F3BC.22965 ; + 4 Jan 2008 15:46:13 -0500 +Received: from paploo.uhi.ac.uk (localhost [127.0.0.1]) + by paploo.uhi.ac.uk (Postfix) with ESMTP id 4AE03BB552; + Fri, 4 Jan 2008 20:46:13 +0000 (GMT) +Message-ID: <200801042044.m04Kiem3007881@nakamura.uits.iupui.edu> +Mime-Version: 1.0 +Content-Transfer-Encoding: 7bit +Received: from prod.collab.uhi.ac.uk ([194.35.219.182]) + by paploo.uhi.ac.uk (JAMES SMTP Server 2.1.3) with SMTP ID 38 + for ; + Fri, 4 Jan 2008 20:45:56 +0000 (GMT) +Received: from nakamura.uits.iupui.edu (nakamura.uits.iupui.edu [134.68.220.122]) + by shmi.uhi.ac.uk (Postfix) with ESMTP id A55D242F57 + for ; Fri, 4 Jan 2008 20:45:52 +0000 (GMT) +Received: from nakamura.uits.iupui.edu (localhost [127.0.0.1]) + by nakamura.uits.iupui.edu (8.12.11.20060308/8.12.11) with ESMTP id m04KieqE007883 + for ; Fri, 4 Jan 2008 15:44:40 -0500 +Received: (from apache@localhost) + by nakamura.uits.iupui.edu (8.12.11.20060308/8.12.11/Submit) id m04Kiem3007881 + for source@collab.sakaiproject.org; Fri, 4 Jan 2008 15:44:40 -0500 +Date: Fri, 4 Jan 2008 15:44:40 -0500 +X-Authentication-Warning: nakamura.uits.iupui.edu: apache set sender to rjlowe@iupui.edu using -f +To: source@collab.sakaiproject.org +From: rjlowe@iupui.edu +Subject: [sakai] svn commit: r39769 - in gradebook/trunk/app/ui/src: java/org/sakaiproject/tool/gradebook/ui/helpers/beans java/org/sakaiproject/tool/gradebook/ui/helpers/producers webapp/WEB-INF webapp/WEB-INF/bundle +X-Content-Type-Outer-Envelope: text/plain; charset=UTF-8 +X-Content-Type-Message-Body: text/plain; charset=UTF-8 +Content-Type: text/plain; charset=UTF-8 +X-DSPAM-Result: Innocent +X-DSPAM-Processed: Fri Jan 4 15:46:24 2008 +X-DSPAM-Confidence: 0.7565 +X-DSPAM-Probability: 0.0000 + +Details: http://source.sakaiproject.org/viewsvn/?view=rev&rev=39769 + +Author: rjlowe@iupui.edu +Date: 2008-01-04 15:44:39 -0500 (Fri, 04 Jan 2008) +New Revision: 39769 + +Modified: +gradebook/trunk/app/ui/src/java/org/sakaiproject/tool/gradebook/ui/helpers/beans/AssignmentGradeRecordBean.java +gradebook/trunk/app/ui/src/java/org/sakaiproject/tool/gradebook/ui/helpers/producers/GradeGradebookItemProducer.java +gradebook/trunk/app/ui/src/webapp/WEB-INF/applicationContext.xml +gradebook/trunk/app/ui/src/webapp/WEB-INF/bundle/messages.properties +gradebook/trunk/app/ui/src/webapp/WEB-INF/requestContext.xml +Log: +SAK-12180 - Fixed errors with grading helper + +---------------------- +This automatic notification message was sent by Sakai Collab (https://collab.sakaiproject.org/portal) from the Source site. +You can modify how you receive notifications at My Workspace > Preferences. + + + +From zqian@umich.edu Fri Jan 4 15:03:18 2008 +Return-Path: +Received: from murder (mail.umich.edu [141.211.14.46]) + by frankenstein.mail.umich.edu (Cyrus v2.3.8) with LMTPA; + Fri, 04 Jan 2008 15:03:18 -0500 +X-Sieve: CMU Sieve 2.3 +Received: from murder ([unix socket]) + by mail.umich.edu (Cyrus v2.2.12) with LMTPA; + Fri, 04 Jan 2008 15:03:18 -0500 +Received: from firestarter.mr.itd.umich.edu (firestarter.mr.itd.umich.edu [141.211.14.83]) + by fan.mail.umich.edu () with ESMTP id m04K3HGF006563; + Fri, 4 Jan 2008 15:03:17 -0500 +Received: FROM paploo.uhi.ac.uk (app1.prod.collab.uhi.ac.uk [194.35.219.184]) + BY firestarter.mr.itd.umich.edu ID 477E9100.8F7F4.1590 ; + 4 Jan 2008 15:03:15 -0500 +Received: from paploo.uhi.ac.uk (localhost [127.0.0.1]) + by paploo.uhi.ac.uk (Postfix) with ESMTP id 57770BB477; + Fri, 4 Jan 2008 20:03:09 +0000 (GMT) +Message-ID: <200801042001.m04K1cO0007738@nakamura.uits.iupui.edu> +Mime-Version: 1.0 +Content-Transfer-Encoding: 7bit +Received: from prod.collab.uhi.ac.uk ([194.35.219.182]) + by paploo.uhi.ac.uk (JAMES SMTP Server 2.1.3) with SMTP ID 622 + for ; + Fri, 4 Jan 2008 20:02:46 +0000 (GMT) +Received: from nakamura.uits.iupui.edu (nakamura.uits.iupui.edu [134.68.220.122]) + by shmi.uhi.ac.uk (Postfix) with ESMTP id AB4D042F4D + for ; Fri, 4 Jan 2008 20:02:50 +0000 (GMT) +Received: from nakamura.uits.iupui.edu (localhost [127.0.0.1]) + by nakamura.uits.iupui.edu (8.12.11.20060308/8.12.11) with ESMTP id m04K1cXv007740 + for ; Fri, 4 Jan 2008 15:01:38 -0500 +Received: (from apache@localhost) + by nakamura.uits.iupui.edu (8.12.11.20060308/8.12.11/Submit) id m04K1cO0007738 + for source@collab.sakaiproject.org; Fri, 4 Jan 2008 15:01:38 -0500 +Date: Fri, 4 Jan 2008 15:01:38 -0500 +X-Authentication-Warning: nakamura.uits.iupui.edu: apache set sender to zqian@umich.edu using -f +To: source@collab.sakaiproject.org +From: zqian@umich.edu +Subject: [sakai] svn commit: r39766 - site-manage/branches/sakai_2-4-x/site-manage-tool/tool/src/java/org/sakaiproject/site/tool +X-Content-Type-Outer-Envelope: text/plain; charset=UTF-8 +X-Content-Type-Message-Body: text/plain; charset=UTF-8 +Content-Type: text/plain; charset=UTF-8 +X-DSPAM-Result: Innocent +X-DSPAM-Processed: Fri Jan 4 15:03:18 2008 +X-DSPAM-Confidence: 0.7626 +X-DSPAM-Probability: 0.0000 + +Details: http://source.sakaiproject.org/viewsvn/?view=rev&rev=39766 + +Author: zqian@umich.edu +Date: 2008-01-04 15:01:37 -0500 (Fri, 04 Jan 2008) +New Revision: 39766 + +Modified: +site-manage/branches/sakai_2-4-x/site-manage-tool/tool/src/java/org/sakaiproject/site/tool/SiteAction.java +Log: +merge fix to SAK-10788 into site-manage 2.4.x branch: + +Sakai Source Repository #38024 Wed Nov 07 14:54:46 MST 2007 zqian@umich.edu Fix to SAK-10788: If a provided id in a couse site is fake or doesn't provide any user information, Site Info appears to be like project site with empty participant list + +Watch for enrollments object being null and concatenate provider ids when there are more than one. +Files Changed +MODIFY /site-manage/trunk/site-manage-tool/tool/src/java/org/sakaiproject/site/tool/SiteAction.java + + + + +---------------------- +This automatic notification message was sent by Sakai Collab (https://collab.sakaiproject.org/portal) from the Source site. +You can modify how you receive notifications at My Workspace > Preferences. + + + +From rjlowe@iupui.edu Fri Jan 4 14:50:18 2008 +Return-Path: +Received: from murder (mail.umich.edu [141.211.14.93]) + by frankenstein.mail.umich.edu (Cyrus v2.3.8) with LMTPA; + Fri, 04 Jan 2008 14:50:18 -0500 +X-Sieve: CMU Sieve 2.3 +Received: from murder ([unix socket]) + by mail.umich.edu (Cyrus v2.2.12) with LMTPA; + Fri, 04 Jan 2008 14:50:18 -0500 +Received: from eyewitness.mr.itd.umich.edu (eyewitness.mr.itd.umich.edu [141.211.93.142]) + by mission.mail.umich.edu () with ESMTP id m04JoHJi019755; + Fri, 4 Jan 2008 14:50:17 -0500 +Received: FROM paploo.uhi.ac.uk (app1.prod.collab.uhi.ac.uk [194.35.219.184]) + BY eyewitness.mr.itd.umich.edu ID 477E8DF2.67B91.5278 ; + 4 Jan 2008 14:50:13 -0500 +Received: from paploo.uhi.ac.uk (localhost [127.0.0.1]) + by paploo.uhi.ac.uk (Postfix) with ESMTP id 2D1B9BB492; + Fri, 4 Jan 2008 19:47:10 +0000 (GMT) +Message-ID: <200801041948.m04JmdwO007705@nakamura.uits.iupui.edu> +Mime-Version: 1.0 +Content-Transfer-Encoding: 7bit +Received: from prod.collab.uhi.ac.uk ([194.35.219.182]) + by paploo.uhi.ac.uk (JAMES SMTP Server 2.1.3) with SMTP ID 960 + for ; + Fri, 4 Jan 2008 19:46:50 +0000 (GMT) +Received: from nakamura.uits.iupui.edu (nakamura.uits.iupui.edu [134.68.220.122]) + by shmi.uhi.ac.uk (Postfix) with ESMTP id B3E6742F4A + for ; Fri, 4 Jan 2008 19:49:51 +0000 (GMT) +Received: from nakamura.uits.iupui.edu (localhost [127.0.0.1]) + by nakamura.uits.iupui.edu (8.12.11.20060308/8.12.11) with ESMTP id m04JmeV9007707 + for ; Fri, 4 Jan 2008 14:48:40 -0500 +Received: (from apache@localhost) + by nakamura.uits.iupui.edu (8.12.11.20060308/8.12.11/Submit) id m04JmdwO007705 + for source@collab.sakaiproject.org; Fri, 4 Jan 2008 14:48:39 -0500 +Date: Fri, 4 Jan 2008 14:48:39 -0500 +X-Authentication-Warning: nakamura.uits.iupui.edu: apache set sender to rjlowe@iupui.edu using -f +To: source@collab.sakaiproject.org +From: rjlowe@iupui.edu +Subject: [sakai] svn commit: r39765 - in gradebook/trunk/app: business/src/java/org/sakaiproject/tool/gradebook/business business/src/java/org/sakaiproject/tool/gradebook/business/impl ui ui/src/java/org/sakaiproject/tool/gradebook/ui/helpers/beans ui/src/java/org/sakaiproject/tool/gradebook/ui/helpers/entity ui/src/java/org/sakaiproject/tool/gradebook/ui/helpers/params ui/src/java/org/sakaiproject/tool/gradebook/ui/helpers/producers ui/src/webapp/WEB-INF ui/src/webapp/WEB-INF/bundle ui/src/webapp/content/templates +X-Content-Type-Outer-Envelope: text/plain; charset=UTF-8 +X-Content-Type-Message-Body: text/plain; charset=UTF-8 +Content-Type: text/plain; charset=UTF-8 +X-DSPAM-Result: Innocent +X-DSPAM-Processed: Fri Jan 4 14:50:18 2008 +X-DSPAM-Confidence: 0.7556 +X-DSPAM-Probability: 0.0000 + +Details: http://source.sakaiproject.org/viewsvn/?view=rev&rev=39765 + +Author: rjlowe@iupui.edu +Date: 2008-01-04 14:48:37 -0500 (Fri, 04 Jan 2008) +New Revision: 39765 + +Added: +gradebook/trunk/app/ui/src/java/org/sakaiproject/tool/gradebook/ui/helpers/beans/AssignmentGradeRecordBean.java +gradebook/trunk/app/ui/src/java/org/sakaiproject/tool/gradebook/ui/helpers/beans/AssignmentGradeRecordCreator.java +gradebook/trunk/app/ui/src/java/org/sakaiproject/tool/gradebook/ui/helpers/entity/GradebookEntryGradeEntityProvider.java +gradebook/trunk/app/ui/src/java/org/sakaiproject/tool/gradebook/ui/helpers/params/GradeGradebookItemViewParams.java +gradebook/trunk/app/ui/src/java/org/sakaiproject/tool/gradebook/ui/helpers/producers/GradeGradebookItemProducer.java +gradebook/trunk/app/ui/src/webapp/content/templates/grade-gradebook-item.html +Modified: +gradebook/trunk/app/business/src/java/org/sakaiproject/tool/gradebook/business/GradebookManager.java +gradebook/trunk/app/business/src/java/org/sakaiproject/tool/gradebook/business/impl/GradebookManagerHibernateImpl.java +gradebook/trunk/app/ui/pom.xml +gradebook/trunk/app/ui/src/java/org/sakaiproject/tool/gradebook/ui/helpers/beans/GradebookItemBean.java +gradebook/trunk/app/ui/src/java/org/sakaiproject/tool/gradebook/ui/helpers/entity/GradebookEntryEntityProvider.java +gradebook/trunk/app/ui/src/java/org/sakaiproject/tool/gradebook/ui/helpers/producers/AddGradebookItemProducer.java +gradebook/trunk/app/ui/src/webapp/WEB-INF/applicationContext.xml +gradebook/trunk/app/ui/src/webapp/WEB-INF/bundle/messages.properties +gradebook/trunk/app/ui/src/webapp/WEB-INF/requestContext.xml +Log: +SAK-12180 - New helper tool to grade an assignment + +---------------------- +This automatic notification message was sent by Sakai Collab (https://collab.sakaiproject.org/portal) from the Source site. +You can modify how you receive notifications at My Workspace > Preferences. + + + +From cwen@iupui.edu Fri Jan 4 11:37:30 2008 +Return-Path: +Received: from murder (mail.umich.edu [141.211.14.46]) + by frankenstein.mail.umich.edu (Cyrus v2.3.8) with LMTPA; + Fri, 04 Jan 2008 11:37:30 -0500 +X-Sieve: CMU Sieve 2.3 +Received: from murder ([unix socket]) + by mail.umich.edu (Cyrus v2.2.12) with LMTPA; + Fri, 04 Jan 2008 11:37:30 -0500 +Received: from tadpole.mr.itd.umich.edu (tadpole.mr.itd.umich.edu [141.211.14.72]) + by fan.mail.umich.edu () with ESMTP id m04GbT9x022078; + Fri, 4 Jan 2008 11:37:29 -0500 +Received: FROM paploo.uhi.ac.uk (app1.prod.collab.uhi.ac.uk [194.35.219.184]) + BY tadpole.mr.itd.umich.edu ID 477E60B2.82756.9904 ; + 4 Jan 2008 11:37:09 -0500 +Received: from paploo.uhi.ac.uk (localhost [127.0.0.1]) + by paploo.uhi.ac.uk (Postfix) with ESMTP id 8D13DBB001; + Fri, 4 Jan 2008 16:37:07 +0000 (GMT) +Message-ID: <200801041635.m04GZQGZ007313@nakamura.uits.iupui.edu> +Mime-Version: 1.0 +Content-Transfer-Encoding: 7bit +Received: from prod.collab.uhi.ac.uk ([194.35.219.182]) + by paploo.uhi.ac.uk (JAMES SMTP Server 2.1.3) with SMTP ID 120 + for ; + Fri, 4 Jan 2008 16:36:40 +0000 (GMT) +Received: from nakamura.uits.iupui.edu (nakamura.uits.iupui.edu [134.68.220.122]) + by shmi.uhi.ac.uk (Postfix) with ESMTP id D430B42E42 + for ; Fri, 4 Jan 2008 16:36:37 +0000 (GMT) +Received: from nakamura.uits.iupui.edu (localhost [127.0.0.1]) + by nakamura.uits.iupui.edu (8.12.11.20060308/8.12.11) with ESMTP id m04GZQ7W007315 + for ; Fri, 4 Jan 2008 11:35:26 -0500 +Received: (from apache@localhost) + by nakamura.uits.iupui.edu (8.12.11.20060308/8.12.11/Submit) id m04GZQGZ007313 + for source@collab.sakaiproject.org; Fri, 4 Jan 2008 11:35:26 -0500 +Date: Fri, 4 Jan 2008 11:35:26 -0500 +X-Authentication-Warning: nakamura.uits.iupui.edu: apache set sender to cwen@iupui.edu using -f +To: source@collab.sakaiproject.org +From: cwen@iupui.edu +Subject: [sakai] svn commit: r39764 - in msgcntr/trunk/messageforums-app/src/java/org/sakaiproject/tool/messageforums: . ui +X-Content-Type-Outer-Envelope: text/plain; charset=UTF-8 +X-Content-Type-Message-Body: text/plain; charset=UTF-8 +Content-Type: text/plain; charset=UTF-8 +X-DSPAM-Result: Innocent +X-DSPAM-Processed: Fri Jan 4 11:37:30 2008 +X-DSPAM-Confidence: 0.7002 +X-DSPAM-Probability: 0.0000 + +Details: http://source.sakaiproject.org/viewsvn/?view=rev&rev=39764 + +Author: cwen@iupui.edu +Date: 2008-01-04 11:35:25 -0500 (Fri, 04 Jan 2008) +New Revision: 39764 + +Modified: +msgcntr/trunk/messageforums-app/src/java/org/sakaiproject/tool/messageforums/PrivateMessagesTool.java +msgcntr/trunk/messageforums-app/src/java/org/sakaiproject/tool/messageforums/ui/PrivateMessageDecoratedBean.java +Log: +unmerge Xingtang's checkin for SAK-12488. + +svn merge -r39558:39557 https://source.sakaiproject.org/svn/msgcntr/trunk +U messageforums-app/src/java/org/sakaiproject/tool/messageforums/PrivateMessagesTool.java +U messageforums-app/src/java/org/sakaiproject/tool/messageforums/ui/PrivateMessageDecoratedBean.java + +svn log -r 39558 +------------------------------------------------------------------------ +r39558 | hu2@iupui.edu | 2007-12-20 15:25:38 -0500 (Thu, 20 Dec 2007) | 3 lines + +SAK-12488 +when send a message to yourself. click reply to all, cc row should be null. +http://jira.sakaiproject.org/jira/browse/SAK-12488 +------------------------------------------------------------------------ + + +---------------------- +This automatic notification message was sent by Sakai Collab (https://collab.sakaiproject.org/portal) from the Source site. +You can modify how you receive notifications at My Workspace > Preferences. + + + +From cwen@iupui.edu Fri Jan 4 11:35:08 2008 +Return-Path: +Received: from murder (mail.umich.edu [141.211.14.46]) + by frankenstein.mail.umich.edu (Cyrus v2.3.8) with LMTPA; + Fri, 04 Jan 2008 11:35:08 -0500 +X-Sieve: CMU Sieve 2.3 +Received: from murder ([unix socket]) + by mail.umich.edu (Cyrus v2.2.12) with LMTPA; + Fri, 04 Jan 2008 11:35:08 -0500 +Received: from it.mr.itd.umich.edu (it.mr.itd.umich.edu [141.211.93.151]) + by fan.mail.umich.edu () with ESMTP id m04GZ6lt020480; + Fri, 4 Jan 2008 11:35:06 -0500 +Received: FROM paploo.uhi.ac.uk (app1.prod.collab.uhi.ac.uk [194.35.219.184]) + BY it.mr.itd.umich.edu ID 477E6033.6469D.21870 ; + 4 Jan 2008 11:35:02 -0500 +Received: from paploo.uhi.ac.uk (localhost [127.0.0.1]) + by paploo.uhi.ac.uk (Postfix) with ESMTP id E40FABAE5B; + Fri, 4 Jan 2008 16:34:38 +0000 (GMT) +Message-ID: <200801041633.m04GX6eG007292@nakamura.uits.iupui.edu> +Mime-Version: 1.0 +Content-Transfer-Encoding: 7bit +Received: from prod.collab.uhi.ac.uk ([194.35.219.182]) + by paploo.uhi.ac.uk (JAMES SMTP Server 2.1.3) with SMTP ID 697 + for ; + Fri, 4 Jan 2008 16:34:01 +0000 (GMT) +Received: from nakamura.uits.iupui.edu (nakamura.uits.iupui.edu [134.68.220.122]) + by shmi.uhi.ac.uk (Postfix) with ESMTP id 1CD0C42E42 + for ; Fri, 4 Jan 2008 16:34:17 +0000 (GMT) +Received: from nakamura.uits.iupui.edu (localhost [127.0.0.1]) + by nakamura.uits.iupui.edu (8.12.11.20060308/8.12.11) with ESMTP id m04GX6Y3007294 + for ; Fri, 4 Jan 2008 11:33:06 -0500 +Received: (from apache@localhost) + by nakamura.uits.iupui.edu (8.12.11.20060308/8.12.11/Submit) id m04GX6eG007292 + for source@collab.sakaiproject.org; Fri, 4 Jan 2008 11:33:06 -0500 +Date: Fri, 4 Jan 2008 11:33:06 -0500 +X-Authentication-Warning: nakamura.uits.iupui.edu: apache set sender to cwen@iupui.edu using -f +To: source@collab.sakaiproject.org +From: cwen@iupui.edu +Subject: [sakai] svn commit: r39763 - in msgcntr/trunk: messageforums-api/src/bundle/org/sakaiproject/api/app/messagecenter/bundle messageforums-app/src/java/org/sakaiproject/tool/messageforums +X-Content-Type-Outer-Envelope: text/plain; charset=UTF-8 +X-Content-Type-Message-Body: text/plain; charset=UTF-8 +Content-Type: text/plain; charset=UTF-8 +X-DSPAM-Result: Innocent +X-DSPAM-Processed: Fri Jan 4 11:35:08 2008 +X-DSPAM-Confidence: 0.7615 +X-DSPAM-Probability: 0.0000 + +Details: http://source.sakaiproject.org/viewsvn/?view=rev&rev=39763 + +Author: cwen@iupui.edu +Date: 2008-01-04 11:33:05 -0500 (Fri, 04 Jan 2008) +New Revision: 39763 + +Modified: +msgcntr/trunk/messageforums-api/src/bundle/org/sakaiproject/api/app/messagecenter/bundle/Messages.properties +msgcntr/trunk/messageforums-app/src/java/org/sakaiproject/tool/messageforums/PrivateMessagesTool.java +Log: +unmerge Xingtang's check in for SAK-12484. + +svn merge -r39571:39570 https://source.sakaiproject.org/svn/msgcntr/trunk +U messageforums-api/src/bundle/org/sakaiproject/api/app/messagecenter/bundle/Messages.properties +U messageforums-app/src/java/org/sakaiproject/tool/messageforums/PrivateMessagesTool.java + +svn log -r 39571 +------------------------------------------------------------------------ +r39571 | hu2@iupui.edu | 2007-12-20 21:26:28 -0500 (Thu, 20 Dec 2007) | 3 lines + +SAK-12484 +reply all cc list should not include the current user name. +http://jira.sakaiproject.org/jira/browse/SAK-12484 +------------------------------------------------------------------------ + + +---------------------- +This automatic notification message was sent by Sakai Collab (https://collab.sakaiproject.org/portal) from the Source site. +You can modify how you receive notifications at My Workspace > Preferences. + + + +From gsilver@umich.edu Fri Jan 4 11:12:37 2008 +Return-Path: +Received: from murder (mail.umich.edu [141.211.14.25]) + by frankenstein.mail.umich.edu (Cyrus v2.3.8) with LMTPA; + Fri, 04 Jan 2008 11:12:37 -0500 +X-Sieve: CMU Sieve 2.3 +Received: from murder ([unix socket]) + by mail.umich.edu (Cyrus v2.2.12) with LMTPA; + Fri, 04 Jan 2008 11:12:37 -0500 +Received: from holes.mr.itd.umich.edu (holes.mr.itd.umich.edu [141.211.14.79]) + by panther.mail.umich.edu () with ESMTP id m04GCaHB030887; + Fri, 4 Jan 2008 11:12:36 -0500 +Received: FROM paploo.uhi.ac.uk (app1.prod.collab.uhi.ac.uk [194.35.219.184]) + BY holes.mr.itd.umich.edu ID 477E5AEB.E670B.28397 ; + 4 Jan 2008 11:12:30 -0500 +Received: from paploo.uhi.ac.uk (localhost [127.0.0.1]) + by paploo.uhi.ac.uk (Postfix) with ESMTP id 99715BAE7D; + Fri, 4 Jan 2008 16:12:27 +0000 (GMT) +Message-ID: <200801041611.m04GB1Lb007221@nakamura.uits.iupui.edu> +Mime-Version: 1.0 +Content-Transfer-Encoding: 7bit +Received: from prod.collab.uhi.ac.uk ([194.35.219.182]) + by paploo.uhi.ac.uk (JAMES SMTP Server 2.1.3) with SMTP ID 272 + for ; + Fri, 4 Jan 2008 16:12:14 +0000 (GMT) +Received: from nakamura.uits.iupui.edu (nakamura.uits.iupui.edu [134.68.220.122]) + by shmi.uhi.ac.uk (Postfix) with ESMTP id 0A6ED42DFC + for ; Fri, 4 Jan 2008 16:12:12 +0000 (GMT) +Received: from nakamura.uits.iupui.edu (localhost [127.0.0.1]) + by nakamura.uits.iupui.edu (8.12.11.20060308/8.12.11) with ESMTP id m04GB1Wt007223 + for ; Fri, 4 Jan 2008 11:11:01 -0500 +Received: (from apache@localhost) + by nakamura.uits.iupui.edu (8.12.11.20060308/8.12.11/Submit) id m04GB1Lb007221 + for source@collab.sakaiproject.org; Fri, 4 Jan 2008 11:11:01 -0500 +Date: Fri, 4 Jan 2008 11:11:01 -0500 +X-Authentication-Warning: nakamura.uits.iupui.edu: apache set sender to gsilver@umich.edu using -f +To: source@collab.sakaiproject.org +From: gsilver@umich.edu +Subject: [sakai] svn commit: r39762 - web/trunk/web-tool/tool/src/bundle +X-Content-Type-Outer-Envelope: text/plain; charset=UTF-8 +X-Content-Type-Message-Body: text/plain; charset=UTF-8 +Content-Type: text/plain; charset=UTF-8 +X-DSPAM-Result: Innocent +X-DSPAM-Processed: Fri Jan 4 11:12:37 2008 +X-DSPAM-Confidence: 0.7601 +X-DSPAM-Probability: 0.0000 + +Details: http://source.sakaiproject.org/viewsvn/?view=rev&rev=39762 + +Author: gsilver@umich.edu +Date: 2008-01-04 11:11:00 -0500 (Fri, 04 Jan 2008) +New Revision: 39762 + +Modified: +web/trunk/web-tool/tool/src/bundle/iframe.properties +Log: +SAK-12596 +http://bugs.sakaiproject.org/jira/browse/SAK-12596 +- left moot (unused) entries commented for now + +---------------------- +This automatic notification message was sent by Sakai Collab (https://collab.sakaiproject.org/portal) from the Source site. +You can modify how you receive notifications at My Workspace > Preferences. + + + +From gsilver@umich.edu Fri Jan 4 11:11:52 2008 +Return-Path: +Received: from murder (mail.umich.edu [141.211.14.36]) + by frankenstein.mail.umich.edu (Cyrus v2.3.8) with LMTPA; + Fri, 04 Jan 2008 11:11:52 -0500 +X-Sieve: CMU Sieve 2.3 +Received: from murder ([unix socket]) + by mail.umich.edu (Cyrus v2.2.12) with LMTPA; + Fri, 04 Jan 2008 11:11:52 -0500 +Received: from creepshow.mr.itd.umich.edu (creepshow.mr.itd.umich.edu [141.211.14.84]) + by godsend.mail.umich.edu () with ESMTP id m04GBqqv025330; + Fri, 4 Jan 2008 11:11:52 -0500 +Received: FROM paploo.uhi.ac.uk (app1.prod.collab.uhi.ac.uk [194.35.219.184]) + BY creepshow.mr.itd.umich.edu ID 477E5AB3.5CC32.30840 ; + 4 Jan 2008 11:11:34 -0500 +Received: from paploo.uhi.ac.uk (localhost [127.0.0.1]) + by paploo.uhi.ac.uk (Postfix) with ESMTP id 62AA4BAE46; + Fri, 4 Jan 2008 16:11:31 +0000 (GMT) +Message-ID: <200801041610.m04GA5KP007209@nakamura.uits.iupui.edu> +Mime-Version: 1.0 +Content-Transfer-Encoding: 7bit +Received: from prod.collab.uhi.ac.uk ([194.35.219.182]) + by paploo.uhi.ac.uk (JAMES SMTP Server 2.1.3) with SMTP ID 1006 + for ; + Fri, 4 Jan 2008 16:11:18 +0000 (GMT) +Received: from nakamura.uits.iupui.edu (nakamura.uits.iupui.edu [134.68.220.122]) + by shmi.uhi.ac.uk (Postfix) with ESMTP id C596A3DFA2 + for ; Fri, 4 Jan 2008 16:11:16 +0000 (GMT) +Received: from nakamura.uits.iupui.edu (localhost [127.0.0.1]) + by nakamura.uits.iupui.edu (8.12.11.20060308/8.12.11) with ESMTP id m04GA5LR007211 + for ; Fri, 4 Jan 2008 11:10:05 -0500 +Received: (from apache@localhost) + by nakamura.uits.iupui.edu (8.12.11.20060308/8.12.11/Submit) id m04GA5KP007209 + for source@collab.sakaiproject.org; Fri, 4 Jan 2008 11:10:05 -0500 +Date: Fri, 4 Jan 2008 11:10:05 -0500 +X-Authentication-Warning: nakamura.uits.iupui.edu: apache set sender to gsilver@umich.edu using -f +To: source@collab.sakaiproject.org +From: gsilver@umich.edu +Subject: [sakai] svn commit: r39761 - site/trunk/site-tool/tool/src/bundle +X-Content-Type-Outer-Envelope: text/plain; charset=UTF-8 +X-Content-Type-Message-Body: text/plain; charset=UTF-8 +Content-Type: text/plain; charset=UTF-8 +X-DSPAM-Result: Innocent +X-DSPAM-Processed: Fri Jan 4 11:11:52 2008 +X-DSPAM-Confidence: 0.7605 +X-DSPAM-Probability: 0.0000 + +Details: http://source.sakaiproject.org/viewsvn/?view=rev&rev=39761 + +Author: gsilver@umich.edu +Date: 2008-01-04 11:10:04 -0500 (Fri, 04 Jan 2008) +New Revision: 39761 + +Modified: +site/trunk/site-tool/tool/src/bundle/admin.properties +Log: +SAK-12595 +http://bugs.sakaiproject.org/jira/browse/SAK-12595 +- left moot (unused) entries commented for now + +---------------------- +This automatic notification message was sent by Sakai Collab (https://collab.sakaiproject.org/portal) from the Source site. +You can modify how you receive notifications at My Workspace > Preferences. + + + +From zqian@umich.edu Fri Jan 4 11:11:03 2008 +Return-Path: +Received: from murder (mail.umich.edu [141.211.14.97]) + by frankenstein.mail.umich.edu (Cyrus v2.3.8) with LMTPA; + Fri, 04 Jan 2008 11:11:03 -0500 +X-Sieve: CMU Sieve 2.3 +Received: from murder ([unix socket]) + by mail.umich.edu (Cyrus v2.2.12) with LMTPA; + Fri, 04 Jan 2008 11:11:03 -0500 +Received: from carrie.mr.itd.umich.edu (carrie.mr.itd.umich.edu [141.211.93.152]) + by sleepers.mail.umich.edu () with ESMTP id m04GB3Vg011502; + Fri, 4 Jan 2008 11:11:03 -0500 +Received: FROM paploo.uhi.ac.uk (app1.prod.collab.uhi.ac.uk [194.35.219.184]) + BY carrie.mr.itd.umich.edu ID 477E5A8D.B378F.24200 ; + 4 Jan 2008 11:10:56 -0500 +Received: from paploo.uhi.ac.uk (localhost [127.0.0.1]) + by paploo.uhi.ac.uk (Postfix) with ESMTP id C7251BAD44; + Fri, 4 Jan 2008 16:10:53 +0000 (GMT) +Message-ID: <200801041609.m04G9EuX007197@nakamura.uits.iupui.edu> +Mime-Version: 1.0 +Content-Transfer-Encoding: 7bit +Received: from prod.collab.uhi.ac.uk ([194.35.219.182]) + by paploo.uhi.ac.uk (JAMES SMTP Server 2.1.3) with SMTP ID 483 + for ; + Fri, 4 Jan 2008 16:10:27 +0000 (GMT) +Received: from nakamura.uits.iupui.edu (nakamura.uits.iupui.edu [134.68.220.122]) + by shmi.uhi.ac.uk (Postfix) with ESMTP id 2E7043DFA2 + for ; Fri, 4 Jan 2008 16:10:26 +0000 (GMT) +Received: from nakamura.uits.iupui.edu (localhost [127.0.0.1]) + by nakamura.uits.iupui.edu (8.12.11.20060308/8.12.11) with ESMTP id m04G9Eqg007199 + for ; Fri, 4 Jan 2008 11:09:15 -0500 +Received: (from apache@localhost) + by nakamura.uits.iupui.edu (8.12.11.20060308/8.12.11/Submit) id m04G9EuX007197 + for source@collab.sakaiproject.org; Fri, 4 Jan 2008 11:09:14 -0500 +Date: Fri, 4 Jan 2008 11:09:14 -0500 +X-Authentication-Warning: nakamura.uits.iupui.edu: apache set sender to zqian@umich.edu using -f +To: source@collab.sakaiproject.org +From: zqian@umich.edu +Subject: [sakai] svn commit: r39760 - in site-manage/trunk/site-manage-tool/tool/src: java/org/sakaiproject/site/tool webapp/vm/sitesetup +X-Content-Type-Outer-Envelope: text/plain; charset=UTF-8 +X-Content-Type-Message-Body: text/plain; charset=UTF-8 +Content-Type: text/plain; charset=UTF-8 +X-DSPAM-Result: Innocent +X-DSPAM-Processed: Fri Jan 4 11:11:03 2008 +X-DSPAM-Confidence: 0.6959 +X-DSPAM-Probability: 0.0000 + +Details: http://source.sakaiproject.org/viewsvn/?view=rev&rev=39760 + +Author: zqian@umich.edu +Date: 2008-01-04 11:09:12 -0500 (Fri, 04 Jan 2008) +New Revision: 39760 + +Modified: +site-manage/trunk/site-manage-tool/tool/src/java/org/sakaiproject/site/tool/SiteAction.java +site-manage/trunk/site-manage-tool/tool/src/webapp/vm/sitesetup/chef_site-siteInfo-list.vm +Log: +fix to SAK-10911: Refactor use of site.upd, site.upd.site.mbrship and site.upd.grp.mbrship permissions + +---------------------- +This automatic notification message was sent by Sakai Collab (https://collab.sakaiproject.org/portal) from the Source site. +You can modify how you receive notifications at My Workspace > Preferences. + + + +From gsilver@umich.edu Fri Jan 4 11:10:22 2008 +Return-Path: +Received: from murder (mail.umich.edu [141.211.14.39]) + by frankenstein.mail.umich.edu (Cyrus v2.3.8) with LMTPA; + Fri, 04 Jan 2008 11:10:22 -0500 +X-Sieve: CMU Sieve 2.3 +Received: from murder ([unix socket]) + by mail.umich.edu (Cyrus v2.2.12) with LMTPA; + Fri, 04 Jan 2008 11:10:22 -0500 +Received: from holes.mr.itd.umich.edu (holes.mr.itd.umich.edu [141.211.14.79]) + by faithful.mail.umich.edu () with ESMTP id m04GAL9k010604; + Fri, 4 Jan 2008 11:10:21 -0500 +Received: FROM paploo.uhi.ac.uk (app1.prod.collab.uhi.ac.uk [194.35.219.184]) + BY holes.mr.itd.umich.edu ID 477E5A67.34350.23015 ; + 4 Jan 2008 11:10:18 -0500 +Received: from paploo.uhi.ac.uk (localhost [127.0.0.1]) + by paploo.uhi.ac.uk (Postfix) with ESMTP id 98D04BAD43; + Fri, 4 Jan 2008 16:10:11 +0000 (GMT) +Message-ID: <200801041608.m04G8d7w007184@nakamura.uits.iupui.edu> +Mime-Version: 1.0 +Content-Transfer-Encoding: 7bit +Received: from prod.collab.uhi.ac.uk ([194.35.219.182]) + by paploo.uhi.ac.uk (JAMES SMTP Server 2.1.3) with SMTP ID 966 + for ; + Fri, 4 Jan 2008 16:09:51 +0000 (GMT) +Received: from nakamura.uits.iupui.edu (nakamura.uits.iupui.edu [134.68.220.122]) + by shmi.uhi.ac.uk (Postfix) with ESMTP id 9F89542DD0 + for ; Fri, 4 Jan 2008 16:09:50 +0000 (GMT) +Received: from nakamura.uits.iupui.edu (localhost [127.0.0.1]) + by nakamura.uits.iupui.edu (8.12.11.20060308/8.12.11) with ESMTP id m04G8dXN007186 + for ; Fri, 4 Jan 2008 11:08:39 -0500 +Received: (from apache@localhost) + by nakamura.uits.iupui.edu (8.12.11.20060308/8.12.11/Submit) id m04G8d7w007184 + for source@collab.sakaiproject.org; Fri, 4 Jan 2008 11:08:39 -0500 +Date: Fri, 4 Jan 2008 11:08:39 -0500 +X-Authentication-Warning: nakamura.uits.iupui.edu: apache set sender to gsilver@umich.edu using -f +To: source@collab.sakaiproject.org +From: gsilver@umich.edu +Subject: [sakai] svn commit: r39759 - mailarchive/trunk/mailarchive-tool/tool/src/bundle +X-Content-Type-Outer-Envelope: text/plain; charset=UTF-8 +X-Content-Type-Message-Body: text/plain; charset=UTF-8 +Content-Type: text/plain; charset=UTF-8 +X-DSPAM-Result: Innocent +X-DSPAM-Processed: Fri Jan 4 11:10:22 2008 +X-DSPAM-Confidence: 0.7606 +X-DSPAM-Probability: 0.0000 + +Details: http://source.sakaiproject.org/viewsvn/?view=rev&rev=39759 + +Author: gsilver@umich.edu +Date: 2008-01-04 11:08:38 -0500 (Fri, 04 Jan 2008) +New Revision: 39759 + +Modified: +mailarchive/trunk/mailarchive-tool/tool/src/bundle/email.properties +Log: +SAK-12592 +http://bugs.sakaiproject.org/jira/browse/SAK-12592 +- left moot (unused) entries commented for now + +---------------------- +This automatic notification message was sent by Sakai Collab (https://collab.sakaiproject.org/portal) from the Source site. +You can modify how you receive notifications at My Workspace > Preferences. + + + +From wagnermr@iupui.edu Fri Jan 4 10:38:42 2008 +Return-Path: +Received: from murder (mail.umich.edu [141.211.14.90]) + by frankenstein.mail.umich.edu (Cyrus v2.3.8) with LMTPA; + Fri, 04 Jan 2008 10:38:42 -0500 +X-Sieve: CMU Sieve 2.3 +Received: from murder ([unix socket]) + by mail.umich.edu (Cyrus v2.2.12) with LMTPA; + Fri, 04 Jan 2008 10:38:42 -0500 +Received: from shining.mr.itd.umich.edu (shining.mr.itd.umich.edu [141.211.93.153]) + by flawless.mail.umich.edu () with ESMTP id m04Fcfjm012313; + Fri, 4 Jan 2008 10:38:41 -0500 +Received: FROM paploo.uhi.ac.uk (app1.prod.collab.uhi.ac.uk [194.35.219.184]) + BY shining.mr.itd.umich.edu ID 477E52FA.E6C6E.24093 ; + 4 Jan 2008 10:38:37 -0500 +Received: from paploo.uhi.ac.uk (localhost [127.0.0.1]) + by paploo.uhi.ac.uk (Postfix) with ESMTP id 6A39594CD2; + Fri, 4 Jan 2008 15:37:36 +0000 (GMT) +Message-ID: <200801041537.m04Fb6Ci007092@nakamura.uits.iupui.edu> +Mime-Version: 1.0 +Content-Transfer-Encoding: 7bit +Received: from prod.collab.uhi.ac.uk ([194.35.219.182]) + by paploo.uhi.ac.uk (JAMES SMTP Server 2.1.3) with SMTP ID 690 + for ; + Fri, 4 Jan 2008 15:37:21 +0000 (GMT) +Received: from nakamura.uits.iupui.edu (nakamura.uits.iupui.edu [134.68.220.122]) + by shmi.uhi.ac.uk (Postfix) with ESMTP id CEFA037ACE + for ; Fri, 4 Jan 2008 15:38:17 +0000 (GMT) +Received: from nakamura.uits.iupui.edu (localhost [127.0.0.1]) + by nakamura.uits.iupui.edu (8.12.11.20060308/8.12.11) with ESMTP id m04Fb6nh007094 + for ; Fri, 4 Jan 2008 10:37:06 -0500 +Received: (from apache@localhost) + by nakamura.uits.iupui.edu (8.12.11.20060308/8.12.11/Submit) id m04Fb6Ci007092 + for source@collab.sakaiproject.org; Fri, 4 Jan 2008 10:37:06 -0500 +Date: Fri, 4 Jan 2008 10:37:06 -0500 +X-Authentication-Warning: nakamura.uits.iupui.edu: apache set sender to wagnermr@iupui.edu using -f +To: source@collab.sakaiproject.org +From: wagnermr@iupui.edu +Subject: [sakai] svn commit: r39758 - in gradebook/trunk: app/business/src/java/org/sakaiproject/tool/gradebook/business/impl service/api/src/java/org/sakaiproject/service/gradebook/shared service/impl/src/java/org/sakaiproject/component/gradebook +X-Content-Type-Outer-Envelope: text/plain; charset=UTF-8 +X-Content-Type-Message-Body: text/plain; charset=UTF-8 +Content-Type: text/plain; charset=UTF-8 +X-DSPAM-Result: Innocent +X-DSPAM-Processed: Fri Jan 4 10:38:42 2008 +X-DSPAM-Confidence: 0.7559 +X-DSPAM-Probability: 0.0000 + +Details: http://source.sakaiproject.org/viewsvn/?view=rev&rev=39758 + +Author: wagnermr@iupui.edu +Date: 2008-01-04 10:37:04 -0500 (Fri, 04 Jan 2008) +New Revision: 39758 + +Modified: +gradebook/trunk/app/business/src/java/org/sakaiproject/tool/gradebook/business/impl/GradebookManagerHibernateImpl.java +gradebook/trunk/service/api/src/java/org/sakaiproject/service/gradebook/shared/GradebookService.java +gradebook/trunk/service/impl/src/java/org/sakaiproject/component/gradebook/GradebookServiceHibernateImpl.java +Log: +SAK-12175 +http://bugs.sakaiproject.org/jira/browse/SAK-12175 +Create methods required for gb integration with the Assignment2 tool +getGradeDefinitionForStudentForItem + +---------------------- +This automatic notification message was sent by Sakai Collab (https://collab.sakaiproject.org/portal) from the Source site. +You can modify how you receive notifications at My Workspace > Preferences. + + + +From zqian@umich.edu Fri Jan 4 10:17:43 2008 +Return-Path: +Received: from murder (mail.umich.edu [141.211.14.97]) + by frankenstein.mail.umich.edu (Cyrus v2.3.8) with LMTPA; + Fri, 04 Jan 2008 10:17:43 -0500 +X-Sieve: CMU Sieve 2.3 +Received: from murder ([unix socket]) + by mail.umich.edu (Cyrus v2.2.12) with LMTPA; + Fri, 04 Jan 2008 10:17:42 -0500 +Received: from creepshow.mr.itd.umich.edu (creepshow.mr.itd.umich.edu [141.211.14.84]) + by sleepers.mail.umich.edu () with ESMTP id m04FHgfs011536; + Fri, 4 Jan 2008 10:17:42 -0500 +Received: FROM paploo.uhi.ac.uk (app1.prod.collab.uhi.ac.uk [194.35.219.184]) + BY creepshow.mr.itd.umich.edu ID 477E4E0F.CCA4B.926 ; + 4 Jan 2008 10:17:38 -0500 +Received: from paploo.uhi.ac.uk (localhost [127.0.0.1]) + by paploo.uhi.ac.uk (Postfix) with ESMTP id BD02DBAC64; + Fri, 4 Jan 2008 15:17:34 +0000 (GMT) +Message-ID: <200801041515.m04FFv42007050@nakamura.uits.iupui.edu> +Mime-Version: 1.0 +Content-Transfer-Encoding: 7bit +Received: from prod.collab.uhi.ac.uk ([194.35.219.182]) + by paploo.uhi.ac.uk (JAMES SMTP Server 2.1.3) with SMTP ID 25 + for ; + Fri, 4 Jan 2008 15:17:11 +0000 (GMT) +Received: from nakamura.uits.iupui.edu (nakamura.uits.iupui.edu [134.68.220.122]) + by shmi.uhi.ac.uk (Postfix) with ESMTP id 5B396236B9 + for ; Fri, 4 Jan 2008 15:17:08 +0000 (GMT) +Received: from nakamura.uits.iupui.edu (localhost [127.0.0.1]) + by nakamura.uits.iupui.edu (8.12.11.20060308/8.12.11) with ESMTP id m04FFv85007052 + for ; Fri, 4 Jan 2008 10:15:57 -0500 +Received: (from apache@localhost) + by nakamura.uits.iupui.edu (8.12.11.20060308/8.12.11/Submit) id m04FFv42007050 + for source@collab.sakaiproject.org; Fri, 4 Jan 2008 10:15:57 -0500 +Date: Fri, 4 Jan 2008 10:15:57 -0500 +X-Authentication-Warning: nakamura.uits.iupui.edu: apache set sender to zqian@umich.edu using -f +To: source@collab.sakaiproject.org +From: zqian@umich.edu +Subject: [sakai] svn commit: r39757 - in assignment/trunk: assignment-impl/impl/src/java/org/sakaiproject/assignment/impl assignment-tool/tool/src/webapp/vm/assignment +X-Content-Type-Outer-Envelope: text/plain; charset=UTF-8 +X-Content-Type-Message-Body: text/plain; charset=UTF-8 +Content-Type: text/plain; charset=UTF-8 +X-DSPAM-Result: Innocent +X-DSPAM-Processed: Fri Jan 4 10:17:42 2008 +X-DSPAM-Confidence: 0.7605 +X-DSPAM-Probability: 0.0000 + +Details: http://source.sakaiproject.org/viewsvn/?view=rev&rev=39757 + +Author: zqian@umich.edu +Date: 2008-01-04 10:15:54 -0500 (Fri, 04 Jan 2008) +New Revision: 39757 + +Modified: +assignment/trunk/assignment-impl/impl/src/java/org/sakaiproject/assignment/impl/BaseAssignmentService.java +assignment/trunk/assignment-tool/tool/src/webapp/vm/assignment/chef_assignments_instructor_list_submissions.vm +Log: +fix to SAK-12604:Don't show groups/sections filter if the site doesn't have any + +---------------------- +This automatic notification message was sent by Sakai Collab (https://collab.sakaiproject.org/portal) from the Source site. +You can modify how you receive notifications at My Workspace > Preferences. + + + +From antranig@caret.cam.ac.uk Fri Jan 4 10:04:14 2008 +Return-Path: +Received: from murder (mail.umich.edu [141.211.14.25]) + by frankenstein.mail.umich.edu (Cyrus v2.3.8) with LMTPA; + Fri, 04 Jan 2008 10:04:14 -0500 +X-Sieve: CMU Sieve 2.3 +Received: from murder ([unix socket]) + by mail.umich.edu (Cyrus v2.2.12) with LMTPA; + Fri, 04 Jan 2008 10:04:14 -0500 +Received: from holes.mr.itd.umich.edu (holes.mr.itd.umich.edu [141.211.14.79]) + by panther.mail.umich.edu () with ESMTP id m04F4Dci015108; + Fri, 4 Jan 2008 10:04:13 -0500 +Received: FROM paploo.uhi.ac.uk (app1.prod.collab.uhi.ac.uk [194.35.219.184]) + BY holes.mr.itd.umich.edu ID 477E4AE3.D7AF.31669 ; + 4 Jan 2008 10:04:05 -0500 +Received: from paploo.uhi.ac.uk (localhost [127.0.0.1]) + by paploo.uhi.ac.uk (Postfix) with ESMTP id 933E3BAC17; + Fri, 4 Jan 2008 15:04:00 +0000 (GMT) +Message-ID: <200801041502.m04F21Jo007031@nakamura.uits.iupui.edu> +Mime-Version: 1.0 +Content-Transfer-Encoding: 7bit +Received: from prod.collab.uhi.ac.uk ([194.35.219.182]) + by paploo.uhi.ac.uk (JAMES SMTP Server 2.1.3) with SMTP ID 32 + for ; + Fri, 4 Jan 2008 15:03:15 +0000 (GMT) +Received: from nakamura.uits.iupui.edu (nakamura.uits.iupui.edu [134.68.220.122]) + by shmi.uhi.ac.uk (Postfix) with ESMTP id AC2F6236B9 + for ; Fri, 4 Jan 2008 15:03:12 +0000 (GMT) +Received: from nakamura.uits.iupui.edu (localhost [127.0.0.1]) + by nakamura.uits.iupui.edu (8.12.11.20060308/8.12.11) with ESMTP id m04F21hn007033 + for ; Fri, 4 Jan 2008 10:02:01 -0500 +Received: (from apache@localhost) + by nakamura.uits.iupui.edu (8.12.11.20060308/8.12.11/Submit) id m04F21Jo007031 + for source@collab.sakaiproject.org; Fri, 4 Jan 2008 10:02:01 -0500 +Date: Fri, 4 Jan 2008 10:02:01 -0500 +X-Authentication-Warning: nakamura.uits.iupui.edu: apache set sender to antranig@caret.cam.ac.uk using -f +To: source@collab.sakaiproject.org +From: antranig@caret.cam.ac.uk +Subject: [sakai] svn commit: r39756 - in component/branches/SAK-12166/component-api/component/src/java/org/sakaiproject/component: impl impl/spring/support impl/spring/support/dynamic impl/support util +X-Content-Type-Outer-Envelope: text/plain; charset=UTF-8 +X-Content-Type-Message-Body: text/plain; charset=UTF-8 +Content-Type: text/plain; charset=UTF-8 +X-DSPAM-Result: Innocent +X-DSPAM-Processed: Fri Jan 4 10:04:14 2008 +X-DSPAM-Confidence: 0.6932 +X-DSPAM-Probability: 0.0000 + +Details: http://source.sakaiproject.org/viewsvn/?view=rev&rev=39756 + +Author: antranig@caret.cam.ac.uk +Date: 2008-01-04 10:01:40 -0500 (Fri, 04 Jan 2008) +New Revision: 39756 + +Added: +component/branches/SAK-12166/component-api/component/src/java/org/sakaiproject/component/impl/spring/support/dynamic/ +component/branches/SAK-12166/component-api/component/src/java/org/sakaiproject/component/impl/spring/support/dynamic/DynamicComponentManager.java +component/branches/SAK-12166/component-api/component/src/java/org/sakaiproject/component/impl/support/ +component/branches/SAK-12166/component-api/component/src/java/org/sakaiproject/component/impl/support/DynamicComponentRecord.java +component/branches/SAK-12166/component-api/component/src/java/org/sakaiproject/component/impl/support/DynamicJARManager.java +component/branches/SAK-12166/component-api/component/src/java/org/sakaiproject/component/impl/support/JARRecord.java +component/branches/SAK-12166/component-api/component/src/java/org/sakaiproject/component/util/ByteToCharBase64.java +component/branches/SAK-12166/component-api/component/src/java/org/sakaiproject/component/util/FileUtil.java +component/branches/SAK-12166/component-api/component/src/java/org/sakaiproject/component/util/RecordFileIO.java +component/branches/SAK-12166/component-api/component/src/java/org/sakaiproject/component/util/RecordReader.java +component/branches/SAK-12166/component-api/component/src/java/org/sakaiproject/component/util/RecordWriter.java +component/branches/SAK-12166/component-api/component/src/java/org/sakaiproject/component/util/StreamDigestor.java +Modified: +component/branches/SAK-12166/component-api/component/src/java/org/sakaiproject/component/impl/spring/support/ComponentsLoaderImpl.java +Log: +Temporary commit of incomplete work on JAR caching + +---------------------- +This automatic notification message was sent by Sakai Collab (https://collab.sakaiproject.org/portal) from the Source site. +You can modify how you receive notifications at My Workspace > Preferences. + + + +From gopal.ramasammycook@gmail.com Fri Jan 4 09:05:31 2008 +Return-Path: +Received: from murder (mail.umich.edu [141.211.14.90]) + by frankenstein.mail.umich.edu (Cyrus v2.3.8) with LMTPA; + Fri, 04 Jan 2008 09:05:31 -0500 +X-Sieve: CMU Sieve 2.3 +Received: from murder ([unix socket]) + by mail.umich.edu (Cyrus v2.2.12) with LMTPA; + Fri, 04 Jan 2008 09:05:31 -0500 +Received: from guys.mr.itd.umich.edu (guys.mr.itd.umich.edu [141.211.14.76]) + by flawless.mail.umich.edu () with ESMTP id m04E5U3C029277; + Fri, 4 Jan 2008 09:05:30 -0500 +Received: FROM paploo.uhi.ac.uk (app1.prod.collab.uhi.ac.uk [194.35.219.184]) + BY guys.mr.itd.umich.edu ID 477E3D23.EE2E7.5237 ; + 4 Jan 2008 09:05:26 -0500 +Received: from paploo.uhi.ac.uk (localhost [127.0.0.1]) + by paploo.uhi.ac.uk (Postfix) with ESMTP id 33C7856DC0; + Fri, 4 Jan 2008 14:05:26 +0000 (GMT) +Message-ID: <200801041403.m04E3psW006926@nakamura.uits.iupui.edu> +Mime-Version: 1.0 +Content-Transfer-Encoding: 7bit +Received: from prod.collab.uhi.ac.uk ([194.35.219.182]) + by paploo.uhi.ac.uk (JAMES SMTP Server 2.1.3) with SMTP ID 575 + for ; + Fri, 4 Jan 2008 14:05:04 +0000 (GMT) +Received: from nakamura.uits.iupui.edu (nakamura.uits.iupui.edu [134.68.220.122]) + by shmi.uhi.ac.uk (Postfix) with ESMTP id 3C0261D617 + for ; Fri, 4 Jan 2008 14:05:03 +0000 (GMT) +Received: from nakamura.uits.iupui.edu (localhost [127.0.0.1]) + by nakamura.uits.iupui.edu (8.12.11.20060308/8.12.11) with ESMTP id m04E3pQS006928 + for ; Fri, 4 Jan 2008 09:03:52 -0500 +Received: (from apache@localhost) + by nakamura.uits.iupui.edu (8.12.11.20060308/8.12.11/Submit) id m04E3psW006926 + for source@collab.sakaiproject.org; Fri, 4 Jan 2008 09:03:51 -0500 +Date: Fri, 4 Jan 2008 09:03:51 -0500 +X-Authentication-Warning: nakamura.uits.iupui.edu: apache set sender to gopal.ramasammycook@gmail.com using -f +To: source@collab.sakaiproject.org +From: gopal.ramasammycook@gmail.com +Subject: [sakai] svn commit: r39755 - in sam/branches/SAK-12065: samigo-api/src/java/org/sakaiproject/tool/assessment/shared/api/grading samigo-app/src/java/org/sakaiproject/tool/assessment/ui/bean/evaluation samigo-app/src/java/org/sakaiproject/tool/assessment/ui/listener/evaluation samigo-services/src/java/org/sakaiproject/tool/assessment/facade samigo-services/src/java/org/sakaiproject/tool/assessment/integration/helper/ifc samigo-services/src/java/org/sakaiproject/tool/assessment/integration/helper/integrated samigo-services/src/java/org/sakaiproject/tool/assessment/integration/helper/standalone samigo-services/src/java/org/sakaiproject/tool/assessment/shared/impl/grading +X-Content-Type-Outer-Envelope: text/plain; charset=UTF-8 +X-Content-Type-Message-Body: text/plain; charset=UTF-8 +Content-Type: text/plain; charset=UTF-8 +X-DSPAM-Result: Innocent +X-DSPAM-Processed: Fri Jan 4 09:05:31 2008 +X-DSPAM-Confidence: 0.7558 +X-DSPAM-Probability: 0.0000 + +Details: http://source.sakaiproject.org/viewsvn/?view=rev&rev=39755 + +Author: gopal.ramasammycook@gmail.com +Date: 2008-01-04 09:02:54 -0500 (Fri, 04 Jan 2008) +New Revision: 39755 + +Modified: +sam/branches/SAK-12065/samigo-api/src/java/org/sakaiproject/tool/assessment/shared/api/grading/GradingSectionAwareServiceAPI.java +sam/branches/SAK-12065/samigo-app/src/java/org/sakaiproject/tool/assessment/ui/bean/evaluation/QuestionScoresBean.java +sam/branches/SAK-12065/samigo-app/src/java/org/sakaiproject/tool/assessment/ui/bean/evaluation/SubmissionStatusBean.java +sam/branches/SAK-12065/samigo-app/src/java/org/sakaiproject/tool/assessment/ui/bean/evaluation/TotalScoresBean.java +sam/branches/SAK-12065/samigo-app/src/java/org/sakaiproject/tool/assessment/ui/listener/evaluation/SubmissionStatusListener.java +sam/branches/SAK-12065/samigo-services/src/java/org/sakaiproject/tool/assessment/facade/PublishedAssessmentFacadeQueries.java +sam/branches/SAK-12065/samigo-services/src/java/org/sakaiproject/tool/assessment/facade/PublishedAssessmentFacadeQueriesAPI.java +sam/branches/SAK-12065/samigo-services/src/java/org/sakaiproject/tool/assessment/integration/helper/ifc/SectionAwareServiceHelper.java +sam/branches/SAK-12065/samigo-services/src/java/org/sakaiproject/tool/assessment/integration/helper/integrated/SectionAwareServiceHelperImpl.java +sam/branches/SAK-12065/samigo-services/src/java/org/sakaiproject/tool/assessment/integration/helper/standalone/SectionAwareServiceHelperImpl.java +sam/branches/SAK-12065/samigo-services/src/java/org/sakaiproject/tool/assessment/shared/impl/grading/GradingSectionAwareServiceImpl.java +Log: +SAK-12065 Gopal - Samigo Group Release. SubmissionStatus/TotalScores/Questions View filter. + +---------------------- +This automatic notification message was sent by Sakai Collab (https://collab.sakaiproject.org/portal) from the Source site. +You can modify how you receive notifications at My Workspace > Preferences. + + + +From david.horwitz@uct.ac.za Fri Jan 4 07:02:32 2008 +Return-Path: +Received: from murder (mail.umich.edu [141.211.14.39]) + by frankenstein.mail.umich.edu (Cyrus v2.3.8) with LMTPA; + Fri, 04 Jan 2008 07:02:32 -0500 +X-Sieve: CMU Sieve 2.3 +Received: from murder ([unix socket]) + by mail.umich.edu (Cyrus v2.2.12) with LMTPA; + Fri, 04 Jan 2008 07:02:32 -0500 +Received: from guys.mr.itd.umich.edu (guys.mr.itd.umich.edu [141.211.14.76]) + by faithful.mail.umich.edu () with ESMTP id m04C2VN7026678; + Fri, 4 Jan 2008 07:02:31 -0500 +Received: FROM paploo.uhi.ac.uk (app1.prod.collab.uhi.ac.uk [194.35.219.184]) + BY guys.mr.itd.umich.edu ID 477E2050.C2599.3263 ; + 4 Jan 2008 07:02:27 -0500 +Received: from paploo.uhi.ac.uk (localhost [127.0.0.1]) + by paploo.uhi.ac.uk (Postfix) with ESMTP id 6497FBA906; + Fri, 4 Jan 2008 12:02:11 +0000 (GMT) +Message-ID: <200801041200.m04C0gfK006793@nakamura.uits.iupui.edu> +Mime-Version: 1.0 +Content-Transfer-Encoding: 7bit +Received: from prod.collab.uhi.ac.uk ([194.35.219.182]) + by paploo.uhi.ac.uk (JAMES SMTP Server 2.1.3) with SMTP ID 611 + for ; + Fri, 4 Jan 2008 12:01:53 +0000 (GMT) +Received: from nakamura.uits.iupui.edu (nakamura.uits.iupui.edu [134.68.220.122]) + by shmi.uhi.ac.uk (Postfix) with ESMTP id 5296342D3C + for ; Fri, 4 Jan 2008 12:01:53 +0000 (GMT) +Received: from nakamura.uits.iupui.edu (localhost [127.0.0.1]) + by nakamura.uits.iupui.edu (8.12.11.20060308/8.12.11) with ESMTP id m04C0gnm006795 + for ; Fri, 4 Jan 2008 07:00:42 -0500 +Received: (from apache@localhost) + by nakamura.uits.iupui.edu (8.12.11.20060308/8.12.11/Submit) id m04C0gfK006793 + for source@collab.sakaiproject.org; Fri, 4 Jan 2008 07:00:42 -0500 +Date: Fri, 4 Jan 2008 07:00:42 -0500 +X-Authentication-Warning: nakamura.uits.iupui.edu: apache set sender to david.horwitz@uct.ac.za using -f +To: source@collab.sakaiproject.org +From: david.horwitz@uct.ac.za +Subject: [sakai] svn commit: r39754 - in polls/branches/sakai_2-5-x: . tool tool/src/java/org/sakaiproject/poll/tool tool/src/java/org/sakaiproject/poll/tool/evolvers tool/src/webapp/WEB-INF +X-Content-Type-Outer-Envelope: text/plain; charset=UTF-8 +X-Content-Type-Message-Body: text/plain; charset=UTF-8 +Content-Type: text/plain; charset=UTF-8 +X-DSPAM-Result: Innocent +X-DSPAM-Processed: Fri Jan 4 07:02:32 2008 +X-DSPAM-Confidence: 0.6526 +X-DSPAM-Probability: 0.0000 + +Details: http://source.sakaiproject.org/viewsvn/?view=rev&rev=39754 + +Author: david.horwitz@uct.ac.za +Date: 2008-01-04 07:00:10 -0500 (Fri, 04 Jan 2008) +New Revision: 39754 + +Added: +polls/branches/sakai_2-5-x/tool/src/java/org/sakaiproject/poll/tool/evolvers/ +polls/branches/sakai_2-5-x/tool/src/java/org/sakaiproject/poll/tool/evolvers/SakaiFCKTextEvolver.java +Removed: +polls/branches/sakai_2-5-x/tool/src/java/org/sakaiproject/poll/tool/evolvers/SakaiFCKTextEvolver.java +Modified: +polls/branches/sakai_2-5-x/.classpath +polls/branches/sakai_2-5-x/tool/pom.xml +polls/branches/sakai_2-5-x/tool/src/webapp/WEB-INF/requestContext.xml +Log: +svn log -r39753 https://source.sakaiproject.org/svn/polls/trunk +------------------------------------------------------------------------ +r39753 | david.horwitz@uct.ac.za | 2008-01-04 13:05:51 +0200 (Fri, 04 Jan 2008) | 1 line + +SAK-12228 implmented workaround sugested by AB - needs to be tested against a trunk build +------------------------------------------------------------------------ +dhorwitz@david-horwitz-6:~/branchManagemnt/sakai_2-5-x> svn merge -c39753 https://source.sakaiproject.org/svn/polls/trunk polls/ +U polls/.classpath +A polls/tool/src/java/org/sakaiproject/poll/tool/evolvers +A polls/tool/src/java/org/sakaiproject/poll/tool/evolvers/SakaiFCKTextEvolver.java +C polls/tool/src/webapp/WEB-INF/requestContext.xml +U polls/tool/pom.xml + +dhorwitz@david-horwitz-6:~/branchManagemnt/sakai_2-5-x> svn resolved polls/tool/src/webapp/WEB-INF/requestContext.xml +Resolved conflicted state of 'polls/tool/src/webapp/WEB-INF/requestContext.xml + + +---------------------- +This automatic notification message was sent by Sakai Collab (https://collab.sakaiproject.org/portal) from the Source site. +You can modify how you receive notifications at My Workspace > Preferences. + + + +From david.horwitz@uct.ac.za Fri Jan 4 06:08:27 2008 +Return-Path: +Received: from murder (mail.umich.edu [141.211.14.98]) + by frankenstein.mail.umich.edu (Cyrus v2.3.8) with LMTPA; + Fri, 04 Jan 2008 06:08:27 -0500 +X-Sieve: CMU Sieve 2.3 +Received: from murder ([unix socket]) + by mail.umich.edu (Cyrus v2.2.12) with LMTPA; + Fri, 04 Jan 2008 06:08:27 -0500 +Received: from firestarter.mr.itd.umich.edu (firestarter.mr.itd.umich.edu [141.211.14.83]) + by casino.mail.umich.edu () with ESMTP id m04B8Qw9001368; + Fri, 4 Jan 2008 06:08:26 -0500 +Received: FROM paploo.uhi.ac.uk (app1.prod.collab.uhi.ac.uk [194.35.219.184]) + BY firestarter.mr.itd.umich.edu ID 477E13A5.30FC0.24054 ; + 4 Jan 2008 06:08:23 -0500 +Received: from paploo.uhi.ac.uk (localhost [127.0.0.1]) + by paploo.uhi.ac.uk (Postfix) with ESMTP id 784A476D7B; + Fri, 4 Jan 2008 11:08:12 +0000 (GMT) +Message-ID: <200801041106.m04B6lK3006677@nakamura.uits.iupui.edu> +Mime-Version: 1.0 +Content-Transfer-Encoding: 7bit +Received: from prod.collab.uhi.ac.uk ([194.35.219.182]) + by paploo.uhi.ac.uk (JAMES SMTP Server 2.1.3) with SMTP ID 585 + for ; + Fri, 4 Jan 2008 11:07:56 +0000 (GMT) +Received: from nakamura.uits.iupui.edu (nakamura.uits.iupui.edu [134.68.220.122]) + by shmi.uhi.ac.uk (Postfix) with ESMTP id 1CACC42D0C + for ; Fri, 4 Jan 2008 11:07:58 +0000 (GMT) +Received: from nakamura.uits.iupui.edu (localhost [127.0.0.1]) + by nakamura.uits.iupui.edu (8.12.11.20060308/8.12.11) with ESMTP id m04B6lWM006679 + for ; Fri, 4 Jan 2008 06:06:47 -0500 +Received: (from apache@localhost) + by nakamura.uits.iupui.edu (8.12.11.20060308/8.12.11/Submit) id m04B6lK3006677 + for source@collab.sakaiproject.org; Fri, 4 Jan 2008 06:06:47 -0500 +Date: Fri, 4 Jan 2008 06:06:47 -0500 +X-Authentication-Warning: nakamura.uits.iupui.edu: apache set sender to david.horwitz@uct.ac.za using -f +To: source@collab.sakaiproject.org +From: david.horwitz@uct.ac.za +Subject: [sakai] svn commit: r39753 - in polls/trunk: . tool tool/src/java/org/sakaiproject/poll/tool tool/src/java/org/sakaiproject/poll/tool/evolvers tool/src/webapp/WEB-INF +X-Content-Type-Outer-Envelope: text/plain; charset=UTF-8 +X-Content-Type-Message-Body: text/plain; charset=UTF-8 +Content-Type: text/plain; charset=UTF-8 +X-DSPAM-Result: Innocent +X-DSPAM-Processed: Fri Jan 4 06:08:27 2008 +X-DSPAM-Confidence: 0.6948 +X-DSPAM-Probability: 0.0000 + +Details: http://source.sakaiproject.org/viewsvn/?view=rev&rev=39753 + +Author: david.horwitz@uct.ac.za +Date: 2008-01-04 06:05:51 -0500 (Fri, 04 Jan 2008) +New Revision: 39753 + +Added: +polls/trunk/tool/src/java/org/sakaiproject/poll/tool/evolvers/ +polls/trunk/tool/src/java/org/sakaiproject/poll/tool/evolvers/SakaiFCKTextEvolver.java +Modified: +polls/trunk/.classpath +polls/trunk/tool/pom.xml +polls/trunk/tool/src/webapp/WEB-INF/requestContext.xml +Log: +SAK-12228 implmented workaround sugested by AB - needs to be tested against a trunk build + +---------------------- +This automatic notification message was sent by Sakai Collab (https://collab.sakaiproject.org/portal) from the Source site. +You can modify how you receive notifications at My Workspace > Preferences. + + + +From david.horwitz@uct.ac.za Fri Jan 4 04:49:08 2008 +Return-Path: +Received: from murder (mail.umich.edu [141.211.14.92]) + by frankenstein.mail.umich.edu (Cyrus v2.3.8) with LMTPA; + Fri, 04 Jan 2008 04:49:08 -0500 +X-Sieve: CMU Sieve 2.3 +Received: from murder ([unix socket]) + by mail.umich.edu (Cyrus v2.2.12) with LMTPA; + Fri, 04 Jan 2008 04:49:08 -0500 +Received: from galaxyquest.mr.itd.umich.edu (galaxyquest.mr.itd.umich.edu [141.211.93.145]) + by score.mail.umich.edu () with ESMTP id m049n60G017588; + Fri, 4 Jan 2008 04:49:06 -0500 +Received: FROM paploo.uhi.ac.uk (app1.prod.collab.uhi.ac.uk [194.35.219.184]) + BY galaxyquest.mr.itd.umich.edu ID 477E010C.48C2.10259 ; + 4 Jan 2008 04:49:03 -0500 +Received: from paploo.uhi.ac.uk (localhost [127.0.0.1]) + by paploo.uhi.ac.uk (Postfix) with ESMTP id 254CC8CDEE; + Fri, 4 Jan 2008 09:48:55 +0000 (GMT) +Message-ID: <200801040947.m049lUxo006517@nakamura.uits.iupui.edu> +Mime-Version: 1.0 +Content-Transfer-Encoding: 7bit +Received: from prod.collab.uhi.ac.uk ([194.35.219.182]) + by paploo.uhi.ac.uk (JAMES SMTP Server 2.1.3) with SMTP ID 246 + for ; + Fri, 4 Jan 2008 09:48:36 +0000 (GMT) +Received: from nakamura.uits.iupui.edu (nakamura.uits.iupui.edu [134.68.220.122]) + by shmi.uhi.ac.uk (Postfix) with ESMTP id 8C13342C92 + for ; Fri, 4 Jan 2008 09:48:40 +0000 (GMT) +Received: from nakamura.uits.iupui.edu (localhost [127.0.0.1]) + by nakamura.uits.iupui.edu (8.12.11.20060308/8.12.11) with ESMTP id m049lU3P006519 + for ; Fri, 4 Jan 2008 04:47:30 -0500 +Received: (from apache@localhost) + by nakamura.uits.iupui.edu (8.12.11.20060308/8.12.11/Submit) id m049lUxo006517 + for source@collab.sakaiproject.org; Fri, 4 Jan 2008 04:47:30 -0500 +Date: Fri, 4 Jan 2008 04:47:30 -0500 +X-Authentication-Warning: nakamura.uits.iupui.edu: apache set sender to david.horwitz@uct.ac.za using -f +To: source@collab.sakaiproject.org +From: david.horwitz@uct.ac.za +Subject: [sakai] svn commit: r39752 - in podcasts/branches/sakai_2-5-x/podcasts-app/src/webapp: css podcasts +X-Content-Type-Outer-Envelope: text/plain; charset=UTF-8 +X-Content-Type-Message-Body: text/plain; charset=UTF-8 +Content-Type: text/plain; charset=UTF-8 +X-DSPAM-Result: Innocent +X-DSPAM-Processed: Fri Jan 4 04:49:08 2008 +X-DSPAM-Confidence: 0.6528 +X-DSPAM-Probability: 0.0000 + +Details: http://source.sakaiproject.org/viewsvn/?view=rev&rev=39752 + +Author: david.horwitz@uct.ac.za +Date: 2008-01-04 04:47:16 -0500 (Fri, 04 Jan 2008) +New Revision: 39752 + +Modified: +podcasts/branches/sakai_2-5-x/podcasts-app/src/webapp/css/podcaster.css +podcasts/branches/sakai_2-5-x/podcasts-app/src/webapp/podcasts/podMain.jsp +Log: +svn log -r39641 https://source.sakaiproject.org/svn/podcasts/trunk +------------------------------------------------------------------------ +r39641 | josrodri@iupui.edu | 2007-12-28 23:44:24 +0200 (Fri, 28 Dec 2007) | 1 line + +SAK-9882: refactored podMain.jsp the right way (at least much closer to) +------------------------------------------------------------------------ + +dhorwitz@david-horwitz-6:~/branchManagemnt/sakai_2-5-x> svn merge -c39641 https://source.sakaiproject.org/svn/podcasts/trunk podcasts/ +C podcasts/podcasts-app/src/webapp/podcasts/podMain.jsp +U podcasts/podcasts-app/src/webapp/css/podcaster.css + +conflict merged manualy + + + +---------------------- +This automatic notification message was sent by Sakai Collab (https://collab.sakaiproject.org/portal) from the Source site. +You can modify how you receive notifications at My Workspace > Preferences. + + + +From david.horwitz@uct.ac.za Fri Jan 4 04:33:44 2008 +Return-Path: +Received: from murder (mail.umich.edu [141.211.14.46]) + by frankenstein.mail.umich.edu (Cyrus v2.3.8) with LMTPA; + Fri, 04 Jan 2008 04:33:44 -0500 +X-Sieve: CMU Sieve 2.3 +Received: from murder ([unix socket]) + by mail.umich.edu (Cyrus v2.2.12) with LMTPA; + Fri, 04 Jan 2008 04:33:44 -0500 +Received: from workinggirl.mr.itd.umich.edu (workinggirl.mr.itd.umich.edu [141.211.93.143]) + by fan.mail.umich.edu () with ESMTP id m049Xge3031803; + Fri, 4 Jan 2008 04:33:42 -0500 +Received: FROM paploo.uhi.ac.uk (app1.prod.collab.uhi.ac.uk [194.35.219.184]) + BY workinggirl.mr.itd.umich.edu ID 477DFD6C.75DBE.26054 ; + 4 Jan 2008 04:33:35 -0500 +Received: from paploo.uhi.ac.uk (localhost [127.0.0.1]) + by paploo.uhi.ac.uk (Postfix) with ESMTP id 6C929BA656; + Fri, 4 Jan 2008 09:33:27 +0000 (GMT) +Message-ID: <200801040932.m049W2i5006493@nakamura.uits.iupui.edu> +Mime-Version: 1.0 +Content-Transfer-Encoding: 7bit +Received: from prod.collab.uhi.ac.uk ([194.35.219.182]) + by paploo.uhi.ac.uk (JAMES SMTP Server 2.1.3) with SMTP ID 153 + for ; + Fri, 4 Jan 2008 09:33:10 +0000 (GMT) +Received: from nakamura.uits.iupui.edu (nakamura.uits.iupui.edu [134.68.220.122]) + by shmi.uhi.ac.uk (Postfix) with ESMTP id 6C69423767 + for ; Fri, 4 Jan 2008 09:33:13 +0000 (GMT) +Received: from nakamura.uits.iupui.edu (localhost [127.0.0.1]) + by nakamura.uits.iupui.edu (8.12.11.20060308/8.12.11) with ESMTP id m049W3fl006495 + for ; Fri, 4 Jan 2008 04:32:03 -0500 +Received: (from apache@localhost) + by nakamura.uits.iupui.edu (8.12.11.20060308/8.12.11/Submit) id m049W2i5006493 + for source@collab.sakaiproject.org; Fri, 4 Jan 2008 04:32:02 -0500 +Date: Fri, 4 Jan 2008 04:32:02 -0500 +X-Authentication-Warning: nakamura.uits.iupui.edu: apache set sender to david.horwitz@uct.ac.za using -f +To: source@collab.sakaiproject.org +From: david.horwitz@uct.ac.za +Subject: [sakai] svn commit: r39751 - in podcasts/branches/sakai_2-5-x/podcasts-app/src/webapp: css images podcasts +X-Content-Type-Outer-Envelope: text/plain; charset=UTF-8 +X-Content-Type-Message-Body: text/plain; charset=UTF-8 +Content-Type: text/plain; charset=UTF-8 +X-DSPAM-Result: Innocent +X-DSPAM-Processed: Fri Jan 4 04:33:44 2008 +X-DSPAM-Confidence: 0.7002 +X-DSPAM-Probability: 0.0000 + +Details: http://source.sakaiproject.org/viewsvn/?view=rev&rev=39751 + +Author: david.horwitz@uct.ac.za +Date: 2008-01-04 04:31:35 -0500 (Fri, 04 Jan 2008) +New Revision: 39751 + +Removed: +podcasts/branches/sakai_2-5-x/podcasts-app/src/webapp/images/rss-feed-icon.png +podcasts/branches/sakai_2-5-x/podcasts-app/src/webapp/podcasts/podPermissions.jsp +Modified: +podcasts/branches/sakai_2-5-x/podcasts-app/src/webapp/css/podcaster.css +podcasts/branches/sakai_2-5-x/podcasts-app/src/webapp/podcasts/podDelete.jsp +podcasts/branches/sakai_2-5-x/podcasts-app/src/webapp/podcasts/podMain.jsp +podcasts/branches/sakai_2-5-x/podcasts-app/src/webapp/podcasts/podNoResource.jsp +podcasts/branches/sakai_2-5-x/podcasts-app/src/webapp/podcasts/podOptions.jsp +Log: +svn log -r39146 https://source.sakaiproject.org/svn/podcasts/trunk +------------------------------------------------------------------------ +r39146 | josrodri@iupui.edu | 2007-12-12 21:40:33 +0200 (Wed, 12 Dec 2007) | 1 line + +SAK-9882: refactored the other pages as well to take advantage of proper jsp components as well as validation cleanup. +------------------------------------------------------------------------ +dhorwitz@david-horwitz-6:~/branchManagemnt/sakai_2-5-x> svn merge -c39146 https://source.sakaiproject.org/svn/podcasts/trunk podcasts/ +D podcasts/podcasts-app/src/webapp/podcasts/podPermissions.jsp +U podcasts/podcasts-app/src/webapp/podcasts/podDelete.jsp +U podcasts/podcasts-app/src/webapp/podcasts/podMain.jsp +U podcasts/podcasts-app/src/webapp/podcasts/podNoResource.jsp +U podcasts/podcasts-app/src/webapp/podcasts/podOptions.jsp +D podcasts/podcasts-app/src/webapp/images/rss-feed-icon.png +U podcasts/podcasts-app/src/webapp/css/podcaster.css + + + +---------------------- +This automatic notification message was sent by Sakai Collab (https://collab.sakaiproject.org/portal) from the Source site. +You can modify how you receive notifications at My Workspace > Preferences. + + + +From stephen.marquard@uct.ac.za Fri Jan 4 04:07:34 2008 +Return-Path: +Received: from murder (mail.umich.edu [141.211.14.25]) + by frankenstein.mail.umich.edu (Cyrus v2.3.8) with LMTPA; + Fri, 04 Jan 2008 04:07:34 -0500 +X-Sieve: CMU Sieve 2.3 +Received: from murder ([unix socket]) + by mail.umich.edu (Cyrus v2.2.12) with LMTPA; + Fri, 04 Jan 2008 04:07:34 -0500 +Received: from salemslot.mr.itd.umich.edu (salemslot.mr.itd.umich.edu [141.211.14.58]) + by panther.mail.umich.edu () with ESMTP id m0497WAN027902; + Fri, 4 Jan 2008 04:07:32 -0500 +Received: FROM paploo.uhi.ac.uk (app1.prod.collab.uhi.ac.uk [194.35.219.184]) + BY salemslot.mr.itd.umich.edu ID 477DF74E.49493.30415 ; + 4 Jan 2008 04:07:29 -0500 +Received: from paploo.uhi.ac.uk (localhost [127.0.0.1]) + by paploo.uhi.ac.uk (Postfix) with ESMTP id 88598BA5B6; + Fri, 4 Jan 2008 09:07:19 +0000 (GMT) +Message-ID: <200801040905.m0495rWB006420@nakamura.uits.iupui.edu> +Mime-Version: 1.0 +Content-Transfer-Encoding: 7bit +Received: from prod.collab.uhi.ac.uk ([194.35.219.182]) + by paploo.uhi.ac.uk (JAMES SMTP Server 2.1.3) with SMTP ID 385 + for ; + Fri, 4 Jan 2008 09:07:04 +0000 (GMT) +Received: from nakamura.uits.iupui.edu (nakamura.uits.iupui.edu [134.68.220.122]) + by shmi.uhi.ac.uk (Postfix) with ESMTP id 90636418A8 + for ; Fri, 4 Jan 2008 09:07:04 +0000 (GMT) +Received: from nakamura.uits.iupui.edu (localhost [127.0.0.1]) + by nakamura.uits.iupui.edu (8.12.11.20060308/8.12.11) with ESMTP id m0495sZs006422 + for ; Fri, 4 Jan 2008 04:05:54 -0500 +Received: (from apache@localhost) + by nakamura.uits.iupui.edu (8.12.11.20060308/8.12.11/Submit) id m0495rWB006420 + for source@collab.sakaiproject.org; Fri, 4 Jan 2008 04:05:53 -0500 +Date: Fri, 4 Jan 2008 04:05:53 -0500 +X-Authentication-Warning: nakamura.uits.iupui.edu: apache set sender to stephen.marquard@uct.ac.za using -f +To: source@collab.sakaiproject.org +From: stephen.marquard@uct.ac.za +Subject: [sakai] svn commit: r39750 - event/branches/SAK-6216/event-util/util/src/java/org/sakaiproject/util +X-Content-Type-Outer-Envelope: text/plain; charset=UTF-8 +X-Content-Type-Message-Body: text/plain; charset=UTF-8 +Content-Type: text/plain; charset=UTF-8 +X-DSPAM-Result: Innocent +X-DSPAM-Processed: Fri Jan 4 04:07:34 2008 +X-DSPAM-Confidence: 0.7554 +X-DSPAM-Probability: 0.0000 + +Details: http://source.sakaiproject.org/viewsvn/?view=rev&rev=39750 + +Author: stephen.marquard@uct.ac.za +Date: 2008-01-04 04:05:43 -0500 (Fri, 04 Jan 2008) +New Revision: 39750 + +Modified: +event/branches/SAK-6216/event-util/util/src/java/org/sakaiproject/util/EmailNotification.java +Log: +SAK-6216 merge event change from SAK-11169 (r39033) to synchronize branch with 2-5-x (for convenience for UCT local build) + +---------------------- +This automatic notification message was sent by Sakai Collab (https://collab.sakaiproject.org/portal) from the Source site. +You can modify how you receive notifications at My Workspace > Preferences. + + + +From louis@media.berkeley.edu Thu Jan 3 19:51:21 2008 +Return-Path: +Received: from murder (mail.umich.edu [141.211.14.91]) + by frankenstein.mail.umich.edu (Cyrus v2.3.8) with LMTPA; + Thu, 03 Jan 2008 19:51:21 -0500 +X-Sieve: CMU Sieve 2.3 +Received: from murder ([unix socket]) + by mail.umich.edu (Cyrus v2.2.12) with LMTPA; + Thu, 03 Jan 2008 19:51:21 -0500 +Received: from eyewitness.mr.itd.umich.edu (eyewitness.mr.itd.umich.edu [141.211.93.142]) + by jacknife.mail.umich.edu () with ESMTP id m040pJHB027171; + Thu, 3 Jan 2008 19:51:19 -0500 +Received: FROM paploo.uhi.ac.uk (app1.prod.collab.uhi.ac.uk [194.35.219.184]) + BY eyewitness.mr.itd.umich.edu ID 477D8300.AC098.32562 ; + 3 Jan 2008 19:51:15 -0500 +Received: from paploo.uhi.ac.uk (localhost [127.0.0.1]) + by paploo.uhi.ac.uk (Postfix) with ESMTP id E6CC4B9F8A; + Fri, 4 Jan 2008 00:36:06 +0000 (GMT) +Message-ID: <200801040023.m040NpCc005473@nakamura.uits.iupui.edu> +Mime-Version: 1.0 +Content-Transfer-Encoding: 7bit +Received: from prod.collab.uhi.ac.uk ([194.35.219.182]) + by paploo.uhi.ac.uk (JAMES SMTP Server 2.1.3) with SMTP ID 754 + for ; + Fri, 4 Jan 2008 00:35:43 +0000 (GMT) +Received: from nakamura.uits.iupui.edu (nakamura.uits.iupui.edu [134.68.220.122]) + by shmi.uhi.ac.uk (Postfix) with ESMTP id 8889842C49 + for ; Fri, 4 Jan 2008 00:25:00 +0000 (GMT) +Received: from nakamura.uits.iupui.edu (localhost [127.0.0.1]) + by nakamura.uits.iupui.edu (8.12.11.20060308/8.12.11) with ESMTP id m040NpgM005475 + for ; Thu, 3 Jan 2008 19:23:51 -0500 +Received: (from apache@localhost) + by nakamura.uits.iupui.edu (8.12.11.20060308/8.12.11/Submit) id m040NpCc005473 + for source@collab.sakaiproject.org; Thu, 3 Jan 2008 19:23:51 -0500 +Date: Thu, 3 Jan 2008 19:23:51 -0500 +X-Authentication-Warning: nakamura.uits.iupui.edu: apache set sender to louis@media.berkeley.edu using -f +To: source@collab.sakaiproject.org +From: louis@media.berkeley.edu +Subject: [sakai] svn commit: r39749 - in bspace/site-manage/sakai_2-4-x/site-manage-tool/tool/src: bundle webapp/vm/sitesetup +X-Content-Type-Outer-Envelope: text/plain; charset=UTF-8 +X-Content-Type-Message-Body: text/plain; charset=UTF-8 +Content-Type: text/plain; charset=UTF-8 +X-DSPAM-Result: Innocent +X-DSPAM-Processed: Thu Jan 3 19:51:20 2008 +X-DSPAM-Confidence: 0.6956 +X-DSPAM-Probability: 0.0000 + +Details: http://source.sakaiproject.org/viewsvn/?view=rev&rev=39749 + +Author: louis@media.berkeley.edu +Date: 2008-01-03 19:23:46 -0500 (Thu, 03 Jan 2008) +New Revision: 39749 + +Modified: +bspace/site-manage/sakai_2-4-x/site-manage-tool/tool/src/bundle/sitesetupgeneric.properties +bspace/site-manage/sakai_2-4-x/site-manage-tool/tool/src/webapp/vm/sitesetup/chef_site-importSites.vm +Log: +BSP-1420 Update text to clarify "Re-Use Materials..." option in WS Setup + +---------------------- +This automatic notification message was sent by Sakai Collab (https://collab.sakaiproject.org/portal) from the Source site. +You can modify how you receive notifications at My Workspace > Preferences. + + + +From louis@media.berkeley.edu Thu Jan 3 17:18:23 2008 +Return-Path: +Received: from murder (mail.umich.edu [141.211.14.91]) + by frankenstein.mail.umich.edu (Cyrus v2.3.8) with LMTPA; + Thu, 03 Jan 2008 17:18:23 -0500 +X-Sieve: CMU Sieve 2.3 +Received: from murder ([unix socket]) + by mail.umich.edu (Cyrus v2.2.12) with LMTPA; + Thu, 03 Jan 2008 17:18:23 -0500 +Received: from salemslot.mr.itd.umich.edu (salemslot.mr.itd.umich.edu [141.211.14.58]) + by jacknife.mail.umich.edu () with ESMTP id m03MIMXY027729; + Thu, 3 Jan 2008 17:18:22 -0500 +Received: FROM paploo.uhi.ac.uk (app1.prod.collab.uhi.ac.uk [194.35.219.184]) + BY salemslot.mr.itd.umich.edu ID 477D5F23.797F6.16348 ; + 3 Jan 2008 17:18:14 -0500 +Received: from paploo.uhi.ac.uk (localhost [127.0.0.1]) + by paploo.uhi.ac.uk (Postfix) with ESMTP id EF439B98CE; + Thu, 3 Jan 2008 22:18:19 +0000 (GMT) +Message-ID: <200801032216.m03MGhDa005292@nakamura.uits.iupui.edu> +Mime-Version: 1.0 +Content-Transfer-Encoding: 7bit +Received: from prod.collab.uhi.ac.uk ([194.35.219.182]) + by paploo.uhi.ac.uk (JAMES SMTP Server 2.1.3) with SMTP ID 236 + for ; + Thu, 3 Jan 2008 22:18:04 +0000 (GMT) +Received: from nakamura.uits.iupui.edu (nakamura.uits.iupui.edu [134.68.220.122]) + by shmi.uhi.ac.uk (Postfix) with ESMTP id 905D53C2FD + for ; Thu, 3 Jan 2008 22:17:52 +0000 (GMT) +Received: from nakamura.uits.iupui.edu (localhost [127.0.0.1]) + by nakamura.uits.iupui.edu (8.12.11.20060308/8.12.11) with ESMTP id m03MGhrs005294 + for ; Thu, 3 Jan 2008 17:16:43 -0500 +Received: (from apache@localhost) + by nakamura.uits.iupui.edu (8.12.11.20060308/8.12.11/Submit) id m03MGhDa005292 + for source@collab.sakaiproject.org; Thu, 3 Jan 2008 17:16:43 -0500 +Date: Thu, 3 Jan 2008 17:16:43 -0500 +X-Authentication-Warning: nakamura.uits.iupui.edu: apache set sender to louis@media.berkeley.edu using -f +To: source@collab.sakaiproject.org +From: louis@media.berkeley.edu +Subject: [sakai] svn commit: r39746 - in bspace/site-manage/sakai_2-4-x/site-manage-tool/tool/src: bundle webapp/vm/sitesetup +X-Content-Type-Outer-Envelope: text/plain; charset=UTF-8 +X-Content-Type-Message-Body: text/plain; charset=UTF-8 +Content-Type: text/plain; charset=UTF-8 +X-DSPAM-Result: Innocent +X-DSPAM-Processed: Thu Jan 3 17:18:23 2008 +X-DSPAM-Confidence: 0.6959 +X-DSPAM-Probability: 0.0000 + +Details: http://source.sakaiproject.org/viewsvn/?view=rev&rev=39746 + +Author: louis@media.berkeley.edu +Date: 2008-01-03 17:16:39 -0500 (Thu, 03 Jan 2008) +New Revision: 39746 + +Modified: +bspace/site-manage/sakai_2-4-x/site-manage-tool/tool/src/bundle/sitesetupgeneric.properties +bspace/site-manage/sakai_2-4-x/site-manage-tool/tool/src/webapp/vm/sitesetup/chef_site-siteInfo-duplicate.vm +Log: +BSP-1421 Add text to clarify "Duplicate Site" option in Site Info + +---------------------- +This automatic notification message was sent by Sakai Collab (https://collab.sakaiproject.org/portal) from the Source site. +You can modify how you receive notifications at My Workspace > Preferences. + + + +From ray@media.berkeley.edu Thu Jan 3 17:07:00 2008 +Return-Path: +Received: from murder (mail.umich.edu [141.211.14.39]) + by frankenstein.mail.umich.edu (Cyrus v2.3.8) with LMTPA; + Thu, 03 Jan 2008 17:07:00 -0500 +X-Sieve: CMU Sieve 2.3 +Received: from murder ([unix socket]) + by mail.umich.edu (Cyrus v2.2.12) with LMTPA; + Thu, 03 Jan 2008 17:07:00 -0500 +Received: from anniehall.mr.itd.umich.edu (anniehall.mr.itd.umich.edu [141.211.93.141]) + by faithful.mail.umich.edu () with ESMTP id m03M6xaq014868; + Thu, 3 Jan 2008 17:06:59 -0500 +Received: FROM paploo.uhi.ac.uk (app1.prod.collab.uhi.ac.uk [194.35.219.184]) + BY anniehall.mr.itd.umich.edu ID 477D5C7A.4FE1F.22211 ; + 3 Jan 2008 17:06:53 -0500 +Received: from paploo.uhi.ac.uk (localhost [127.0.0.1]) + by paploo.uhi.ac.uk (Postfix) with ESMTP id 0BC8D7225E; + Thu, 3 Jan 2008 22:06:57 +0000 (GMT) +Message-ID: <200801032205.m03M5Ea7005273@nakamura.uits.iupui.edu> +Mime-Version: 1.0 +Content-Transfer-Encoding: 7bit +Received: from prod.collab.uhi.ac.uk ([194.35.219.182]) + by paploo.uhi.ac.uk (JAMES SMTP Server 2.1.3) with SMTP ID 554 + for ; + Thu, 3 Jan 2008 22:06:34 +0000 (GMT) +Received: from nakamura.uits.iupui.edu (nakamura.uits.iupui.edu [134.68.220.122]) + by shmi.uhi.ac.uk (Postfix) with ESMTP id 2AB513C2FD + for ; Thu, 3 Jan 2008 22:06:23 +0000 (GMT) +Received: from nakamura.uits.iupui.edu (localhost [127.0.0.1]) + by nakamura.uits.iupui.edu (8.12.11.20060308/8.12.11) with ESMTP id m03M5EQa005275 + for ; Thu, 3 Jan 2008 17:05:14 -0500 +Received: (from apache@localhost) + by nakamura.uits.iupui.edu (8.12.11.20060308/8.12.11/Submit) id m03M5Ea7005273 + for source@collab.sakaiproject.org; Thu, 3 Jan 2008 17:05:14 -0500 +Date: Thu, 3 Jan 2008 17:05:14 -0500 +X-Authentication-Warning: nakamura.uits.iupui.edu: apache set sender to ray@media.berkeley.edu using -f +To: source@collab.sakaiproject.org +From: ray@media.berkeley.edu +Subject: [sakai] svn commit: r39745 - providers/trunk/cm/cm-authz-provider/src/java/org/sakaiproject/coursemanagement/impl/provider +X-Content-Type-Outer-Envelope: text/plain; charset=UTF-8 +X-Content-Type-Message-Body: text/plain; charset=UTF-8 +Content-Type: text/plain; charset=UTF-8 +X-DSPAM-Result: Innocent +X-DSPAM-Processed: Thu Jan 3 17:07:00 2008 +X-DSPAM-Confidence: 0.7556 +X-DSPAM-Probability: 0.0000 + +Details: http://source.sakaiproject.org/viewsvn/?view=rev&rev=39745 + +Author: ray@media.berkeley.edu +Date: 2008-01-03 17:05:11 -0500 (Thu, 03 Jan 2008) +New Revision: 39745 + +Modified: +providers/trunk/cm/cm-authz-provider/src/java/org/sakaiproject/coursemanagement/impl/provider/CourseManagementGroupProvider.java +Log: +SAK-12602 Fix logic when a user has multiple roles in a section + +---------------------- +This automatic notification message was sent by Sakai Collab (https://collab.sakaiproject.org/portal) from the Source site. +You can modify how you receive notifications at My Workspace > Preferences. + + + +From cwen@iupui.edu Thu Jan 3 16:34:40 2008 +Return-Path: +Received: from murder (mail.umich.edu [141.211.14.34]) + by frankenstein.mail.umich.edu (Cyrus v2.3.8) with LMTPA; + Thu, 03 Jan 2008 16:34:40 -0500 +X-Sieve: CMU Sieve 2.3 +Received: from murder ([unix socket]) + by mail.umich.edu (Cyrus v2.2.12) with LMTPA; + Thu, 03 Jan 2008 16:34:40 -0500 +Received: from icestorm.mr.itd.umich.edu (icestorm.mr.itd.umich.edu [141.211.93.149]) + by chaos.mail.umich.edu () with ESMTP id m03LYdY1029538; + Thu, 3 Jan 2008 16:34:39 -0500 +Received: FROM paploo.uhi.ac.uk (app1.prod.collab.uhi.ac.uk [194.35.219.184]) + BY icestorm.mr.itd.umich.edu ID 477D54EA.13F34.26602 ; + 3 Jan 2008 16:34:36 -0500 +Received: from paploo.uhi.ac.uk (localhost [127.0.0.1]) + by paploo.uhi.ac.uk (Postfix) with ESMTP id CC710ADC79; + Thu, 3 Jan 2008 21:34:29 +0000 (GMT) +Message-ID: <200801032133.m03LX3gG005191@nakamura.uits.iupui.edu> +Mime-Version: 1.0 +Content-Transfer-Encoding: 7bit +Received: from prod.collab.uhi.ac.uk ([194.35.219.182]) + by paploo.uhi.ac.uk (JAMES SMTP Server 2.1.3) with SMTP ID 611 + for ; + Thu, 3 Jan 2008 21:34:08 +0000 (GMT) +Received: from nakamura.uits.iupui.edu (nakamura.uits.iupui.edu [134.68.220.122]) + by shmi.uhi.ac.uk (Postfix) with ESMTP id 43C4242B55 + for ; Thu, 3 Jan 2008 21:34:12 +0000 (GMT) +Received: from nakamura.uits.iupui.edu (localhost [127.0.0.1]) + by nakamura.uits.iupui.edu (8.12.11.20060308/8.12.11) with ESMTP id m03LX3Vb005193 + for ; Thu, 3 Jan 2008 16:33:03 -0500 +Received: (from apache@localhost) + by nakamura.uits.iupui.edu (8.12.11.20060308/8.12.11/Submit) id m03LX3gG005191 + for source@collab.sakaiproject.org; Thu, 3 Jan 2008 16:33:03 -0500 +Date: Thu, 3 Jan 2008 16:33:03 -0500 +X-Authentication-Warning: nakamura.uits.iupui.edu: apache set sender to cwen@iupui.edu using -f +To: source@collab.sakaiproject.org +From: cwen@iupui.edu +Subject: [sakai] svn commit: r39744 - oncourse/branches/oncourse_OPC_122007 +X-Content-Type-Outer-Envelope: text/plain; charset=UTF-8 +X-Content-Type-Message-Body: text/plain; charset=UTF-8 +Content-Type: text/plain; charset=UTF-8 +X-DSPAM-Result: Innocent +X-DSPAM-Processed: Thu Jan 3 16:34:40 2008 +X-DSPAM-Confidence: 0.9846 +X-DSPAM-Probability: 0.0000 + +Details: http://source.sakaiproject.org/viewsvn/?view=rev&rev=39744 + +Author: cwen@iupui.edu +Date: 2008-01-03 16:33:02 -0500 (Thu, 03 Jan 2008) +New Revision: 39744 + +Modified: +oncourse/branches/oncourse_OPC_122007/ +oncourse/branches/oncourse_OPC_122007/.externals +Log: +update external for GB. + +---------------------- +This automatic notification message was sent by Sakai Collab (https://collab.sakaiproject.org/portal) from the Source site. +You can modify how you receive notifications at My Workspace > Preferences. + + + +From cwen@iupui.edu Thu Jan 3 16:29:07 2008 +Return-Path: +Received: from murder (mail.umich.edu [141.211.14.46]) + by frankenstein.mail.umich.edu (Cyrus v2.3.8) with LMTPA; + Thu, 03 Jan 2008 16:29:07 -0500 +X-Sieve: CMU Sieve 2.3 +Received: from murder ([unix socket]) + by mail.umich.edu (Cyrus v2.2.12) with LMTPA; + Thu, 03 Jan 2008 16:29:07 -0500 +Received: from galaxyquest.mr.itd.umich.edu (galaxyquest.mr.itd.umich.edu [141.211.93.145]) + by fan.mail.umich.edu () with ESMTP id m03LT6uw027749; + Thu, 3 Jan 2008 16:29:06 -0500 +Received: FROM paploo.uhi.ac.uk (app1.prod.collab.uhi.ac.uk [194.35.219.184]) + BY galaxyquest.mr.itd.umich.edu ID 477D5397.E161D.20326 ; + 3 Jan 2008 16:28:58 -0500 +Received: from paploo.uhi.ac.uk (localhost [127.0.0.1]) + by paploo.uhi.ac.uk (Postfix) with ESMTP id DEC65ADC79; + Thu, 3 Jan 2008 21:28:52 +0000 (GMT) +Message-ID: <200801032127.m03LRUqH005177@nakamura.uits.iupui.edu> +Mime-Version: 1.0 +Content-Transfer-Encoding: 7bit +Received: from prod.collab.uhi.ac.uk ([194.35.219.182]) + by paploo.uhi.ac.uk (JAMES SMTP Server 2.1.3) with SMTP ID 917 + for ; + Thu, 3 Jan 2008 21:28:39 +0000 (GMT) +Received: from nakamura.uits.iupui.edu (nakamura.uits.iupui.edu [134.68.220.122]) + by shmi.uhi.ac.uk (Postfix) with ESMTP id 1FBB042B30 + for ; Thu, 3 Jan 2008 21:28:38 +0000 (GMT) +Received: from nakamura.uits.iupui.edu (localhost [127.0.0.1]) + by nakamura.uits.iupui.edu (8.12.11.20060308/8.12.11) with ESMTP id m03LRUk4005179 + for ; Thu, 3 Jan 2008 16:27:30 -0500 +Received: (from apache@localhost) + by nakamura.uits.iupui.edu (8.12.11.20060308/8.12.11/Submit) id m03LRUqH005177 + for source@collab.sakaiproject.org; Thu, 3 Jan 2008 16:27:30 -0500 +Date: Thu, 3 Jan 2008 16:27:30 -0500 +X-Authentication-Warning: nakamura.uits.iupui.edu: apache set sender to cwen@iupui.edu using -f +To: source@collab.sakaiproject.org +From: cwen@iupui.edu +Subject: [sakai] svn commit: r39743 - gradebook/branches/oncourse_2-4-2/app/ui/src/java/org/sakaiproject/tool/gradebook/ui +X-Content-Type-Outer-Envelope: text/plain; charset=UTF-8 +X-Content-Type-Message-Body: text/plain; charset=UTF-8 +Content-Type: text/plain; charset=UTF-8 +X-DSPAM-Result: Innocent +X-DSPAM-Processed: Thu Jan 3 16:29:07 2008 +X-DSPAM-Confidence: 0.8509 +X-DSPAM-Probability: 0.0000 + +Details: http://source.sakaiproject.org/viewsvn/?view=rev&rev=39743 + +Author: cwen@iupui.edu +Date: 2008-01-03 16:27:29 -0500 (Thu, 03 Jan 2008) +New Revision: 39743 + +Modified: +gradebook/branches/oncourse_2-4-2/app/ui/src/java/org/sakaiproject/tool/gradebook/ui/RosterBean.java +Log: +svn merge -c 39403 https://source.sakaiproject.org/svn/gradebook/trunk +U app/ui/src/java/org/sakaiproject/tool/gradebook/ui/RosterBean.java + +svn log -r 39403 https://source.sakaiproject.org/svn/gradebook/trunk +------------------------------------------------------------------------ +r39403 | wagnermr@iupui.edu | 2007-12-17 17:11:08 -0500 (Mon, 17 Dec 2007) | 3 lines + +SAK-12504 +http://jira.sakaiproject.org/jira/browse/SAK-12504 +Viewing "All Grades" page as a TA with grader permissions causes stack trace +------------------------------------------------------------------------ + + +---------------------- +This automatic notification message was sent by Sakai Collab (https://collab.sakaiproject.org/portal) from the Source site. +You can modify how you receive notifications at My Workspace > Preferences. + + + +From cwen@iupui.edu Thu Jan 3 16:23:48 2008 +Return-Path: +Received: from murder (mail.umich.edu [141.211.14.91]) + by frankenstein.mail.umich.edu (Cyrus v2.3.8) with LMTPA; + Thu, 03 Jan 2008 16:23:48 -0500 +X-Sieve: CMU Sieve 2.3 +Received: from murder ([unix socket]) + by mail.umich.edu (Cyrus v2.2.12) with LMTPA; + Thu, 03 Jan 2008 16:23:48 -0500 +Received: from salemslot.mr.itd.umich.edu (salemslot.mr.itd.umich.edu [141.211.14.58]) + by jacknife.mail.umich.edu () with ESMTP id m03LNlf0002115; + Thu, 3 Jan 2008 16:23:47 -0500 +Received: FROM paploo.uhi.ac.uk (app1.prod.collab.uhi.ac.uk [194.35.219.184]) + BY salemslot.mr.itd.umich.edu ID 477D525E.1448.30389 ; + 3 Jan 2008 16:23:44 -0500 +Received: from paploo.uhi.ac.uk (localhost [127.0.0.1]) + by paploo.uhi.ac.uk (Postfix) with ESMTP id 9D005B9D06; + Thu, 3 Jan 2008 21:23:38 +0000 (GMT) +Message-ID: <200801032122.m03LMFo4005148@nakamura.uits.iupui.edu> +Mime-Version: 1.0 +Content-Transfer-Encoding: 7bit +Received: from prod.collab.uhi.ac.uk ([194.35.219.182]) + by paploo.uhi.ac.uk (JAMES SMTP Server 2.1.3) with SMTP ID 6 + for ; + Thu, 3 Jan 2008 21:23:24 +0000 (GMT) +Received: from nakamura.uits.iupui.edu (nakamura.uits.iupui.edu [134.68.220.122]) + by shmi.uhi.ac.uk (Postfix) with ESMTP id 3535542B69 + for ; Thu, 3 Jan 2008 21:23:24 +0000 (GMT) +Received: from nakamura.uits.iupui.edu (localhost [127.0.0.1]) + by nakamura.uits.iupui.edu (8.12.11.20060308/8.12.11) with ESMTP id m03LMFtT005150 + for ; Thu, 3 Jan 2008 16:22:15 -0500 +Received: (from apache@localhost) + by nakamura.uits.iupui.edu (8.12.11.20060308/8.12.11/Submit) id m03LMFo4005148 + for source@collab.sakaiproject.org; Thu, 3 Jan 2008 16:22:15 -0500 +Date: Thu, 3 Jan 2008 16:22:15 -0500 +X-Authentication-Warning: nakamura.uits.iupui.edu: apache set sender to cwen@iupui.edu using -f +To: source@collab.sakaiproject.org +From: cwen@iupui.edu +Subject: [sakai] svn commit: r39742 - gradebook/branches/oncourse_2-4-2/app/ui/src/java/org/sakaiproject/tool/gradebook/ui +X-Content-Type-Outer-Envelope: text/plain; charset=UTF-8 +X-Content-Type-Message-Body: text/plain; charset=UTF-8 +Content-Type: text/plain; charset=UTF-8 +X-DSPAM-Result: Innocent +X-DSPAM-Processed: Thu Jan 3 16:23:48 2008 +X-DSPAM-Confidence: 0.9907 +X-DSPAM-Probability: 0.0000 + +Details: http://source.sakaiproject.org/viewsvn/?view=rev&rev=39742 + +Author: cwen@iupui.edu +Date: 2008-01-03 16:22:14 -0500 (Thu, 03 Jan 2008) +New Revision: 39742 + +Modified: +gradebook/branches/oncourse_2-4-2/app/ui/src/java/org/sakaiproject/tool/gradebook/ui/RosterBean.java +Log: +svn merge -c 35014 https://source.sakaiproject.org/svn/gradebook/trunk +U app/ui/src/java/org/sakaiproject/tool/gradebook/ui/RosterBean.java + +svn log -r 35014 https://source.sakaiproject.org/svn/gradebook/trunk +------------------------------------------------------------------------ +r35014 | wagnermr@iupui.edu | 2007-09-12 16:17:59 -0400 (Wed, 12 Sep 2007) | 3 lines + +SAK-11458 +http://bugs.sakaiproject.org/jira/browse/SAK-11458 +Course grade does not appear on "All Grades" page if no categories in gb +------------------------------------------------------------------------ + + +---------------------- +This automatic notification message was sent by Sakai Collab (https://collab.sakaiproject.org/portal) from the Source site. +You can modify how you receive notifications at My Workspace > Preferences. + diff --git a/Assignment 8.5/readme.md b/Assignment 8.5/readme.md new file mode 100644 index 0000000..b73bf55 --- /dev/null +++ b/Assignment 8.5/readme.md @@ -0,0 +1,39 @@ +8.5 Open the file mbox-short.txt and read it line by line. When you find a line that starts with 'From ' like the following line: + +```From stephen.marquard@uct.ac.za Sat Jan 5 09:14:16 2008``` + +You will parse the From line using split() and print out the second word in the line (i.e. the entire address of the person who sent the message). Then print out a count at the end. + +Hint: make sure not to include the lines that start with 'From:'. Also look at the last line of the sample output to see how to print the count. + +Desired Output: +``` +stephen.marquard@uct.ac.za +louis@media.berkeley.edu +zqian@umich.edu +rjlowe@iupui.edu +zqian@umich.edu +rjlowe@iupui.edu +cwen@iupui.edu +cwen@iupui.edu +gsilver@umich.edu +gsilver@umich.edu +zqian@umich.edu +gsilver@umich.edu +wagnermr@iupui.edu +zqian@umich.edu +antranig@caret.cam.ac.uk +gopal.ramasammycook@gmail.com +david.horwitz@uct.ac.za +david.horwitz@uct.ac.za +david.horwitz@uct.ac.za +david.horwitz@uct.ac.za +stephen.marquard@uct.ac.za +louis@media.berkeley.edu +louis@media.berkeley.edu +ray@media.berkeley.edu +cwen@iupui.edu +cwen@iupui.edu +cwen@iupui.edu +There were 27 lines in the file with From as the first word +``` \ No newline at end of file diff --git a/Assignment 9.4/main.py b/Assignment 9.4/main.py new file mode 100644 index 0000000..bfa9aa4 --- /dev/null +++ b/Assignment 9.4/main.py @@ -0,0 +1,24 @@ +name = input("Enter file:") +if len(name) < 1: + name = "mbox-short.txt" +handle = open(name) + +senders = dict() + +for line in handle: + line = line.strip().split() + + if len(line) > 0: + if line[0] == "From": + email = line[1] + senders[email] = senders.get(email, 0) + 1 + +largesender = None +largecount = None + +for email,count in senders.items(): + if largesender == None or count > largecount: + largesender = email + largecount = count + +print(largesender,largecount) \ No newline at end of file diff --git a/Assignment 9.4/mbox-short.txt b/Assignment 9.4/mbox-short.txt new file mode 100644 index 0000000..716dacb --- /dev/null +++ b/Assignment 9.4/mbox-short.txt @@ -0,0 +1,1909 @@ +From stephen.marquard@uct.ac.za Sat Jan 5 09:14:16 2008 ← Mismatch +Return-Path: +Received: from murder (mail.umich.edu [141.211.14.90]) +by frankenstein.mail.umich.edu (Cyrus v2.3.8) with LMTPA; +Sat, 05 Jan 2008 09:14:16 -0500 +X-Sieve: CMU Sieve 2.3 +Received: from murder ([unix socket]) +by mail.umich.edu (Cyrus v2.2.12) with LMTPA; +Sat, 05 Jan 2008 09:14:16 -0500 +Received: from holes.mr.itd.umich.edu (holes.mr.itd.umich.edu [141.211.14.79]) +by flawless.mail.umich.edu () with ESMTP id m05EEFR1013674; +Sat, 5 Jan 2008 09:14:15 -0500 +Received: FROM paploo.uhi.ac.uk (app1.prod.collab.uhi.ac.uk [194.35.219.184]) +BY holes.mr.itd.umich.edu ID 477F90B0.2DB2F.12494 ; +5 Jan 2008 09:14:10 -0500 +Received: from paploo.uhi.ac.uk (localhost [127.0.0.1]) +by paploo.uhi.ac.uk (Postfix) with ESMTP id 5F919BC2F2; +Sat, 5 Jan 2008 14:10:05 +0000 (GMT) +Message-ID: <200801051412.m05ECIaH010327@nakamura.uits.iupui.edu> +Mime-Version: 1.0 +Content-Transfer-Encoding: 7bit +Received: from prod.collab.uhi.ac.uk ([194.35.219.182]) +by paploo.uhi.ac.uk (JAMES SMTP Server 2.1.3) with SMTP ID 899 +for ; +Sat, 5 Jan 2008 14:09:50 +0000 (GMT) +Received: from nakamura.uits.iupui.edu (nakamura.uits.iupui.edu [134.68.220.122]) +by shmi.uhi.ac.uk (Postfix) with ESMTP id A215243002 +for ; Sat, 5 Jan 2008 14:13:33 +0000 (GMT) +Received: from nakamura.uits.iupui.edu (localhost [127.0.0.1]) +by nakamura.uits.iupui.edu (8.12.11.20060308/8.12.11) with ESMTP id m05ECJVp010329 +for ; Sat, 5 Jan 2008 09:12:19 -0500 +Received: (from apache@localhost) +by nakamura.uits.iupui.edu (8.12.11.20060308/8.12.11/Submit) id m05ECIaH010327 +for source@collab.sakaiproject.org; Sat, 5 Jan 2008 09:12:18 -0500 +Date: Sat, 5 Jan 2008 09:12:18 -0500 +X-Authentication-Warning: nakamura.uits.iupui.edu: apache set sender to stephen.marquard@uct.ac.za using -f +To: source@collab.sakaiproject.org +From: stephen.marquard@uct.ac.za +Subject: [sakai] svn commit: r39772 - content/branches/sakai_2-5-x/content-impl/impl/src/java/org/sakaiproject/content/impl +X-Content-Type-Outer-Envelope: text/plain; charset=UTF-8 +X-Content-Type-Message-Body: text/plain; charset=UTF-8 +Content-Type: text/plain; charset=UTF-8 +X-DSPAM-Result: Innocent +X-DSPAM-Processed: Sat Jan 5 09:14:16 2008 +X-DSPAM-Confidence: 0.8475 +X-DSPAM-Probability: 0.0000 + +Details: http://source.sakaiproject.org/viewsvn/?view=rev&rev=39772 + +Author: stephen.marquard@uct.ac.za +Date: 2008-01-05 09:12:07 -0500 (Sat, 05 Jan 2008) +New Revision: 39772 + +Modified: +content/branches/sakai_2-5-x/content-impl/impl/src/java/org/sakaiproject/content/impl/ContentServiceSqlOracle.java +content/branches/sakai_2-5-x/content-impl/impl/src/java/org/sakaiproject/content/impl/DbContentService.java +Log: +SAK-12501 merge to 2-5-x: r39622, r39624:5, r39632:3 (resolve conflict from differing linebreaks for r39622) + +---------------------- +This automatic notification message was sent by Sakai Collab (https://collab.sakaiproject.org/portal) from the Source site. +You can modify how you receive notifications at My Workspace > Preferences. + + + +From louis@media.berkeley.edu Fri Jan 4 18:10:48 2008 +Return-Path: +Received: from murder (mail.umich.edu [141.211.14.97]) +by frankenstein.mail.umich.edu (Cyrus v2.3.8) with LMTPA; +Fri, 04 Jan 2008 18:10:48 -0500 +X-Sieve: CMU Sieve 2.3 +Received: from murder ([unix socket]) +by mail.umich.edu (Cyrus v2.2.12) with LMTPA; +Fri, 04 Jan 2008 18:10:48 -0500 +Received: from icestorm.mr.itd.umich.edu (icestorm.mr.itd.umich.edu [141.211.93.149]) +by sleepers.mail.umich.edu () with ESMTP id m04NAbGa029441; +Fri, 4 Jan 2008 18:10:37 -0500 +Received: FROM paploo.uhi.ac.uk (app1.prod.collab.uhi.ac.uk [194.35.219.184]) +BY icestorm.mr.itd.umich.edu ID 477EBCE3.161BB.4320 ; +4 Jan 2008 18:10:31 -0500 +Received: from paploo.uhi.ac.uk (localhost [127.0.0.1]) +by paploo.uhi.ac.uk (Postfix) with ESMTP id 07969BB706; +Fri, 4 Jan 2008 23:10:33 +0000 (GMT) +Message-ID: <200801042308.m04N8v6O008125@nakamura.uits.iupui.edu> +Mime-Version: 1.0 +Content-Transfer-Encoding: 7bit +Received: from prod.collab.uhi.ac.uk ([194.35.219.182]) +by paploo.uhi.ac.uk (JAMES SMTP Server 2.1.3) with SMTP ID 710 +for ; +Fri, 4 Jan 2008 23:10:10 +0000 (GMT) +Received: from nakamura.uits.iupui.edu (nakamura.uits.iupui.edu [134.68.220.122]) +by shmi.uhi.ac.uk (Postfix) with ESMTP id 4BA2F42F57 +for ; Fri, 4 Jan 2008 23:10:10 +0000 (GMT) +Received: from nakamura.uits.iupui.edu (localhost [127.0.0.1]) +by nakamura.uits.iupui.edu (8.12.11.20060308/8.12.11) with ESMTP id m04N8vHG008127 +for ; Fri, 4 Jan 2008 18:08:57 -0500 +Received: (from apache@localhost) +by nakamura.uits.iupui.edu (8.12.11.20060308/8.12.11/Submit) id m04N8v6O008125 +for source@collab.sakaiproject.org; Fri, 4 Jan 2008 18:08:57 -0500 +Date: Fri, 4 Jan 2008 18:08:57 -0500 +X-Authentication-Warning: nakamura.uits.iupui.edu: apache set sender to louis@media.berkeley.edu using -f +To: source@collab.sakaiproject.org +From: louis@media.berkeley.edu +Subject: [sakai] svn commit: r39771 - in bspace/site-manage/sakai_2-4-x/site-manage-tool/tool/src: bundle java/org/sakaiproject/site/tool +X-Content-Type-Outer-Envelope: text/plain; charset=UTF-8 +X-Content-Type-Message-Body: text/plain; charset=UTF-8 +Content-Type: text/plain; charset=UTF-8 +X-DSPAM-Result: Innocent +X-DSPAM-Processed: Fri Jan 4 18:10:48 2008 +X-DSPAM-Confidence: 0.6178 +X-DSPAM-Probability: 0.0000 + +Details: http://source.sakaiproject.org/viewsvn/?view=rev&rev=39771 + +Author: louis@media.berkeley.edu +Date: 2008-01-04 18:08:50 -0500 (Fri, 04 Jan 2008) +New Revision: 39771 + +Modified: +bspace/site-manage/sakai_2-4-x/site-manage-tool/tool/src/bundle/sitesetupgeneric.properties +bspace/site-manage/sakai_2-4-x/site-manage-tool/tool/src/java/org/sakaiproject/site/tool/SiteAction.java +Log: +BSP-1415 New (Guest) user Notification + +---------------------- +This automatic notification message was sent by Sakai Collab (https://collab.sakaiproject.org/portal) from the Source site. +You can modify how you receive notifications at My Workspace > Preferences. + + + +From zqian@umich.edu Fri Jan 4 16:10:39 2008 +Return-Path: +Received: from murder (mail.umich.edu [141.211.14.25]) +by frankenstein.mail.umich.edu (Cyrus v2.3.8) with LMTPA; +Fri, 04 Jan 2008 16:10:39 -0500 +X-Sieve: CMU Sieve 2.3 +Received: from murder ([unix socket]) +by mail.umich.edu (Cyrus v2.2.12) with LMTPA; +Fri, 04 Jan 2008 16:10:39 -0500 +Received: from ghostbusters.mr.itd.umich.edu (ghostbusters.mr.itd.umich.edu [141.211.93.144]) +by panther.mail.umich.edu () with ESMTP id m04LAcZw014275; +Fri, 4 Jan 2008 16:10:38 -0500 +Received: FROM paploo.uhi.ac.uk (app1.prod.collab.uhi.ac.uk [194.35.219.184]) +BY ghostbusters.mr.itd.umich.edu ID 477EA0C6.A0214.25480 ; +4 Jan 2008 16:10:33 -0500 +Received: from paploo.uhi.ac.uk (localhost [127.0.0.1]) +by paploo.uhi.ac.uk (Postfix) with ESMTP id C48CDBB490; +Fri, 4 Jan 2008 21:10:31 +0000 (GMT) +Message-ID: <200801042109.m04L92hb007923@nakamura.uits.iupui.edu> +Mime-Version: 1.0 +Content-Transfer-Encoding: 7bit +Received: from prod.collab.uhi.ac.uk ([194.35.219.182]) +by paploo.uhi.ac.uk (JAMES SMTP Server 2.1.3) with SMTP ID 906 +for ; +Fri, 4 Jan 2008 21:10:18 +0000 (GMT) +Received: from nakamura.uits.iupui.edu (nakamura.uits.iupui.edu [134.68.220.122]) +by shmi.uhi.ac.uk (Postfix) with ESMTP id 7D13042F71 +for ; Fri, 4 Jan 2008 21:10:14 +0000 (GMT) +Received: from nakamura.uits.iupui.edu (localhost [127.0.0.1]) +by nakamura.uits.iupui.edu (8.12.11.20060308/8.12.11) with ESMTP id m04L927E007925 +for ; Fri, 4 Jan 2008 16:09:02 -0500 +Received: (from apache@localhost) +by nakamura.uits.iupui.edu (8.12.11.20060308/8.12.11/Submit) id m04L92hb007923 +for source@collab.sakaiproject.org; Fri, 4 Jan 2008 16:09:02 -0500 +Date: Fri, 4 Jan 2008 16:09:02 -0500 +X-Authentication-Warning: nakamura.uits.iupui.edu: apache set sender to zqian@umich.edu using -f +To: source@collab.sakaiproject.org +From: zqian@umich.edu +Subject: [sakai] svn commit: r39770 - site-manage/branches/sakai_2-5-x/site-manage-tool/tool/src/webapp/vm/sitesetup +X-Content-Type-Outer-Envelope: text/plain; charset=UTF-8 +X-Content-Type-Message-Body: text/plain; charset=UTF-8 +Content-Type: text/plain; charset=UTF-8 +X-DSPAM-Result: Innocent +X-DSPAM-Processed: Fri Jan 4 16:10:39 2008 +X-DSPAM-Confidence: 0.6961 +X-DSPAM-Probability: 0.0000 + +Details: http://source.sakaiproject.org/viewsvn/?view=rev&rev=39770 + +Author: zqian@umich.edu +Date: 2008-01-04 16:09:01 -0500 (Fri, 04 Jan 2008) +New Revision: 39770 + +Modified: +site-manage/branches/sakai_2-5-x/site-manage-tool/tool/src/webapp/vm/sitesetup/chef_site-siteInfo-list.vm +Log: +merge fix to SAK-9996 into 2-5-x branch: svn merge -r 39687:39688 https://source.sakaiproject.org/svn/site-manage/trunk/ + +---------------------- +This automatic notification message was sent by Sakai Collab (https://collab.sakaiproject.org/portal) from the Source site. +You can modify how you receive notifications at My Workspace > Preferences. + + + +From rjlowe@iupui.edu Fri Jan 4 15:46:24 2008 +Return-Path: +Received: from murder (mail.umich.edu [141.211.14.25]) +by frankenstein.mail.umich.edu (Cyrus v2.3.8) with LMTPA; +Fri, 04 Jan 2008 15:46:24 -0500 +X-Sieve: CMU Sieve 2.3 +Received: from murder ([unix socket]) +by mail.umich.edu (Cyrus v2.2.12) with LMTPA; +Fri, 04 Jan 2008 15:46:24 -0500 +Received: from dreamcatcher.mr.itd.umich.edu (dreamcatcher.mr.itd.umich.edu [141.211.14.43]) +by panther.mail.umich.edu () with ESMTP id m04KkNbx032077; +Fri, 4 Jan 2008 15:46:23 -0500 +Received: FROM paploo.uhi.ac.uk (app1.prod.collab.uhi.ac.uk [194.35.219.184]) +BY dreamcatcher.mr.itd.umich.edu ID 477E9B13.2F3BC.22965 ; +4 Jan 2008 15:46:13 -0500 +Received: from paploo.uhi.ac.uk (localhost [127.0.0.1]) +by paploo.uhi.ac.uk (Postfix) with ESMTP id 4AE03BB552; +Fri, 4 Jan 2008 20:46:13 +0000 (GMT) +Message-ID: <200801042044.m04Kiem3007881@nakamura.uits.iupui.edu> +Mime-Version: 1.0 +Content-Transfer-Encoding: 7bit +Received: from prod.collab.uhi.ac.uk ([194.35.219.182]) +by paploo.uhi.ac.uk (JAMES SMTP Server 2.1.3) with SMTP ID 38 +for ; +Fri, 4 Jan 2008 20:45:56 +0000 (GMT) +Received: from nakamura.uits.iupui.edu (nakamura.uits.iupui.edu [134.68.220.122]) +by shmi.uhi.ac.uk (Postfix) with ESMTP id A55D242F57 +for ; Fri, 4 Jan 2008 20:45:52 +0000 (GMT) +Received: from nakamura.uits.iupui.edu (localhost [127.0.0.1]) +by nakamura.uits.iupui.edu (8.12.11.20060308/8.12.11) with ESMTP id m04KieqE007883 +for ; Fri, 4 Jan 2008 15:44:40 -0500 +Received: (from apache@localhost) +by nakamura.uits.iupui.edu (8.12.11.20060308/8.12.11/Submit) id m04Kiem3007881 +for source@collab.sakaiproject.org; Fri, 4 Jan 2008 15:44:40 -0500 +Date: Fri, 4 Jan 2008 15:44:40 -0500 +X-Authentication-Warning: nakamura.uits.iupui.edu: apache set sender to rjlowe@iupui.edu using -f +To: source@collab.sakaiproject.org +From: rjlowe@iupui.edu +Subject: [sakai] svn commit: r39769 - in gradebook/trunk/app/ui/src: java/org/sakaiproject/tool/gradebook/ui/helpers/beans java/org/sakaiproject/tool/gradebook/ui/helpers/producers webapp/WEB-INF webapp/WEB-INF/bundle +X-Content-Type-Outer-Envelope: text/plain; charset=UTF-8 +X-Content-Type-Message-Body: text/plain; charset=UTF-8 +Content-Type: text/plain; charset=UTF-8 +X-DSPAM-Result: Innocent +X-DSPAM-Processed: Fri Jan 4 15:46:24 2008 +X-DSPAM-Confidence: 0.7565 +X-DSPAM-Probability: 0.0000 + +Details: http://source.sakaiproject.org/viewsvn/?view=rev&rev=39769 + +Author: rjlowe@iupui.edu +Date: 2008-01-04 15:44:39 -0500 (Fri, 04 Jan 2008) +New Revision: 39769 + +Modified: +gradebook/trunk/app/ui/src/java/org/sakaiproject/tool/gradebook/ui/helpers/beans/AssignmentGradeRecordBean.java +gradebook/trunk/app/ui/src/java/org/sakaiproject/tool/gradebook/ui/helpers/producers/GradeGradebookItemProducer.java +gradebook/trunk/app/ui/src/webapp/WEB-INF/applicationContext.xml +gradebook/trunk/app/ui/src/webapp/WEB-INF/bundle/messages.properties +gradebook/trunk/app/ui/src/webapp/WEB-INF/requestContext.xml +Log: +SAK-12180 - Fixed errors with grading helper + +---------------------- +This automatic notification message was sent by Sakai Collab (https://collab.sakaiproject.org/portal) from the Source site. +You can modify how you receive notifications at My Workspace > Preferences. + + + +From zqian@umich.edu Fri Jan 4 15:03:18 2008 +Return-Path: +Received: from murder (mail.umich.edu [141.211.14.46]) +by frankenstein.mail.umich.edu (Cyrus v2.3.8) with LMTPA; +Fri, 04 Jan 2008 15:03:18 -0500 +X-Sieve: CMU Sieve 2.3 +Received: from murder ([unix socket]) +by mail.umich.edu (Cyrus v2.2.12) with LMTPA; +Fri, 04 Jan 2008 15:03:18 -0500 +Received: from firestarter.mr.itd.umich.edu (firestarter.mr.itd.umich.edu [141.211.14.83]) +by fan.mail.umich.edu () with ESMTP id m04K3HGF006563; +Fri, 4 Jan 2008 15:03:17 -0500 +Received: FROM paploo.uhi.ac.uk (app1.prod.collab.uhi.ac.uk [194.35.219.184]) +BY firestarter.mr.itd.umich.edu ID 477E9100.8F7F4.1590 ; +4 Jan 2008 15:03:15 -0500 +Received: from paploo.uhi.ac.uk (localhost [127.0.0.1]) +by paploo.uhi.ac.uk (Postfix) with ESMTP id 57770BB477; +Fri, 4 Jan 2008 20:03:09 +0000 (GMT) +Message-ID: <200801042001.m04K1cO0007738@nakamura.uits.iupui.edu> +Mime-Version: 1.0 +Content-Transfer-Encoding: 7bit +Received: from prod.collab.uhi.ac.uk ([194.35.219.182]) +by paploo.uhi.ac.uk (JAMES SMTP Server 2.1.3) with SMTP ID 622 +for ; +Fri, 4 Jan 2008 20:02:46 +0000 (GMT) +Received: from nakamura.uits.iupui.edu (nakamura.uits.iupui.edu [134.68.220.122]) +by shmi.uhi.ac.uk (Postfix) with ESMTP id AB4D042F4D +for ; Fri, 4 Jan 2008 20:02:50 +0000 (GMT) +Received: from nakamura.uits.iupui.edu (localhost [127.0.0.1]) +by nakamura.uits.iupui.edu (8.12.11.20060308/8.12.11) with ESMTP id m04K1cXv007740 +for ; Fri, 4 Jan 2008 15:01:38 -0500 +Received: (from apache@localhost) +by nakamura.uits.iupui.edu (8.12.11.20060308/8.12.11/Submit) id m04K1cO0007738 +for source@collab.sakaiproject.org; Fri, 4 Jan 2008 15:01:38 -0500 +Date: Fri, 4 Jan 2008 15:01:38 -0500 +X-Authentication-Warning: nakamura.uits.iupui.edu: apache set sender to zqian@umich.edu using -f +To: source@collab.sakaiproject.org +From: zqian@umich.edu +Subject: [sakai] svn commit: r39766 - site-manage/branches/sakai_2-4-x/site-manage-tool/tool/src/java/org/sakaiproject/site/tool +X-Content-Type-Outer-Envelope: text/plain; charset=UTF-8 +X-Content-Type-Message-Body: text/plain; charset=UTF-8 +Content-Type: text/plain; charset=UTF-8 +X-DSPAM-Result: Innocent +X-DSPAM-Processed: Fri Jan 4 15:03:18 2008 +X-DSPAM-Confidence: 0.7626 +X-DSPAM-Probability: 0.0000 + +Details: http://source.sakaiproject.org/viewsvn/?view=rev&rev=39766 + +Author: zqian@umich.edu +Date: 2008-01-04 15:01:37 -0500 (Fri, 04 Jan 2008) +New Revision: 39766 + +Modified: +site-manage/branches/sakai_2-4-x/site-manage-tool/tool/src/java/org/sakaiproject/site/tool/SiteAction.java +Log: +merge fix to SAK-10788 into site-manage 2.4.x branch: + +Sakai Source Repository #38024 Wed Nov 07 14:54:46 MST 2007 zqian@umich.edu Fix to SAK-10788: If a provided id in a couse site is fake or doesn't provide any user information, Site Info appears to be like project site with empty participant list + +Watch for enrollments object being null and concatenate provider ids when there are more than one. +Files Changed +MODIFY /site-manage/trunk/site-manage-tool/tool/src/java/org/sakaiproject/site/tool/SiteAction.java + + + + +---------------------- +This automatic notification message was sent by Sakai Collab (https://collab.sakaiproject.org/portal) from the Source site. +You can modify how you receive notifications at My Workspace > Preferences. + + + +From rjlowe@iupui.edu Fri Jan 4 14:50:18 2008 +Return-Path: +Received: from murder (mail.umich.edu [141.211.14.93]) +by frankenstein.mail.umich.edu (Cyrus v2.3.8) with LMTPA; +Fri, 04 Jan 2008 14:50:18 -0500 +X-Sieve: CMU Sieve 2.3 +Received: from murder ([unix socket]) +by mail.umich.edu (Cyrus v2.2.12) with LMTPA; +Fri, 04 Jan 2008 14:50:18 -0500 +Received: from eyewitness.mr.itd.umich.edu (eyewitness.mr.itd.umich.edu [141.211.93.142]) +by mission.mail.umich.edu () with ESMTP id m04JoHJi019755; +Fri, 4 Jan 2008 14:50:17 -0500 +Received: FROM paploo.uhi.ac.uk (app1.prod.collab.uhi.ac.uk [194.35.219.184]) +BY eyewitness.mr.itd.umich.edu ID 477E8DF2.67B91.5278 ; +4 Jan 2008 14:50:13 -0500 +Received: from paploo.uhi.ac.uk (localhost [127.0.0.1]) +by paploo.uhi.ac.uk (Postfix) with ESMTP id 2D1B9BB492; +Fri, 4 Jan 2008 19:47:10 +0000 (GMT) +Message-ID: <200801041948.m04JmdwO007705@nakamura.uits.iupui.edu> +Mime-Version: 1.0 +Content-Transfer-Encoding: 7bit +Received: from prod.collab.uhi.ac.uk ([194.35.219.182]) +by paploo.uhi.ac.uk (JAMES SMTP Server 2.1.3) with SMTP ID 960 +for ; +Fri, 4 Jan 2008 19:46:50 +0000 (GMT) +Received: from nakamura.uits.iupui.edu (nakamura.uits.iupui.edu [134.68.220.122]) +by shmi.uhi.ac.uk (Postfix) with ESMTP id B3E6742F4A +for ; Fri, 4 Jan 2008 19:49:51 +0000 (GMT) +Received: from nakamura.uits.iupui.edu (localhost [127.0.0.1]) +by nakamura.uits.iupui.edu (8.12.11.20060308/8.12.11) with ESMTP id m04JmeV9007707 +for ; Fri, 4 Jan 2008 14:48:40 -0500 +Received: (from apache@localhost) +by nakamura.uits.iupui.edu (8.12.11.20060308/8.12.11/Submit) id m04JmdwO007705 +for source@collab.sakaiproject.org; Fri, 4 Jan 2008 14:48:39 -0500 +Date: Fri, 4 Jan 2008 14:48:39 -0500 +X-Authentication-Warning: nakamura.uits.iupui.edu: apache set sender to rjlowe@iupui.edu using -f +To: source@collab.sakaiproject.org +From: rjlowe@iupui.edu +Subject: [sakai] svn commit: r39765 - in gradebook/trunk/app: business/src/java/org/sakaiproject/tool/gradebook/business business/src/java/org/sakaiproject/tool/gradebook/business/impl ui ui/src/java/org/sakaiproject/tool/gradebook/ui/helpers/beans ui/src/java/org/sakaiproject/tool/gradebook/ui/helpers/entity ui/src/java/org/sakaiproject/tool/gradebook/ui/helpers/params ui/src/java/org/sakaiproject/tool/gradebook/ui/helpers/producers ui/src/webapp/WEB-INF ui/src/webapp/WEB-INF/bundle ui/src/webapp/content/templates +X-Content-Type-Outer-Envelope: text/plain; charset=UTF-8 +X-Content-Type-Message-Body: text/plain; charset=UTF-8 +Content-Type: text/plain; charset=UTF-8 +X-DSPAM-Result: Innocent +X-DSPAM-Processed: Fri Jan 4 14:50:18 2008 +X-DSPAM-Confidence: 0.7556 +X-DSPAM-Probability: 0.0000 + +Details: http://source.sakaiproject.org/viewsvn/?view=rev&rev=39765 + +Author: rjlowe@iupui.edu +Date: 2008-01-04 14:48:37 -0500 (Fri, 04 Jan 2008) +New Revision: 39765 + +Added: +gradebook/trunk/app/ui/src/java/org/sakaiproject/tool/gradebook/ui/helpers/beans/AssignmentGradeRecordBean.java +gradebook/trunk/app/ui/src/java/org/sakaiproject/tool/gradebook/ui/helpers/beans/AssignmentGradeRecordCreator.java +gradebook/trunk/app/ui/src/java/org/sakaiproject/tool/gradebook/ui/helpers/entity/GradebookEntryGradeEntityProvider.java +gradebook/trunk/app/ui/src/java/org/sakaiproject/tool/gradebook/ui/helpers/params/GradeGradebookItemViewParams.java +gradebook/trunk/app/ui/src/java/org/sakaiproject/tool/gradebook/ui/helpers/producers/GradeGradebookItemProducer.java +gradebook/trunk/app/ui/src/webapp/content/templates/grade-gradebook-item.html +Modified: +gradebook/trunk/app/business/src/java/org/sakaiproject/tool/gradebook/business/GradebookManager.java +gradebook/trunk/app/business/src/java/org/sakaiproject/tool/gradebook/business/impl/GradebookManagerHibernateImpl.java +gradebook/trunk/app/ui/pom.xml +gradebook/trunk/app/ui/src/java/org/sakaiproject/tool/gradebook/ui/helpers/beans/GradebookItemBean.java +gradebook/trunk/app/ui/src/java/org/sakaiproject/tool/gradebook/ui/helpers/entity/GradebookEntryEntityProvider.java +gradebook/trunk/app/ui/src/java/org/sakaiproject/tool/gradebook/ui/helpers/producers/AddGradebookItemProducer.java +gradebook/trunk/app/ui/src/webapp/WEB-INF/applicationContext.xml +gradebook/trunk/app/ui/src/webapp/WEB-INF/bundle/messages.properties +gradebook/trunk/app/ui/src/webapp/WEB-INF/requestContext.xml +Log: +SAK-12180 - New helper tool to grade an assignment + +---------------------- +This automatic notification message was sent by Sakai Collab (https://collab.sakaiproject.org/portal) from the Source site. +You can modify how you receive notifications at My Workspace > Preferences. + + + +From cwen@iupui.edu Fri Jan 4 11:37:30 2008 +Return-Path: +Received: from murder (mail.umich.edu [141.211.14.46]) +by frankenstein.mail.umich.edu (Cyrus v2.3.8) with LMTPA; +Fri, 04 Jan 2008 11:37:30 -0500 +X-Sieve: CMU Sieve 2.3 +Received: from murder ([unix socket]) +by mail.umich.edu (Cyrus v2.2.12) with LMTPA; +Fri, 04 Jan 2008 11:37:30 -0500 +Received: from tadpole.mr.itd.umich.edu (tadpole.mr.itd.umich.edu [141.211.14.72]) +by fan.mail.umich.edu () with ESMTP id m04GbT9x022078; +Fri, 4 Jan 2008 11:37:29 -0500 +Received: FROM paploo.uhi.ac.uk (app1.prod.collab.uhi.ac.uk [194.35.219.184]) +BY tadpole.mr.itd.umich.edu ID 477E60B2.82756.9904 ; +4 Jan 2008 11:37:09 -0500 +Received: from paploo.uhi.ac.uk (localhost [127.0.0.1]) +by paploo.uhi.ac.uk (Postfix) with ESMTP id 8D13DBB001; +Fri, 4 Jan 2008 16:37:07 +0000 (GMT) +Message-ID: <200801041635.m04GZQGZ007313@nakamura.uits.iupui.edu> +Mime-Version: 1.0 +Content-Transfer-Encoding: 7bit +Received: from prod.collab.uhi.ac.uk ([194.35.219.182]) +by paploo.uhi.ac.uk (JAMES SMTP Server 2.1.3) with SMTP ID 120 +for ; +Fri, 4 Jan 2008 16:36:40 +0000 (GMT) +Received: from nakamura.uits.iupui.edu (nakamura.uits.iupui.edu [134.68.220.122]) +by shmi.uhi.ac.uk (Postfix) with ESMTP id D430B42E42 +for ; Fri, 4 Jan 2008 16:36:37 +0000 (GMT) +Received: from nakamura.uits.iupui.edu (localhost [127.0.0.1]) +by nakamura.uits.iupui.edu (8.12.11.20060308/8.12.11) with ESMTP id m04GZQ7W007315 +for ; Fri, 4 Jan 2008 11:35:26 -0500 +Received: (from apache@localhost) +by nakamura.uits.iupui.edu (8.12.11.20060308/8.12.11/Submit) id m04GZQGZ007313 +for source@collab.sakaiproject.org; Fri, 4 Jan 2008 11:35:26 -0500 +Date: Fri, 4 Jan 2008 11:35:26 -0500 +X-Authentication-Warning: nakamura.uits.iupui.edu: apache set sender to cwen@iupui.edu using -f +To: source@collab.sakaiproject.org +From: cwen@iupui.edu +Subject: [sakai] svn commit: r39764 - in msgcntr/trunk/messageforums-app/src/java/org/sakaiproject/tool/messageforums: . ui +X-Content-Type-Outer-Envelope: text/plain; charset=UTF-8 +X-Content-Type-Message-Body: text/plain; charset=UTF-8 +Content-Type: text/plain; charset=UTF-8 +X-DSPAM-Result: Innocent +X-DSPAM-Processed: Fri Jan 4 11:37:30 2008 +X-DSPAM-Confidence: 0.7002 +X-DSPAM-Probability: 0.0000 + +Details: http://source.sakaiproject.org/viewsvn/?view=rev&rev=39764 + +Author: cwen@iupui.edu +Date: 2008-01-04 11:35:25 -0500 (Fri, 04 Jan 2008) +New Revision: 39764 + +Modified: +msgcntr/trunk/messageforums-app/src/java/org/sakaiproject/tool/messageforums/PrivateMessagesTool.java +msgcntr/trunk/messageforums-app/src/java/org/sakaiproject/tool/messageforums/ui/PrivateMessageDecoratedBean.java +Log: +unmerge Xingtang's checkin for SAK-12488. + +svn merge -r39558:39557 https://source.sakaiproject.org/svn/msgcntr/trunk +U messageforums-app/src/java/org/sakaiproject/tool/messageforums/PrivateMessagesTool.java +U messageforums-app/src/java/org/sakaiproject/tool/messageforums/ui/PrivateMessageDecoratedBean.java + +svn log -r 39558 +------------------------------------------------------------------------ +r39558 | hu2@iupui.edu | 2007-12-20 15:25:38 -0500 (Thu, 20 Dec 2007) | 3 lines + +SAK-12488 +when send a message to yourself. click reply to all, cc row should be null. +http://jira.sakaiproject.org/jira/browse/SAK-12488 +------------------------------------------------------------------------ + + +---------------------- +This automatic notification message was sent by Sakai Collab (https://collab.sakaiproject.org/portal) from the Source site. +You can modify how you receive notifications at My Workspace > Preferences. + + + +From cwen@iupui.edu Fri Jan 4 11:35:08 2008 +Return-Path: +Received: from murder (mail.umich.edu [141.211.14.46]) +by frankenstein.mail.umich.edu (Cyrus v2.3.8) with LMTPA; +Fri, 04 Jan 2008 11:35:08 -0500 +X-Sieve: CMU Sieve 2.3 +Received: from murder ([unix socket]) +by mail.umich.edu (Cyrus v2.2.12) with LMTPA; +Fri, 04 Jan 2008 11:35:08 -0500 +Received: from it.mr.itd.umich.edu (it.mr.itd.umich.edu [141.211.93.151]) +by fan.mail.umich.edu () with ESMTP id m04GZ6lt020480; +Fri, 4 Jan 2008 11:35:06 -0500 +Received: FROM paploo.uhi.ac.uk (app1.prod.collab.uhi.ac.uk [194.35.219.184]) +BY it.mr.itd.umich.edu ID 477E6033.6469D.21870 ; +4 Jan 2008 11:35:02 -0500 +Received: from paploo.uhi.ac.uk (localhost [127.0.0.1]) +by paploo.uhi.ac.uk (Postfix) with ESMTP id E40FABAE5B; +Fri, 4 Jan 2008 16:34:38 +0000 (GMT) +Message-ID: <200801041633.m04GX6eG007292@nakamura.uits.iupui.edu> +Mime-Version: 1.0 +Content-Transfer-Encoding: 7bit +Received: from prod.collab.uhi.ac.uk ([194.35.219.182]) +by paploo.uhi.ac.uk (JAMES SMTP Server 2.1.3) with SMTP ID 697 +for ; +Fri, 4 Jan 2008 16:34:01 +0000 (GMT) +Received: from nakamura.uits.iupui.edu (nakamura.uits.iupui.edu [134.68.220.122]) +by shmi.uhi.ac.uk (Postfix) with ESMTP id 1CD0C42E42 +for ; Fri, 4 Jan 2008 16:34:17 +0000 (GMT) +Received: from nakamura.uits.iupui.edu (localhost [127.0.0.1]) +by nakamura.uits.iupui.edu (8.12.11.20060308/8.12.11) with ESMTP id m04GX6Y3007294 +for ; Fri, 4 Jan 2008 11:33:06 -0500 +Received: (from apache@localhost) +by nakamura.uits.iupui.edu (8.12.11.20060308/8.12.11/Submit) id m04GX6eG007292 +for source@collab.sakaiproject.org; Fri, 4 Jan 2008 11:33:06 -0500 +Date: Fri, 4 Jan 2008 11:33:06 -0500 +X-Authentication-Warning: nakamura.uits.iupui.edu: apache set sender to cwen@iupui.edu using -f +To: source@collab.sakaiproject.org +From: cwen@iupui.edu +Subject: [sakai] svn commit: r39763 - in msgcntr/trunk: messageforums-api/src/bundle/org/sakaiproject/api/app/messagecenter/bundle messageforums-app/src/java/org/sakaiproject/tool/messageforums +X-Content-Type-Outer-Envelope: text/plain; charset=UTF-8 +X-Content-Type-Message-Body: text/plain; charset=UTF-8 +Content-Type: text/plain; charset=UTF-8 +X-DSPAM-Result: Innocent +X-DSPAM-Processed: Fri Jan 4 11:35:08 2008 +X-DSPAM-Confidence: 0.7615 +X-DSPAM-Probability: 0.0000 + +Details: http://source.sakaiproject.org/viewsvn/?view=rev&rev=39763 + +Author: cwen@iupui.edu +Date: 2008-01-04 11:33:05 -0500 (Fri, 04 Jan 2008) +New Revision: 39763 + +Modified: +msgcntr/trunk/messageforums-api/src/bundle/org/sakaiproject/api/app/messagecenter/bundle/Messages.properties +msgcntr/trunk/messageforums-app/src/java/org/sakaiproject/tool/messageforums/PrivateMessagesTool.java +Log: +unmerge Xingtang's check in for SAK-12484. + +svn merge -r39571:39570 https://source.sakaiproject.org/svn/msgcntr/trunk +U messageforums-api/src/bundle/org/sakaiproject/api/app/messagecenter/bundle/Messages.properties +U messageforums-app/src/java/org/sakaiproject/tool/messageforums/PrivateMessagesTool.java + +svn log -r 39571 +------------------------------------------------------------------------ +r39571 | hu2@iupui.edu | 2007-12-20 21:26:28 -0500 (Thu, 20 Dec 2007) | 3 lines + +SAK-12484 +reply all cc list should not include the current user name. +http://jira.sakaiproject.org/jira/browse/SAK-12484 +------------------------------------------------------------------------ + + +---------------------- +This automatic notification message was sent by Sakai Collab (https://collab.sakaiproject.org/portal) from the Source site. +You can modify how you receive notifications at My Workspace > Preferences. + + + +From gsilver@umich.edu Fri Jan 4 11:12:37 2008 +Return-Path: +Received: from murder (mail.umich.edu [141.211.14.25]) +by frankenstein.mail.umich.edu (Cyrus v2.3.8) with LMTPA; +Fri, 04 Jan 2008 11:12:37 -0500 +X-Sieve: CMU Sieve 2.3 +Received: from murder ([unix socket]) +by mail.umich.edu (Cyrus v2.2.12) with LMTPA; +Fri, 04 Jan 2008 11:12:37 -0500 +Received: from holes.mr.itd.umich.edu (holes.mr.itd.umich.edu [141.211.14.79]) +by panther.mail.umich.edu () with ESMTP id m04GCaHB030887; +Fri, 4 Jan 2008 11:12:36 -0500 +Received: FROM paploo.uhi.ac.uk (app1.prod.collab.uhi.ac.uk [194.35.219.184]) +BY holes.mr.itd.umich.edu ID 477E5AEB.E670B.28397 ; +4 Jan 2008 11:12:30 -0500 +Received: from paploo.uhi.ac.uk (localhost [127.0.0.1]) +by paploo.uhi.ac.uk (Postfix) with ESMTP id 99715BAE7D; +Fri, 4 Jan 2008 16:12:27 +0000 (GMT) +Message-ID: <200801041611.m04GB1Lb007221@nakamura.uits.iupui.edu> +Mime-Version: 1.0 +Content-Transfer-Encoding: 7bit +Received: from prod.collab.uhi.ac.uk ([194.35.219.182]) +by paploo.uhi.ac.uk (JAMES SMTP Server 2.1.3) with SMTP ID 272 +for ; +Fri, 4 Jan 2008 16:12:14 +0000 (GMT) +Received: from nakamura.uits.iupui.edu (nakamura.uits.iupui.edu [134.68.220.122]) +by shmi.uhi.ac.uk (Postfix) with ESMTP id 0A6ED42DFC +for ; Fri, 4 Jan 2008 16:12:12 +0000 (GMT) +Received: from nakamura.uits.iupui.edu (localhost [127.0.0.1]) +by nakamura.uits.iupui.edu (8.12.11.20060308/8.12.11) with ESMTP id m04GB1Wt007223 +for ; Fri, 4 Jan 2008 11:11:01 -0500 +Received: (from apache@localhost) +by nakamura.uits.iupui.edu (8.12.11.20060308/8.12.11/Submit) id m04GB1Lb007221 +for source@collab.sakaiproject.org; Fri, 4 Jan 2008 11:11:01 -0500 +Date: Fri, 4 Jan 2008 11:11:01 -0500 +X-Authentication-Warning: nakamura.uits.iupui.edu: apache set sender to gsilver@umich.edu using -f +To: source@collab.sakaiproject.org +From: gsilver@umich.edu +Subject: [sakai] svn commit: r39762 - web/trunk/web-tool/tool/src/bundle +X-Content-Type-Outer-Envelope: text/plain; charset=UTF-8 +X-Content-Type-Message-Body: text/plain; charset=UTF-8 +Content-Type: text/plain; charset=UTF-8 +X-DSPAM-Result: Innocent +X-DSPAM-Processed: Fri Jan 4 11:12:37 2008 +X-DSPAM-Confidence: 0.7601 +X-DSPAM-Probability: 0.0000 + +Details: http://source.sakaiproject.org/viewsvn/?view=rev&rev=39762 + +Author: gsilver@umich.edu +Date: 2008-01-04 11:11:00 -0500 (Fri, 04 Jan 2008) +New Revision: 39762 + +Modified: +web/trunk/web-tool/tool/src/bundle/iframe.properties +Log: +SAK-12596 +http://bugs.sakaiproject.org/jira/browse/SAK-12596 +- left moot (unused) entries commented for now + +---------------------- +This automatic notification message was sent by Sakai Collab (https://collab.sakaiproject.org/portal) from the Source site. +You can modify how you receive notifications at My Workspace > Preferences. + + + +From gsilver@umich.edu Fri Jan 4 11:11:52 2008 +Return-Path: +Received: from murder (mail.umich.edu [141.211.14.36]) +by frankenstein.mail.umich.edu (Cyrus v2.3.8) with LMTPA; +Fri, 04 Jan 2008 11:11:52 -0500 +X-Sieve: CMU Sieve 2.3 +Received: from murder ([unix socket]) +by mail.umich.edu (Cyrus v2.2.12) with LMTPA; +Fri, 04 Jan 2008 11:11:52 -0500 +Received: from creepshow.mr.itd.umich.edu (creepshow.mr.itd.umich.edu [141.211.14.84]) +by godsend.mail.umich.edu () with ESMTP id m04GBqqv025330; +Fri, 4 Jan 2008 11:11:52 -0500 +Received: FROM paploo.uhi.ac.uk (app1.prod.collab.uhi.ac.uk [194.35.219.184]) +BY creepshow.mr.itd.umich.edu ID 477E5AB3.5CC32.30840 ; +4 Jan 2008 11:11:34 -0500 +Received: from paploo.uhi.ac.uk (localhost [127.0.0.1]) +by paploo.uhi.ac.uk (Postfix) with ESMTP id 62AA4BAE46; +Fri, 4 Jan 2008 16:11:31 +0000 (GMT) +Message-ID: <200801041610.m04GA5KP007209@nakamura.uits.iupui.edu> +Mime-Version: 1.0 +Content-Transfer-Encoding: 7bit +Received: from prod.collab.uhi.ac.uk ([194.35.219.182]) +by paploo.uhi.ac.uk (JAMES SMTP Server 2.1.3) with SMTP ID 1006 +for ; +Fri, 4 Jan 2008 16:11:18 +0000 (GMT) +Received: from nakamura.uits.iupui.edu (nakamura.uits.iupui.edu [134.68.220.122]) +by shmi.uhi.ac.uk (Postfix) with ESMTP id C596A3DFA2 +for ; Fri, 4 Jan 2008 16:11:16 +0000 (GMT) +Received: from nakamura.uits.iupui.edu (localhost [127.0.0.1]) +by nakamura.uits.iupui.edu (8.12.11.20060308/8.12.11) with ESMTP id m04GA5LR007211 +for ; Fri, 4 Jan 2008 11:10:05 -0500 +Received: (from apache@localhost) +by nakamura.uits.iupui.edu (8.12.11.20060308/8.12.11/Submit) id m04GA5KP007209 +for source@collab.sakaiproject.org; Fri, 4 Jan 2008 11:10:05 -0500 +Date: Fri, 4 Jan 2008 11:10:05 -0500 +X-Authentication-Warning: nakamura.uits.iupui.edu: apache set sender to gsilver@umich.edu using -f +To: source@collab.sakaiproject.org +From: gsilver@umich.edu +Subject: [sakai] svn commit: r39761 - site/trunk/site-tool/tool/src/bundle +X-Content-Type-Outer-Envelope: text/plain; charset=UTF-8 +X-Content-Type-Message-Body: text/plain; charset=UTF-8 +Content-Type: text/plain; charset=UTF-8 +X-DSPAM-Result: Innocent +X-DSPAM-Processed: Fri Jan 4 11:11:52 2008 +X-DSPAM-Confidence: 0.7605 +X-DSPAM-Probability: 0.0000 + +Details: http://source.sakaiproject.org/viewsvn/?view=rev&rev=39761 + +Author: gsilver@umich.edu +Date: 2008-01-04 11:10:04 -0500 (Fri, 04 Jan 2008) +New Revision: 39761 + +Modified: +site/trunk/site-tool/tool/src/bundle/admin.properties +Log: +SAK-12595 +http://bugs.sakaiproject.org/jira/browse/SAK-12595 +- left moot (unused) entries commented for now + +---------------------- +This automatic notification message was sent by Sakai Collab (https://collab.sakaiproject.org/portal) from the Source site. +You can modify how you receive notifications at My Workspace > Preferences. + + + +From zqian@umich.edu Fri Jan 4 11:11:03 2008 +Return-Path: +Received: from murder (mail.umich.edu [141.211.14.97]) +by frankenstein.mail.umich.edu (Cyrus v2.3.8) with LMTPA; +Fri, 04 Jan 2008 11:11:03 -0500 +X-Sieve: CMU Sieve 2.3 +Received: from murder ([unix socket]) +by mail.umich.edu (Cyrus v2.2.12) with LMTPA; +Fri, 04 Jan 2008 11:11:03 -0500 +Received: from carrie.mr.itd.umich.edu (carrie.mr.itd.umich.edu [141.211.93.152]) +by sleepers.mail.umich.edu () with ESMTP id m04GB3Vg011502; +Fri, 4 Jan 2008 11:11:03 -0500 +Received: FROM paploo.uhi.ac.uk (app1.prod.collab.uhi.ac.uk [194.35.219.184]) +BY carrie.mr.itd.umich.edu ID 477E5A8D.B378F.24200 ; +4 Jan 2008 11:10:56 -0500 +Received: from paploo.uhi.ac.uk (localhost [127.0.0.1]) +by paploo.uhi.ac.uk (Postfix) with ESMTP id C7251BAD44; +Fri, 4 Jan 2008 16:10:53 +0000 (GMT) +Message-ID: <200801041609.m04G9EuX007197@nakamura.uits.iupui.edu> +Mime-Version: 1.0 +Content-Transfer-Encoding: 7bit +Received: from prod.collab.uhi.ac.uk ([194.35.219.182]) +by paploo.uhi.ac.uk (JAMES SMTP Server 2.1.3) with SMTP ID 483 +for ; +Fri, 4 Jan 2008 16:10:27 +0000 (GMT) +Received: from nakamura.uits.iupui.edu (nakamura.uits.iupui.edu [134.68.220.122]) +by shmi.uhi.ac.uk (Postfix) with ESMTP id 2E7043DFA2 +for ; Fri, 4 Jan 2008 16:10:26 +0000 (GMT) +Received: from nakamura.uits.iupui.edu (localhost [127.0.0.1]) +by nakamura.uits.iupui.edu (8.12.11.20060308/8.12.11) with ESMTP id m04G9Eqg007199 +for ; Fri, 4 Jan 2008 11:09:15 -0500 +Received: (from apache@localhost) +by nakamura.uits.iupui.edu (8.12.11.20060308/8.12.11/Submit) id m04G9EuX007197 +for source@collab.sakaiproject.org; Fri, 4 Jan 2008 11:09:14 -0500 +Date: Fri, 4 Jan 2008 11:09:14 -0500 +X-Authentication-Warning: nakamura.uits.iupui.edu: apache set sender to zqian@umich.edu using -f +To: source@collab.sakaiproject.org +From: zqian@umich.edu +Subject: [sakai] svn commit: r39760 - in site-manage/trunk/site-manage-tool/tool/src: java/org/sakaiproject/site/tool webapp/vm/sitesetup +X-Content-Type-Outer-Envelope: text/plain; charset=UTF-8 +X-Content-Type-Message-Body: text/plain; charset=UTF-8 +Content-Type: text/plain; charset=UTF-8 +X-DSPAM-Result: Innocent +X-DSPAM-Processed: Fri Jan 4 11:11:03 2008 +X-DSPAM-Confidence: 0.6959 +X-DSPAM-Probability: 0.0000 + +Details: http://source.sakaiproject.org/viewsvn/?view=rev&rev=39760 + +Author: zqian@umich.edu +Date: 2008-01-04 11:09:12 -0500 (Fri, 04 Jan 2008) +New Revision: 39760 + +Modified: +site-manage/trunk/site-manage-tool/tool/src/java/org/sakaiproject/site/tool/SiteAction.java +site-manage/trunk/site-manage-tool/tool/src/webapp/vm/sitesetup/chef_site-siteInfo-list.vm +Log: +fix to SAK-10911: Refactor use of site.upd, site.upd.site.mbrship and site.upd.grp.mbrship permissions + +---------------------- +This automatic notification message was sent by Sakai Collab (https://collab.sakaiproject.org/portal) from the Source site. +You can modify how you receive notifications at My Workspace > Preferences. + + + +From gsilver@umich.edu Fri Jan 4 11:10:22 2008 +Return-Path: +Received: from murder (mail.umich.edu [141.211.14.39]) +by frankenstein.mail.umich.edu (Cyrus v2.3.8) with LMTPA; +Fri, 04 Jan 2008 11:10:22 -0500 +X-Sieve: CMU Sieve 2.3 +Received: from murder ([unix socket]) +by mail.umich.edu (Cyrus v2.2.12) with LMTPA; +Fri, 04 Jan 2008 11:10:22 -0500 +Received: from holes.mr.itd.umich.edu (holes.mr.itd.umich.edu [141.211.14.79]) +by faithful.mail.umich.edu () with ESMTP id m04GAL9k010604; +Fri, 4 Jan 2008 11:10:21 -0500 +Received: FROM paploo.uhi.ac.uk (app1.prod.collab.uhi.ac.uk [194.35.219.184]) +BY holes.mr.itd.umich.edu ID 477E5A67.34350.23015 ; +4 Jan 2008 11:10:18 -0500 +Received: from paploo.uhi.ac.uk (localhost [127.0.0.1]) +by paploo.uhi.ac.uk (Postfix) with ESMTP id 98D04BAD43; +Fri, 4 Jan 2008 16:10:11 +0000 (GMT) +Message-ID: <200801041608.m04G8d7w007184@nakamura.uits.iupui.edu> +Mime-Version: 1.0 +Content-Transfer-Encoding: 7bit +Received: from prod.collab.uhi.ac.uk ([194.35.219.182]) +by paploo.uhi.ac.uk (JAMES SMTP Server 2.1.3) with SMTP ID 966 +for ; +Fri, 4 Jan 2008 16:09:51 +0000 (GMT) +Received: from nakamura.uits.iupui.edu (nakamura.uits.iupui.edu [134.68.220.122]) +by shmi.uhi.ac.uk (Postfix) with ESMTP id 9F89542DD0 +for ; Fri, 4 Jan 2008 16:09:50 +0000 (GMT) +Received: from nakamura.uits.iupui.edu (localhost [127.0.0.1]) +by nakamura.uits.iupui.edu (8.12.11.20060308/8.12.11) with ESMTP id m04G8dXN007186 +for ; Fri, 4 Jan 2008 11:08:39 -0500 +Received: (from apache@localhost) +by nakamura.uits.iupui.edu (8.12.11.20060308/8.12.11/Submit) id m04G8d7w007184 +for source@collab.sakaiproject.org; Fri, 4 Jan 2008 11:08:39 -0500 +Date: Fri, 4 Jan 2008 11:08:39 -0500 +X-Authentication-Warning: nakamura.uits.iupui.edu: apache set sender to gsilver@umich.edu using -f +To: source@collab.sakaiproject.org +From: gsilver@umich.edu +Subject: [sakai] svn commit: r39759 - mailarchive/trunk/mailarchive-tool/tool/src/bundle +X-Content-Type-Outer-Envelope: text/plain; charset=UTF-8 +X-Content-Type-Message-Body: text/plain; charset=UTF-8 +Content-Type: text/plain; charset=UTF-8 +X-DSPAM-Result: Innocent +X-DSPAM-Processed: Fri Jan 4 11:10:22 2008 +X-DSPAM-Confidence: 0.7606 +X-DSPAM-Probability: 0.0000 + +Details: http://source.sakaiproject.org/viewsvn/?view=rev&rev=39759 + +Author: gsilver@umich.edu +Date: 2008-01-04 11:08:38 -0500 (Fri, 04 Jan 2008) +New Revision: 39759 + +Modified: +mailarchive/trunk/mailarchive-tool/tool/src/bundle/email.properties +Log: +SAK-12592 +http://bugs.sakaiproject.org/jira/browse/SAK-12592 +- left moot (unused) entries commented for now + +---------------------- +This automatic notification message was sent by Sakai Collab (https://collab.sakaiproject.org/portal) from the Source site. +You can modify how you receive notifications at My Workspace > Preferences. + + + +From wagnermr@iupui.edu Fri Jan 4 10:38:42 2008 +Return-Path: +Received: from murder (mail.umich.edu [141.211.14.90]) +by frankenstein.mail.umich.edu (Cyrus v2.3.8) with LMTPA; +Fri, 04 Jan 2008 10:38:42 -0500 +X-Sieve: CMU Sieve 2.3 +Received: from murder ([unix socket]) +by mail.umich.edu (Cyrus v2.2.12) with LMTPA; +Fri, 04 Jan 2008 10:38:42 -0500 +Received: from shining.mr.itd.umich.edu (shining.mr.itd.umich.edu [141.211.93.153]) +by flawless.mail.umich.edu () with ESMTP id m04Fcfjm012313; +Fri, 4 Jan 2008 10:38:41 -0500 +Received: FROM paploo.uhi.ac.uk (app1.prod.collab.uhi.ac.uk [194.35.219.184]) +BY shining.mr.itd.umich.edu ID 477E52FA.E6C6E.24093 ; +4 Jan 2008 10:38:37 -0500 +Received: from paploo.uhi.ac.uk (localhost [127.0.0.1]) +by paploo.uhi.ac.uk (Postfix) with ESMTP id 6A39594CD2; +Fri, 4 Jan 2008 15:37:36 +0000 (GMT) +Message-ID: <200801041537.m04Fb6Ci007092@nakamura.uits.iupui.edu> +Mime-Version: 1.0 +Content-Transfer-Encoding: 7bit +Received: from prod.collab.uhi.ac.uk ([194.35.219.182]) +by paploo.uhi.ac.uk (JAMES SMTP Server 2.1.3) with SMTP ID 690 +for ; +Fri, 4 Jan 2008 15:37:21 +0000 (GMT) +Received: from nakamura.uits.iupui.edu (nakamura.uits.iupui.edu [134.68.220.122]) +by shmi.uhi.ac.uk (Postfix) with ESMTP id CEFA037ACE +for ; Fri, 4 Jan 2008 15:38:17 +0000 (GMT) +Received: from nakamura.uits.iupui.edu (localhost [127.0.0.1]) +by nakamura.uits.iupui.edu (8.12.11.20060308/8.12.11) with ESMTP id m04Fb6nh007094 +for ; Fri, 4 Jan 2008 10:37:06 -0500 +Received: (from apache@localhost) +by nakamura.uits.iupui.edu (8.12.11.20060308/8.12.11/Submit) id m04Fb6Ci007092 +for source@collab.sakaiproject.org; Fri, 4 Jan 2008 10:37:06 -0500 +Date: Fri, 4 Jan 2008 10:37:06 -0500 +X-Authentication-Warning: nakamura.uits.iupui.edu: apache set sender to wagnermr@iupui.edu using -f +To: source@collab.sakaiproject.org +From: wagnermr@iupui.edu +Subject: [sakai] svn commit: r39758 - in gradebook/trunk: app/business/src/java/org/sakaiproject/tool/gradebook/business/impl service/api/src/java/org/sakaiproject/service/gradebook/shared service/impl/src/java/org/sakaiproject/component/gradebook +X-Content-Type-Outer-Envelope: text/plain; charset=UTF-8 +X-Content-Type-Message-Body: text/plain; charset=UTF-8 +Content-Type: text/plain; charset=UTF-8 +X-DSPAM-Result: Innocent +X-DSPAM-Processed: Fri Jan 4 10:38:42 2008 +X-DSPAM-Confidence: 0.7559 +X-DSPAM-Probability: 0.0000 + +Details: http://source.sakaiproject.org/viewsvn/?view=rev&rev=39758 + +Author: wagnermr@iupui.edu +Date: 2008-01-04 10:37:04 -0500 (Fri, 04 Jan 2008) +New Revision: 39758 + +Modified: +gradebook/trunk/app/business/src/java/org/sakaiproject/tool/gradebook/business/impl/GradebookManagerHibernateImpl.java +gradebook/trunk/service/api/src/java/org/sakaiproject/service/gradebook/shared/GradebookService.java +gradebook/trunk/service/impl/src/java/org/sakaiproject/component/gradebook/GradebookServiceHibernateImpl.java +Log: +SAK-12175 +http://bugs.sakaiproject.org/jira/browse/SAK-12175 +Create methods required for gb integration with the Assignment2 tool +get_gradeDefinitionForStudentForItem + +---------------------- +This automatic notification message was sent by Sakai Collab (https://collab.sakaiproject.org/portal) from the Source site. +You can modify how you receive notifications at My Workspace > Preferences. + + + +From zqian@umich.edu Fri Jan 4 10:17:43 2008 +Return-Path: +Received: from murder (mail.umich.edu [141.211.14.97]) +by frankenstein.mail.umich.edu (Cyrus v2.3.8) with LMTPA; +Fri, 04 Jan 2008 10:17:43 -0500 +X-Sieve: CMU Sieve 2.3 +Received: from murder ([unix socket]) +by mail.umich.edu (Cyrus v2.2.12) with LMTPA; +Fri, 04 Jan 2008 10:17:42 -0500 +Received: from creepshow.mr.itd.umich.edu (creepshow.mr.itd.umich.edu [141.211.14.84]) +by sleepers.mail.umich.edu () with ESMTP id m04FHgfs011536; +Fri, 4 Jan 2008 10:17:42 -0500 +Received: FROM paploo.uhi.ac.uk (app1.prod.collab.uhi.ac.uk [194.35.219.184]) +BY creepshow.mr.itd.umich.edu ID 477E4E0F.CCA4B.926 ; +4 Jan 2008 10:17:38 -0500 +Received: from paploo.uhi.ac.uk (localhost [127.0.0.1]) +by paploo.uhi.ac.uk (Postfix) with ESMTP id BD02DBAC64; +Fri, 4 Jan 2008 15:17:34 +0000 (GMT) +Message-ID: <200801041515.m04FFv42007050@nakamura.uits.iupui.edu> +Mime-Version: 1.0 +Content-Transfer-Encoding: 7bit +Received: from prod.collab.uhi.ac.uk ([194.35.219.182]) +by paploo.uhi.ac.uk (JAMES SMTP Server 2.1.3) with SMTP ID 25 +for ; +Fri, 4 Jan 2008 15:17:11 +0000 (GMT) +Received: from nakamura.uits.iupui.edu (nakamura.uits.iupui.edu [134.68.220.122]) +by shmi.uhi.ac.uk (Postfix) with ESMTP id 5B396236B9 +for ; Fri, 4 Jan 2008 15:17:08 +0000 (GMT) +Received: from nakamura.uits.iupui.edu (localhost [127.0.0.1]) +by nakamura.uits.iupui.edu (8.12.11.20060308/8.12.11) with ESMTP id m04FFv85007052 +for ; Fri, 4 Jan 2008 10:15:57 -0500 +Received: (from apache@localhost) +by nakamura.uits.iupui.edu (8.12.11.20060308/8.12.11/Submit) id m04FFv42007050 +for source@collab.sakaiproject.org; Fri, 4 Jan 2008 10:15:57 -0500 +Date: Fri, 4 Jan 2008 10:15:57 -0500 +X-Authentication-Warning: nakamura.uits.iupui.edu: apache set sender to zqian@umich.edu using -f +To: source@collab.sakaiproject.org +From: zqian@umich.edu +Subject: [sakai] svn commit: r39757 - in assignment/trunk: assignment-impl/impl/src/java/org/sakaiproject/assignment/impl assignment-tool/tool/src/webapp/vm/assignment +X-Content-Type-Outer-Envelope: text/plain; charset=UTF-8 +X-Content-Type-Message-Body: text/plain; charset=UTF-8 +Content-Type: text/plain; charset=UTF-8 +X-DSPAM-Result: Innocent +X-DSPAM-Processed: Fri Jan 4 10:17:42 2008 +X-DSPAM-Confidence: 0.7605 +X-DSPAM-Probability: 0.0000 + +Details: http://source.sakaiproject.org/viewsvn/?view=rev&rev=39757 + +Author: zqian@umich.edu +Date: 2008-01-04 10:15:54 -0500 (Fri, 04 Jan 2008) +New Revision: 39757 + +Modified: +assignment/trunk/assignment-impl/impl/src/java/org/sakaiproject/assignment/impl/BaseAssignmentService.java +assignment/trunk/assignment-tool/tool/src/webapp/vm/assignment/chef_assignments_instructor_list_submissions.vm +Log: +fix to SAK-12604:Don't show groups/sections filter if the site doesn't have any + +---------------------- +This automatic notification message was sent by Sakai Collab (https://collab.sakaiproject.org/portal) from the Source site. +You can modify how you receive notifications at My Workspace > Preferences. + + + +From antranig@caret.cam.ac.uk Fri Jan 4 10:04:14 2008 +Return-Path: +Received: from murder (mail.umich.edu [141.211.14.25]) +by frankenstein.mail.umich.edu (Cyrus v2.3.8) with LMTPA; +Fri, 04 Jan 2008 10:04:14 -0500 +X-Sieve: CMU Sieve 2.3 +Received: from murder ([unix socket]) +by mail.umich.edu (Cyrus v2.2.12) with LMTPA; +Fri, 04 Jan 2008 10:04:14 -0500 +Received: from holes.mr.itd.umich.edu (holes.mr.itd.umich.edu [141.211.14.79]) +by panther.mail.umich.edu () with ESMTP id m04F4Dci015108; +Fri, 4 Jan 2008 10:04:13 -0500 +Received: FROM paploo.uhi.ac.uk (app1.prod.collab.uhi.ac.uk [194.35.219.184]) +BY holes.mr.itd.umich.edu ID 477E4AE3.D7AF.31669 ; +4 Jan 2008 10:04:05 -0500 +Received: from paploo.uhi.ac.uk (localhost [127.0.0.1]) +by paploo.uhi.ac.uk (Postfix) with ESMTP id 933E3BAC17; +Fri, 4 Jan 2008 15:04:00 +0000 (GMT) +Message-ID: <200801041502.m04F21Jo007031@nakamura.uits.iupui.edu> +Mime-Version: 1.0 +Content-Transfer-Encoding: 7bit +Received: from prod.collab.uhi.ac.uk ([194.35.219.182]) +by paploo.uhi.ac.uk (JAMES SMTP Server 2.1.3) with SMTP ID 32 +for ; +Fri, 4 Jan 2008 15:03:15 +0000 (GMT) +Received: from nakamura.uits.iupui.edu (nakamura.uits.iupui.edu [134.68.220.122]) +by shmi.uhi.ac.uk (Postfix) with ESMTP id AC2F6236B9 +for ; Fri, 4 Jan 2008 15:03:12 +0000 (GMT) +Received: from nakamura.uits.iupui.edu (localhost [127.0.0.1]) +by nakamura.uits.iupui.edu (8.12.11.20060308/8.12.11) with ESMTP id m04F21hn007033 +for ; Fri, 4 Jan 2008 10:02:01 -0500 +Received: (from apache@localhost) +by nakamura.uits.iupui.edu (8.12.11.20060308/8.12.11/Submit) id m04F21Jo007031 +for source@collab.sakaiproject.org; Fri, 4 Jan 2008 10:02:01 -0500 +Date: Fri, 4 Jan 2008 10:02:01 -0500 +X-Authentication-Warning: nakamura.uits.iupui.edu: apache set sender to antranig@caret.cam.ac.uk using -f +To: source@collab.sakaiproject.org +From: antranig@caret.cam.ac.uk +Subject: [sakai] svn commit: r39756 - in component/branches/SAK-12166/component-api/component/src/java/org/sakaiproject/component: impl impl/spring/support impl/spring/support/dynamic impl/support util +X-Content-Type-Outer-Envelope: text/plain; charset=UTF-8 +X-Content-Type-Message-Body: text/plain; charset=UTF-8 +Content-Type: text/plain; charset=UTF-8 +X-DSPAM-Result: Innocent +X-DSPAM-Processed: Fri Jan 4 10:04:14 2008 +X-DSPAM-Confidence: 0.6932 +X-DSPAM-Probability: 0.0000 + +Details: http://source.sakaiproject.org/viewsvn/?view=rev&rev=39756 + +Author: antranig@caret.cam.ac.uk +Date: 2008-01-04 10:01:40 -0500 (Fri, 04 Jan 2008) +New Revision: 39756 + +Added: +component/branches/SAK-12166/component-api/component/src/java/org/sakaiproject/component/impl/spring/support/dynamic/ +component/branches/SAK-12166/component-api/component/src/java/org/sakaiproject/component/impl/spring/support/dynamic/DynamicComponentManager.java +component/branches/SAK-12166/component-api/component/src/java/org/sakaiproject/component/impl/support/ +component/branches/SAK-12166/component-api/component/src/java/org/sakaiproject/component/impl/support/DynamicComponentRecord.java +component/branches/SAK-12166/component-api/component/src/java/org/sakaiproject/component/impl/support/DynamicJARManager.java +component/branches/SAK-12166/component-api/component/src/java/org/sakaiproject/component/impl/support/JARRecord.java +component/branches/SAK-12166/component-api/component/src/java/org/sakaiproject/component/util/ByteToCharBase64.java +component/branches/SAK-12166/component-api/component/src/java/org/sakaiproject/component/util/FileUtil.java +component/branches/SAK-12166/component-api/component/src/java/org/sakaiproject/component/util/RecordFileIO.java +component/branches/SAK-12166/component-api/component/src/java/org/sakaiproject/component/util/RecordReader.java +component/branches/SAK-12166/component-api/component/src/java/org/sakaiproject/component/util/RecordWriter.java +component/branches/SAK-12166/component-api/component/src/java/org/sakaiproject/component/util/StreamDigestor.java +Modified: +component/branches/SAK-12166/component-api/component/src/java/org/sakaiproject/component/impl/spring/support/ComponentsLoaderImpl.java +Log: +Temporary commit of incomplete work on JAR caching + +---------------------- +This automatic notification message was sent by Sakai Collab (https://collab.sakaiproject.org/portal) from the Source site. +You can modify how you receive notifications at My Workspace > Preferences. + + + +From gopal.ramasammycook@gmail.com Fri Jan 4 09:05:31 2008 +Return-Path: +Received: from murder (mail.umich.edu [141.211.14.90]) +by frankenstein.mail.umich.edu (Cyrus v2.3.8) with LMTPA; +Fri, 04 Jan 2008 09:05:31 -0500 +X-Sieve: CMU Sieve 2.3 +Received: from murder ([unix socket]) +by mail.umich.edu (Cyrus v2.2.12) with LMTPA; +Fri, 04 Jan 2008 09:05:31 -0500 +Received: from guys.mr.itd.umich.edu (guys.mr.itd.umich.edu [141.211.14.76]) +by flawless.mail.umich.edu () with ESMTP id m04E5U3C029277; +Fri, 4 Jan 2008 09:05:30 -0500 +Received: FROM paploo.uhi.ac.uk (app1.prod.collab.uhi.ac.uk [194.35.219.184]) +BY guys.mr.itd.umich.edu ID 477E3D23.EE2E7.5237 ; +4 Jan 2008 09:05:26 -0500 +Received: from paploo.uhi.ac.uk (localhost [127.0.0.1]) +by paploo.uhi.ac.uk (Postfix) with ESMTP id 33C7856DC0; +Fri, 4 Jan 2008 14:05:26 +0000 (GMT) +Message-ID: <200801041403.m04E3psW006926@nakamura.uits.iupui.edu> +Mime-Version: 1.0 +Content-Transfer-Encoding: 7bit +Received: from prod.collab.uhi.ac.uk ([194.35.219.182]) +by paploo.uhi.ac.uk (JAMES SMTP Server 2.1.3) with SMTP ID 575 +for ; +Fri, 4 Jan 2008 14:05:04 +0000 (GMT) +Received: from nakamura.uits.iupui.edu (nakamura.uits.iupui.edu [134.68.220.122]) +by shmi.uhi.ac.uk (Postfix) with ESMTP id 3C0261D617 +for ; Fri, 4 Jan 2008 14:05:03 +0000 (GMT) +Received: from nakamura.uits.iupui.edu (localhost [127.0.0.1]) +by nakamura.uits.iupui.edu (8.12.11.20060308/8.12.11) with ESMTP id m04E3pQS006928 +for ; Fri, 4 Jan 2008 09:03:52 -0500 +Received: (from apache@localhost) +by nakamura.uits.iupui.edu (8.12.11.20060308/8.12.11/Submit) id m04E3psW006926 +for source@collab.sakaiproject.org; Fri, 4 Jan 2008 09:03:51 -0500 +Date: Fri, 4 Jan 2008 09:03:51 -0500 +X-Authentication-Warning: nakamura.uits.iupui.edu: apache set sender to gopal.ramasammycook@gmail.com using -f +To: source@collab.sakaiproject.org +From: gopal.ramasammycook@gmail.com +Subject: [sakai] svn commit: r39755 - in sam/branches/SAK-12065: samigo-api/src/java/org/sakaiproject/tool/assessment/shared/api/grading samigo-app/src/java/org/sakaiproject/tool/assessment/ui/bean/evaluation samigo-app/src/java/org/sakaiproject/tool/assessment/ui/listener/evaluation samigo-services/src/java/org/sakaiproject/tool/assessment/facade samigo-services/src/java/org/sakaiproject/tool/assessment/integration/helper/ifc samigo-services/src/java/org/sakaiproject/tool/assessment/integration/helper/integrated samigo-services/src/java/org/sakaiproject/tool/assessment/integration/helper/standalone samigo-services/src/java/org/sakaiproject/tool/assessment/shared/impl/grading +X-Content-Type-Outer-Envelope: text/plain; charset=UTF-8 +X-Content-Type-Message-Body: text/plain; charset=UTF-8 +Content-Type: text/plain; charset=UTF-8 +X-DSPAM-Result: Innocent +X-DSPAM-Processed: Fri Jan 4 09:05:31 2008 +X-DSPAM-Confidence: 0.7558 +X-DSPAM-Probability: 0.0000 + +Details: http://source.sakaiproject.org/viewsvn/?view=rev&rev=39755 + +Author: gopal.ramasammycook@gmail.com +Date: 2008-01-04 09:02:54 -0500 (Fri, 04 Jan 2008) +New Revision: 39755 + +Modified: +sam/branches/SAK-12065/samigo-api/src/java/org/sakaiproject/tool/assessment/shared/api/grading/GradingSectionAwareServiceAPI.java +sam/branches/SAK-12065/samigo-app/src/java/org/sakaiproject/tool/assessment/ui/bean/evaluation/QuestionScoresBean.java +sam/branches/SAK-12065/samigo-app/src/java/org/sakaiproject/tool/assessment/ui/bean/evaluation/SubmissionStatusBean.java +sam/branches/SAK-12065/samigo-app/src/java/org/sakaiproject/tool/assessment/ui/bean/evaluation/TotalScoresBean.java +sam/branches/SAK-12065/samigo-app/src/java/org/sakaiproject/tool/assessment/ui/listener/evaluation/SubmissionStatusListener.java +sam/branches/SAK-12065/samigo-services/src/java/org/sakaiproject/tool/assessment/facade/PublishedAssessmentFacadeQueries.java +sam/branches/SAK-12065/samigo-services/src/java/org/sakaiproject/tool/assessment/facade/PublishedAssessmentFacadeQueriesAPI.java +sam/branches/SAK-12065/samigo-services/src/java/org/sakaiproject/tool/assessment/integration/helper/ifc/SectionAwareServiceHelper.java +sam/branches/SAK-12065/samigo-services/src/java/org/sakaiproject/tool/assessment/integration/helper/integrated/SectionAwareServiceHelperImpl.java +sam/branches/SAK-12065/samigo-services/src/java/org/sakaiproject/tool/assessment/integration/helper/standalone/SectionAwareServiceHelperImpl.java +sam/branches/SAK-12065/samigo-services/src/java/org/sakaiproject/tool/assessment/shared/impl/grading/GradingSectionAwareServiceImpl.java +Log: +SAK-12065 Gopal - Samigo Group Release. SubmissionStatus/TotalScores/Questions View filter. + +---------------------- +This automatic notification message was sent by Sakai Collab (https://collab.sakaiproject.org/portal) from the Source site. +You can modify how you receive notifications at My Workspace > Preferences. + + + +From david.horwitz@uct.ac.za Fri Jan 4 07:02:32 2008 +Return-Path: +Received: from murder (mail.umich.edu [141.211.14.39]) +by frankenstein.mail.umich.edu (Cyrus v2.3.8) with LMTPA; +Fri, 04 Jan 2008 07:02:32 -0500 +X-Sieve: CMU Sieve 2.3 +Received: from murder ([unix socket]) +by mail.umich.edu (Cyrus v2.2.12) with LMTPA; +Fri, 04 Jan 2008 07:02:32 -0500 +Received: from guys.mr.itd.umich.edu (guys.mr.itd.umich.edu [141.211.14.76]) +by faithful.mail.umich.edu () with ESMTP id m04C2VN7026678; +Fri, 4 Jan 2008 07:02:31 -0500 +Received: FROM paploo.uhi.ac.uk (app1.prod.collab.uhi.ac.uk [194.35.219.184]) +BY guys.mr.itd.umich.edu ID 477E2050.C2599.3263 ; +4 Jan 2008 07:02:27 -0500 +Received: from paploo.uhi.ac.uk (localhost [127.0.0.1]) +by paploo.uhi.ac.uk (Postfix) with ESMTP id 6497FBA906; +Fri, 4 Jan 2008 12:02:11 +0000 (GMT) +Message-ID: <200801041200.m04C0gfK006793@nakamura.uits.iupui.edu> +Mime-Version: 1.0 +Content-Transfer-Encoding: 7bit +Received: from prod.collab.uhi.ac.uk ([194.35.219.182]) +by paploo.uhi.ac.uk (JAMES SMTP Server 2.1.3) with SMTP ID 611 +for ; +Fri, 4 Jan 2008 12:01:53 +0000 (GMT) +Received: from nakamura.uits.iupui.edu (nakamura.uits.iupui.edu [134.68.220.122]) +by shmi.uhi.ac.uk (Postfix) with ESMTP id 5296342D3C +for ; Fri, 4 Jan 2008 12:01:53 +0000 (GMT) +Received: from nakamura.uits.iupui.edu (localhost [127.0.0.1]) +by nakamura.uits.iupui.edu (8.12.11.20060308/8.12.11) with ESMTP id m04C0gnm006795 +for ; Fri, 4 Jan 2008 07:00:42 -0500 +Received: (from apache@localhost) +by nakamura.uits.iupui.edu (8.12.11.20060308/8.12.11/Submit) id m04C0gfK006793 +for source@collab.sakaiproject.org; Fri, 4 Jan 2008 07:00:42 -0500 +Date: Fri, 4 Jan 2008 07:00:42 -0500 +X-Authentication-Warning: nakamura.uits.iupui.edu: apache set sender to david.horwitz@uct.ac.za using -f +To: source@collab.sakaiproject.org +From: david.horwitz@uct.ac.za +Subject: [sakai] svn commit: r39754 - in polls/branches/sakai_2-5-x: . tool tool/src/java/org/sakaiproject/poll/tool tool/src/java/org/sakaiproject/poll/tool/evolvers tool/src/webapp/WEB-INF +X-Content-Type-Outer-Envelope: text/plain; charset=UTF-8 +X-Content-Type-Message-Body: text/plain; charset=UTF-8 +Content-Type: text/plain; charset=UTF-8 +X-DSPAM-Result: Innocent +X-DSPAM-Processed: Fri Jan 4 07:02:32 2008 +X-DSPAM-Confidence: 0.6526 +X-DSPAM-Probability: 0.0000 + +Details: http://source.sakaiproject.org/viewsvn/?view=rev&rev=39754 + +Author: david.horwitz@uct.ac.za +Date: 2008-01-04 07:00:10 -0500 (Fri, 04 Jan 2008) +New Revision: 39754 + +Added: +polls/branches/sakai_2-5-x/tool/src/java/org/sakaiproject/poll/tool/evolvers/ +polls/branches/sakai_2-5-x/tool/src/java/org/sakaiproject/poll/tool/evolvers/SakaiFCKTextEvolver.java +Removed: +polls/branches/sakai_2-5-x/tool/src/java/org/sakaiproject/poll/tool/evolvers/SakaiFCKTextEvolver.java +Modified: +polls/branches/sakai_2-5-x/.classpath +polls/branches/sakai_2-5-x/tool/pom.xml +polls/branches/sakai_2-5-x/tool/src/webapp/WEB-INF/requestContext.xml +Log: +svn log -r39753 https://source.sakaiproject.org/svn/polls/trunk +------------------------------------------------------------------------ +r39753 | david.horwitz@uct.ac.za | 2008-01-04 13:05:51 +0200 (Fri, 04 Jan 2008) | 1 line + +SAK-12228 implmented workaround sugested by AB - needs to be tested against a trunk build +------------------------------------------------------------------------ +dhorwitz@david-horwitz-6:~/branchManagemnt/sakai_2-5-x> svn merge -c39753 https://source.sakaiproject.org/svn/polls/trunk polls/ +U polls/.classpath +A polls/tool/src/java/org/sakaiproject/poll/tool/evolvers +A polls/tool/src/java/org/sakaiproject/poll/tool/evolvers/SakaiFCKTextEvolver.java +C polls/tool/src/webapp/WEB-INF/requestContext.xml +U polls/tool/pom.xml + +dhorwitz@david-horwitz-6:~/branchManagemnt/sakai_2-5-x> svn resolved polls/tool/src/webapp/WEB-INF/requestContext.xml +Resolved conflicted state of 'polls/tool/src/webapp/WEB-INF/requestContext.xml + + +---------------------- +This automatic notification message was sent by Sakai Collab (https://collab.sakaiproject.org/portal) from the Source site. +You can modify how you receive notifications at My Workspace > Preferences. + + + +From david.horwitz@uct.ac.za Fri Jan 4 06:08:27 2008 +Return-Path: +Received: from murder (mail.umich.edu [141.211.14.98]) +by frankenstein.mail.umich.edu (Cyrus v2.3.8) with LMTPA; +Fri, 04 Jan 2008 06:08:27 -0500 +X-Sieve: CMU Sieve 2.3 +Received: from murder ([unix socket]) +by mail.umich.edu (Cyrus v2.2.12) with LMTPA; +Fri, 04 Jan 2008 06:08:27 -0500 +Received: from firestarter.mr.itd.umich.edu (firestarter.mr.itd.umich.edu [141.211.14.83]) +by casino.mail.umich.edu () with ESMTP id m04B8Qw9001368; +Fri, 4 Jan 2008 06:08:26 -0500 +Received: FROM paploo.uhi.ac.uk (app1.prod.collab.uhi.ac.uk [194.35.219.184]) +BY firestarter.mr.itd.umich.edu ID 477E13A5.30FC0.24054 ; +4 Jan 2008 06:08:23 -0500 +Received: from paploo.uhi.ac.uk (localhost [127.0.0.1]) +by paploo.uhi.ac.uk (Postfix) with ESMTP id 784A476D7B; +Fri, 4 Jan 2008 11:08:12 +0000 (GMT) +Message-ID: <200801041106.m04B6lK3006677@nakamura.uits.iupui.edu> +Mime-Version: 1.0 +Content-Transfer-Encoding: 7bit +Received: from prod.collab.uhi.ac.uk ([194.35.219.182]) +by paploo.uhi.ac.uk (JAMES SMTP Server 2.1.3) with SMTP ID 585 +for ; +Fri, 4 Jan 2008 11:07:56 +0000 (GMT) +Received: from nakamura.uits.iupui.edu (nakamura.uits.iupui.edu [134.68.220.122]) +by shmi.uhi.ac.uk (Postfix) with ESMTP id 1CACC42D0C +for ; Fri, 4 Jan 2008 11:07:58 +0000 (GMT) +Received: from nakamura.uits.iupui.edu (localhost [127.0.0.1]) +by nakamura.uits.iupui.edu (8.12.11.20060308/8.12.11) with ESMTP id m04B6lWM006679 +for ; Fri, 4 Jan 2008 06:06:47 -0500 +Received: (from apache@localhost) +by nakamura.uits.iupui.edu (8.12.11.20060308/8.12.11/Submit) id m04B6lK3006677 +for source@collab.sakaiproject.org; Fri, 4 Jan 2008 06:06:47 -0500 +Date: Fri, 4 Jan 2008 06:06:47 -0500 +X-Authentication-Warning: nakamura.uits.iupui.edu: apache set sender to david.horwitz@uct.ac.za using -f +To: source@collab.sakaiproject.org +From: david.horwitz@uct.ac.za +Subject: [sakai] svn commit: r39753 - in polls/trunk: . tool tool/src/java/org/sakaiproject/poll/tool tool/src/java/org/sakaiproject/poll/tool/evolvers tool/src/webapp/WEB-INF +X-Content-Type-Outer-Envelope: text/plain; charset=UTF-8 +X-Content-Type-Message-Body: text/plain; charset=UTF-8 +Content-Type: text/plain; charset=UTF-8 +X-DSPAM-Result: Innocent +X-DSPAM-Processed: Fri Jan 4 06:08:27 2008 +X-DSPAM-Confidence: 0.6948 +X-DSPAM-Probability: 0.0000 + +Details: http://source.sakaiproject.org/viewsvn/?view=rev&rev=39753 + +Author: david.horwitz@uct.ac.za +Date: 2008-01-04 06:05:51 -0500 (Fri, 04 Jan 2008) +New Revision: 39753 + +Added: +polls/trunk/tool/src/java/org/sakaiproject/poll/tool/evolvers/ +polls/trunk/tool/src/java/org/sakaiproject/poll/tool/evolvers/SakaiFCKTextEvolver.java +Modified: +polls/trunk/.classpath +polls/trunk/tool/pom.xml +polls/trunk/tool/src/webapp/WEB-INF/requestContext.xml +Log: +SAK-12228 implmented workaround sugested by AB - needs to be tested against a trunk build + +---------------------- +This automatic notification message was sent by Sakai Collab (https://collab.sakaiproject.org/portal) from the Source site. +You can modify how you receive notifications at My Workspace > Preferences. + + + +From david.horwitz@uct.ac.za Fri Jan 4 04:49:08 2008 +Return-Path: +Received: from murder (mail.umich.edu [141.211.14.92]) +by frankenstein.mail.umich.edu (Cyrus v2.3.8) with LMTPA; +Fri, 04 Jan 2008 04:49:08 -0500 +X-Sieve: CMU Sieve 2.3 +Received: from murder ([unix socket]) +by mail.umich.edu (Cyrus v2.2.12) with LMTPA; +Fri, 04 Jan 2008 04:49:08 -0500 +Received: from galaxyquest.mr.itd.umich.edu (galaxyquest.mr.itd.umich.edu [141.211.93.145]) +by score.mail.umich.edu () with ESMTP id m049n60G017588; +Fri, 4 Jan 2008 04:49:06 -0500 +Received: FROM paploo.uhi.ac.uk (app1.prod.collab.uhi.ac.uk [194.35.219.184]) +BY galaxyquest.mr.itd.umich.edu ID 477E010C.48C2.10259 ; +4 Jan 2008 04:49:03 -0500 +Received: from paploo.uhi.ac.uk (localhost [127.0.0.1]) +by paploo.uhi.ac.uk (Postfix) with ESMTP id 254CC8CDEE; +Fri, 4 Jan 2008 09:48:55 +0000 (GMT) +Message-ID: <200801040947.m049lUxo006517@nakamura.uits.iupui.edu> +Mime-Version: 1.0 +Content-Transfer-Encoding: 7bit +Received: from prod.collab.uhi.ac.uk ([194.35.219.182]) +by paploo.uhi.ac.uk (JAMES SMTP Server 2.1.3) with SMTP ID 246 +for ; +Fri, 4 Jan 2008 09:48:36 +0000 (GMT) +Received: from nakamura.uits.iupui.edu (nakamura.uits.iupui.edu [134.68.220.122]) +by shmi.uhi.ac.uk (Postfix) with ESMTP id 8C13342C92 +for ; Fri, 4 Jan 2008 09:48:40 +0000 (GMT) +Received: from nakamura.uits.iupui.edu (localhost [127.0.0.1]) +by nakamura.uits.iupui.edu (8.12.11.20060308/8.12.11) with ESMTP id m049lU3P006519 +for ; Fri, 4 Jan 2008 04:47:30 -0500 +Received: (from apache@localhost) +by nakamura.uits.iupui.edu (8.12.11.20060308/8.12.11/Submit) id m049lUxo006517 +for source@collab.sakaiproject.org; Fri, 4 Jan 2008 04:47:30 -0500 +Date: Fri, 4 Jan 2008 04:47:30 -0500 +X-Authentication-Warning: nakamura.uits.iupui.edu: apache set sender to david.horwitz@uct.ac.za using -f +To: source@collab.sakaiproject.org +From: david.horwitz@uct.ac.za +Subject: [sakai] svn commit: r39752 - in podcasts/branches/sakai_2-5-x/podcasts-app/src/webapp: css podcasts +X-Content-Type-Outer-Envelope: text/plain; charset=UTF-8 +X-Content-Type-Message-Body: text/plain; charset=UTF-8 +Content-Type: text/plain; charset=UTF-8 +X-DSPAM-Result: Innocent +X-DSPAM-Processed: Fri Jan 4 04:49:08 2008 +X-DSPAM-Confidence: 0.6528 +X-DSPAM-Probability: 0.0000 + +Details: http://source.sakaiproject.org/viewsvn/?view=rev&rev=39752 + +Author: david.horwitz@uct.ac.za +Date: 2008-01-04 04:47:16 -0500 (Fri, 04 Jan 2008) +New Revision: 39752 + +Modified: +podcasts/branches/sakai_2-5-x/podcasts-app/src/webapp/css/podcaster.css +podcasts/branches/sakai_2-5-x/podcasts-app/src/webapp/podcasts/podMain.jsp +Log: +svn log -r39641 https://source.sakaiproject.org/svn/podcasts/trunk +------------------------------------------------------------------------ +r39641 | josrodri@iupui.edu | 2007-12-28 23:44:24 +0200 (Fri, 28 Dec 2007) | 1 line + +SAK-9882: refactored podMain.jsp the right way (at least much closer to) +------------------------------------------------------------------------ + +dhorwitz@david-horwitz-6:~/branchManagemnt/sakai_2-5-x> svn merge -c39641 https://source.sakaiproject.org/svn/podcasts/trunk podcasts/ +C podcasts/podcasts-app/src/webapp/podcasts/podMain.jsp +U podcasts/podcasts-app/src/webapp/css/podcaster.css + +conflict merged manualy + + + +---------------------- +This automatic notification message was sent by Sakai Collab (https://collab.sakaiproject.org/portal) from the Source site. +You can modify how you receive notifications at My Workspace > Preferences. + + + +From david.horwitz@uct.ac.za Fri Jan 4 04:33:44 2008 +Return-Path: +Received: from murder (mail.umich.edu [141.211.14.46]) +by frankenstein.mail.umich.edu (Cyrus v2.3.8) with LMTPA; +Fri, 04 Jan 2008 04:33:44 -0500 +X-Sieve: CMU Sieve 2.3 +Received: from murder ([unix socket]) +by mail.umich.edu (Cyrus v2.2.12) with LMTPA; +Fri, 04 Jan 2008 04:33:44 -0500 +Received: from workinggirl.mr.itd.umich.edu (workinggirl.mr.itd.umich.edu [141.211.93.143]) +by fan.mail.umich.edu () with ESMTP id m049Xge3031803; +Fri, 4 Jan 2008 04:33:42 -0500 +Received: FROM paploo.uhi.ac.uk (app1.prod.collab.uhi.ac.uk [194.35.219.184]) +BY workinggirl.mr.itd.umich.edu ID 477DFD6C.75DBE.26054 ; +4 Jan 2008 04:33:35 -0500 +Received: from paploo.uhi.ac.uk (localhost [127.0.0.1]) +by paploo.uhi.ac.uk (Postfix) with ESMTP id 6C929BA656; +Fri, 4 Jan 2008 09:33:27 +0000 (GMT) +Message-ID: <200801040932.m049W2i5006493@nakamura.uits.iupui.edu> +Mime-Version: 1.0 +Content-Transfer-Encoding: 7bit +Received: from prod.collab.uhi.ac.uk ([194.35.219.182]) +by paploo.uhi.ac.uk (JAMES SMTP Server 2.1.3) with SMTP ID 153 +for ; +Fri, 4 Jan 2008 09:33:10 +0000 (GMT) +Received: from nakamura.uits.iupui.edu (nakamura.uits.iupui.edu [134.68.220.122]) +by shmi.uhi.ac.uk (Postfix) with ESMTP id 6C69423767 +for ; Fri, 4 Jan 2008 09:33:13 +0000 (GMT) +Received: from nakamura.uits.iupui.edu (localhost [127.0.0.1]) +by nakamura.uits.iupui.edu (8.12.11.20060308/8.12.11) with ESMTP id m049W3fl006495 +for ; Fri, 4 Jan 2008 04:32:03 -0500 +Received: (from apache@localhost) +by nakamura.uits.iupui.edu (8.12.11.20060308/8.12.11/Submit) id m049W2i5006493 +for source@collab.sakaiproject.org; Fri, 4 Jan 2008 04:32:02 -0500 +Date: Fri, 4 Jan 2008 04:32:02 -0500 +X-Authentication-Warning: nakamura.uits.iupui.edu: apache set sender to david.horwitz@uct.ac.za using -f +To: source@collab.sakaiproject.org +From: david.horwitz@uct.ac.za +Subject: [sakai] svn commit: r39751 - in podcasts/branches/sakai_2-5-x/podcasts-app/src/webapp: css images podcasts +X-Content-Type-Outer-Envelope: text/plain; charset=UTF-8 +X-Content-Type-Message-Body: text/plain; charset=UTF-8 +Content-Type: text/plain; charset=UTF-8 +X-DSPAM-Result: Innocent +X-DSPAM-Processed: Fri Jan 4 04:33:44 2008 +X-DSPAM-Confidence: 0.7002 +X-DSPAM-Probability: 0.0000 + +Details: http://source.sakaiproject.org/viewsvn/?view=rev&rev=39751 + +Author: david.horwitz@uct.ac.za +Date: 2008-01-04 04:31:35 -0500 (Fri, 04 Jan 2008) +New Revision: 39751 + +Removed: +podcasts/branches/sakai_2-5-x/podcasts-app/src/webapp/images/rss-feed-icon.png +podcasts/branches/sakai_2-5-x/podcasts-app/src/webapp/podcasts/podPermissions.jsp +Modified: +podcasts/branches/sakai_2-5-x/podcasts-app/src/webapp/css/podcaster.css +podcasts/branches/sakai_2-5-x/podcasts-app/src/webapp/podcasts/podDelete.jsp +podcasts/branches/sakai_2-5-x/podcasts-app/src/webapp/podcasts/podMain.jsp +podcasts/branches/sakai_2-5-x/podcasts-app/src/webapp/podcasts/podNoResource.jsp +podcasts/branches/sakai_2-5-x/podcasts-app/src/webapp/podcasts/podOptions.jsp +Log: +svn log -r39146 https://source.sakaiproject.org/svn/podcasts/trunk +------------------------------------------------------------------------ +r39146 | josrodri@iupui.edu | 2007-12-12 21:40:33 +0200 (Wed, 12 Dec 2007) | 1 line + +SAK-9882: refactored the other pages as well to take advantage of proper jsp components as well as validation cleanup. +------------------------------------------------------------------------ +dhorwitz@david-horwitz-6:~/branchManagemnt/sakai_2-5-x> svn merge -c39146 https://source.sakaiproject.org/svn/podcasts/trunk podcasts/ +D podcasts/podcasts-app/src/webapp/podcasts/podPermissions.jsp +U podcasts/podcasts-app/src/webapp/podcasts/podDelete.jsp +U podcasts/podcasts-app/src/webapp/podcasts/podMain.jsp +U podcasts/podcasts-app/src/webapp/podcasts/podNoResource.jsp +U podcasts/podcasts-app/src/webapp/podcasts/podOptions.jsp +D podcasts/podcasts-app/src/webapp/images/rss-feed-icon.png +U podcasts/podcasts-app/src/webapp/css/podcaster.css + + + +---------------------- +This automatic notification message was sent by Sakai Collab (https://collab.sakaiproject.org/portal) from the Source site. +You can modify how you receive notifications at My Workspace > Preferences. + + + +From stephen.marquard@uct.ac.za Fri Jan 4 04:07:34 2008 +Return-Path: +Received: from murder (mail.umich.edu [141.211.14.25]) +by frankenstein.mail.umich.edu (Cyrus v2.3.8) with LMTPA; +Fri, 04 Jan 2008 04:07:34 -0500 +X-Sieve: CMU Sieve 2.3 +Received: from murder ([unix socket]) +by mail.umich.edu (Cyrus v2.2.12) with LMTPA; +Fri, 04 Jan 2008 04:07:34 -0500 +Received: from salemslot.mr.itd.umich.edu (salemslot.mr.itd.umich.edu [141.211.14.58]) +by panther.mail.umich.edu () with ESMTP id m0497WAN027902; +Fri, 4 Jan 2008 04:07:32 -0500 +Received: FROM paploo.uhi.ac.uk (app1.prod.collab.uhi.ac.uk [194.35.219.184]) +BY salemslot.mr.itd.umich.edu ID 477DF74E.49493.30415 ; +4 Jan 2008 04:07:29 -0500 +Received: from paploo.uhi.ac.uk (localhost [127.0.0.1]) +by paploo.uhi.ac.uk (Postfix) with ESMTP id 88598BA5B6; +Fri, 4 Jan 2008 09:07:19 +0000 (GMT) +Message-ID: <200801040905.m0495rWB006420@nakamura.uits.iupui.edu> +Mime-Version: 1.0 +Content-Transfer-Encoding: 7bit +Received: from prod.collab.uhi.ac.uk ([194.35.219.182]) +by paploo.uhi.ac.uk (JAMES SMTP Server 2.1.3) with SMTP ID 385 +for ; +Fri, 4 Jan 2008 09:07:04 +0000 (GMT) +Received: from nakamura.uits.iupui.edu (nakamura.uits.iupui.edu [134.68.220.122]) +by shmi.uhi.ac.uk (Postfix) with ESMTP id 90636418A8 +for ; Fri, 4 Jan 2008 09:07:04 +0000 (GMT) +Received: from nakamura.uits.iupui.edu (localhost [127.0.0.1]) +by nakamura.uits.iupui.edu (8.12.11.20060308/8.12.11) with ESMTP id m0495sZs006422 +for ; Fri, 4 Jan 2008 04:05:54 -0500 +Received: (from apache@localhost) +by nakamura.uits.iupui.edu (8.12.11.20060308/8.12.11/Submit) id m0495rWB006420 +for source@collab.sakaiproject.org; Fri, 4 Jan 2008 04:05:53 -0500 +Date: Fri, 4 Jan 2008 04:05:53 -0500 +X-Authentication-Warning: nakamura.uits.iupui.edu: apache set sender to stephen.marquard@uct.ac.za using -f +To: source@collab.sakaiproject.org +From: stephen.marquard@uct.ac.za +Subject: [sakai] svn commit: r39750 - event/branches/SAK-6216/event-util/util/src/java/org/sakaiproject/util +X-Content-Type-Outer-Envelope: text/plain; charset=UTF-8 +X-Content-Type-Message-Body: text/plain; charset=UTF-8 +Content-Type: text/plain; charset=UTF-8 +X-DSPAM-Result: Innocent +X-DSPAM-Processed: Fri Jan 4 04:07:34 2008 +X-DSPAM-Confidence: 0.7554 +X-DSPAM-Probability: 0.0000 + +Details: http://source.sakaiproject.org/viewsvn/?view=rev&rev=39750 + +Author: stephen.marquard@uct.ac.za +Date: 2008-01-04 04:05:43 -0500 (Fri, 04 Jan 2008) +New Revision: 39750 + +Modified: +event/branches/SAK-6216/event-util/util/src/java/org/sakaiproject/util/EmailNotification.java +Log: +SAK-6216 merge event change from SAK-11169 (r39033) to synchronize branch with 2-5-x (for convenience for UCT local build) + +---------------------- +This automatic notification message was sent by Sakai Collab (https://collab.sakaiproject.org/portal) from the Source site. +You can modify how you receive notifications at My Workspace > Preferences. + + + +From louis@media.berkeley.edu Thu Jan 3 19:51:21 2008 +Return-Path: +Received: from murder (mail.umich.edu [141.211.14.91]) +by frankenstein.mail.umich.edu (Cyrus v2.3.8) with LMTPA; +Thu, 03 Jan 2008 19:51:21 -0500 +X-Sieve: CMU Sieve 2.3 +Received: from murder ([unix socket]) +by mail.umich.edu (Cyrus v2.2.12) with LMTPA; +Thu, 03 Jan 2008 19:51:21 -0500 +Received: from eyewitness.mr.itd.umich.edu (eyewitness.mr.itd.umich.edu [141.211.93.142]) +by jacknife.mail.umich.edu () with ESMTP id m040pJHB027171; +Thu, 3 Jan 2008 19:51:19 -0500 +Received: FROM paploo.uhi.ac.uk (app1.prod.collab.uhi.ac.uk [194.35.219.184]) +BY eyewitness.mr.itd.umich.edu ID 477D8300.AC098.32562 ; +3 Jan 2008 19:51:15 -0500 +Received: from paploo.uhi.ac.uk (localhost [127.0.0.1]) +by paploo.uhi.ac.uk (Postfix) with ESMTP id E6CC4B9F8A; +Fri, 4 Jan 2008 00:36:06 +0000 (GMT) +Message-ID: <200801040023.m040NpCc005473@nakamura.uits.iupui.edu> +Mime-Version: 1.0 +Content-Transfer-Encoding: 7bit +Received: from prod.collab.uhi.ac.uk ([194.35.219.182]) +by paploo.uhi.ac.uk (JAMES SMTP Server 2.1.3) with SMTP ID 754 +for ; +Fri, 4 Jan 2008 00:35:43 +0000 (GMT) +Received: from nakamura.uits.iupui.edu (nakamura.uits.iupui.edu [134.68.220.122]) +by shmi.uhi.ac.uk (Postfix) with ESMTP id 8889842C49 +for ; Fri, 4 Jan 2008 00:25:00 +0000 (GMT) +Received: from nakamura.uits.iupui.edu (localhost [127.0.0.1]) +by nakamura.uits.iupui.edu (8.12.11.20060308/8.12.11) with ESMTP id m040NpgM005475 +for ; Thu, 3 Jan 2008 19:23:51 -0500 +Received: (from apache@localhost) +by nakamura.uits.iupui.edu (8.12.11.20060308/8.12.11/Submit) id m040NpCc005473 +for source@collab.sakaiproject.org; Thu, 3 Jan 2008 19:23:51 -0500 +Date: Thu, 3 Jan 2008 19:23:51 -0500 +X-Authentication-Warning: nakamura.uits.iupui.edu: apache set sender to louis@media.berkeley.edu using -f +To: source@collab.sakaiproject.org +From: louis@media.berkeley.edu +Subject: [sakai] svn commit: r39749 - in bspace/site-manage/sakai_2-4-x/site-manage-tool/tool/src: bundle webapp/vm/sitesetup +X-Content-Type-Outer-Envelope: text/plain; charset=UTF-8 +X-Content-Type-Message-Body: text/plain; charset=UTF-8 +Content-Type: text/plain; charset=UTF-8 +X-DSPAM-Result: Innocent +X-DSPAM-Processed: Thu Jan 3 19:51:20 2008 +X-DSPAM-Confidence: 0.6956 +X-DSPAM-Probability: 0.0000 + +Details: http://source.sakaiproject.org/viewsvn/?view=rev&rev=39749 + +Author: louis@media.berkeley.edu +Date: 2008-01-03 19:23:46 -0500 (Thu, 03 Jan 2008) +New Revision: 39749 + +Modified: +bspace/site-manage/sakai_2-4-x/site-manage-tool/tool/src/bundle/sitesetupgeneric.properties +bspace/site-manage/sakai_2-4-x/site-manage-tool/tool/src/webapp/vm/sitesetup/chef_site-importSites.vm +Log: +BSP-1420 Update text to clarify "Re-Use Materials..." option in WS Setup + +---------------------- +This automatic notification message was sent by Sakai Collab (https://collab.sakaiproject.org/portal) from the Source site. +You can modify how you receive notifications at My Workspace > Preferences. + + + +From louis@media.berkeley.edu Thu Jan 3 17:18:23 2008 +Return-Path: +Received: from murder (mail.umich.edu [141.211.14.91]) +by frankenstein.mail.umich.edu (Cyrus v2.3.8) with LMTPA; +Thu, 03 Jan 2008 17:18:23 -0500 +X-Sieve: CMU Sieve 2.3 +Received: from murder ([unix socket]) +by mail.umich.edu (Cyrus v2.2.12) with LMTPA; +Thu, 03 Jan 2008 17:18:23 -0500 +Received: from salemslot.mr.itd.umich.edu (salemslot.mr.itd.umich.edu [141.211.14.58]) +by jacknife.mail.umich.edu () with ESMTP id m03MIMXY027729; +Thu, 3 Jan 2008 17:18:22 -0500 +Received: FROM paploo.uhi.ac.uk (app1.prod.collab.uhi.ac.uk [194.35.219.184]) +BY salemslot.mr.itd.umich.edu ID 477D5F23.797F6.16348 ; +3 Jan 2008 17:18:14 -0500 +Received: from paploo.uhi.ac.uk (localhost [127.0.0.1]) +by paploo.uhi.ac.uk (Postfix) with ESMTP id EF439B98CE; +Thu, 3 Jan 2008 22:18:19 +0000 (GMT) +Message-ID: <200801032216.m03MGhDa005292@nakamura.uits.iupui.edu> +Mime-Version: 1.0 +Content-Transfer-Encoding: 7bit +Received: from prod.collab.uhi.ac.uk ([194.35.219.182]) +by paploo.uhi.ac.uk (JAMES SMTP Server 2.1.3) with SMTP ID 236 +for ; +Thu, 3 Jan 2008 22:18:04 +0000 (GMT) +Received: from nakamura.uits.iupui.edu (nakamura.uits.iupui.edu [134.68.220.122]) +by shmi.uhi.ac.uk (Postfix) with ESMTP id 905D53C2FD +for ; Thu, 3 Jan 2008 22:17:52 +0000 (GMT) +Received: from nakamura.uits.iupui.edu (localhost [127.0.0.1]) +by nakamura.uits.iupui.edu (8.12.11.20060308/8.12.11) with ESMTP id m03MGhrs005294 +for ; Thu, 3 Jan 2008 17:16:43 -0500 +Received: (from apache@localhost) +by nakamura.uits.iupui.edu (8.12.11.20060308/8.12.11/Submit) id m03MGhDa005292 +for source@collab.sakaiproject.org; Thu, 3 Jan 2008 17:16:43 -0500 +Date: Thu, 3 Jan 2008 17:16:43 -0500 +X-Authentication-Warning: nakamura.uits.iupui.edu: apache set sender to louis@media.berkeley.edu using -f +To: source@collab.sakaiproject.org +From: louis@media.berkeley.edu +Subject: [sakai] svn commit: r39746 - in bspace/site-manage/sakai_2-4-x/site-manage-tool/tool/src: bundle webapp/vm/sitesetup +X-Content-Type-Outer-Envelope: text/plain; charset=UTF-8 +X-Content-Type-Message-Body: text/plain; charset=UTF-8 +Content-Type: text/plain; charset=UTF-8 +X-DSPAM-Result: Innocent +X-DSPAM-Processed: Thu Jan 3 17:18:23 2008 +X-DSPAM-Confidence: 0.6959 +X-DSPAM-Probability: 0.0000 + +Details: http://source.sakaiproject.org/viewsvn/?view=rev&rev=39746 + +Author: louis@media.berkeley.edu +Date: 2008-01-03 17:16:39 -0500 (Thu, 03 Jan 2008) +New Revision: 39746 + +Modified: +bspace/site-manage/sakai_2-4-x/site-manage-tool/tool/src/bundle/sitesetupgeneric.properties +bspace/site-manage/sakai_2-4-x/site-manage-tool/tool/src/webapp/vm/sitesetup/chef_site-siteInfo-duplicate.vm +Log: +BSP-1421 Add text to clarify "Duplicate Site" option in Site Info + +---------------------- +This automatic notification message was sent by Sakai Collab (https://collab.sakaiproject.org/portal) from the Source site. +You can modify how you receive notifications at My Workspace > Preferences. + + + +From ray@media.berkeley.edu Thu Jan 3 17:07:00 2008 +Return-Path: +Received: from murder (mail.umich.edu [141.211.14.39]) +by frankenstein.mail.umich.edu (Cyrus v2.3.8) with LMTPA; +Thu, 03 Jan 2008 17:07:00 -0500 +X-Sieve: CMU Sieve 2.3 +Received: from murder ([unix socket]) +by mail.umich.edu (Cyrus v2.2.12) with LMTPA; +Thu, 03 Jan 2008 17:07:00 -0500 +Received: from anniehall.mr.itd.umich.edu (anniehall.mr.itd.umich.edu [141.211.93.141]) +by faithful.mail.umich.edu () with ESMTP id m03M6xaq014868; +Thu, 3 Jan 2008 17:06:59 -0500 +Received: FROM paploo.uhi.ac.uk (app1.prod.collab.uhi.ac.uk [194.35.219.184]) +BY anniehall.mr.itd.umich.edu ID 477D5C7A.4FE1F.22211 ; +3 Jan 2008 17:06:53 -0500 +Received: from paploo.uhi.ac.uk (localhost [127.0.0.1]) +by paploo.uhi.ac.uk (Postfix) with ESMTP id 0BC8D7225E; +Thu, 3 Jan 2008 22:06:57 +0000 (GMT) +Message-ID: <200801032205.m03M5Ea7005273@nakamura.uits.iupui.edu> +Mime-Version: 1.0 +Content-Transfer-Encoding: 7bit +Received: from prod.collab.uhi.ac.uk ([194.35.219.182]) +by paploo.uhi.ac.uk (JAMES SMTP Server 2.1.3) with SMTP ID 554 +for ; +Thu, 3 Jan 2008 22:06:34 +0000 (GMT) +Received: from nakamura.uits.iupui.edu (nakamura.uits.iupui.edu [134.68.220.122]) +by shmi.uhi.ac.uk (Postfix) with ESMTP id 2AB513C2FD +for ; Thu, 3 Jan 2008 22:06:23 +0000 (GMT) +Received: from nakamura.uits.iupui.edu (localhost [127.0.0.1]) +by nakamura.uits.iupui.edu (8.12.11.20060308/8.12.11) with ESMTP id m03M5EQa005275 +for ; Thu, 3 Jan 2008 17:05:14 -0500 +Received: (from apache@localhost) +by nakamura.uits.iupui.edu (8.12.11.20060308/8.12.11/Submit) id m03M5Ea7005273 +for source@collab.sakaiproject.org; Thu, 3 Jan 2008 17:05:14 -0500 +Date: Thu, 3 Jan 2008 17:05:14 -0500 +X-Authentication-Warning: nakamura.uits.iupui.edu: apache set sender to ray@media.berkeley.edu using -f +To: source@collab.sakaiproject.org +From: ray@media.berkeley.edu +Subject: [sakai] svn commit: r39745 - providers/trunk/cm/cm-authz-provider/src/java/org/sakaiproject/coursemanagement/impl/provider +X-Content-Type-Outer-Envelope: text/plain; charset=UTF-8 +X-Content-Type-Message-Body: text/plain; charset=UTF-8 +Content-Type: text/plain; charset=UTF-8 +X-DSPAM-Result: Innocent +X-DSPAM-Processed: Thu Jan 3 17:07:00 2008 +X-DSPAM-Confidence: 0.7556 +X-DSPAM-Probability: 0.0000 + +Details: http://source.sakaiproject.org/viewsvn/?view=rev&rev=39745 + +Author: ray@media.berkeley.edu +Date: 2008-01-03 17:05:11 -0500 (Thu, 03 Jan 2008) +New Revision: 39745 + +Modified: +providers/trunk/cm/cm-authz-provider/src/java/org/sakaiproject/coursemanagement/impl/provider/CourseManagementGroupProvider.java +Log: +SAK-12602 Fix logic when a user has multiple roles in a section + +---------------------- +This automatic notification message was sent by Sakai Collab (https://collab.sakaiproject.org/portal) from the Source site. +You can modify how you receive notifications at My Workspace > Preferences. + + + +From cwen@iupui.edu Thu Jan 3 16:34:40 2008 +Return-Path: +Received: from murder (mail.umich.edu [141.211.14.34]) +by frankenstein.mail.umich.edu (Cyrus v2.3.8) with LMTPA; +Thu, 03 Jan 2008 16:34:40 -0500 +X-Sieve: CMU Sieve 2.3 +Received: from murder ([unix socket]) +by mail.umich.edu (Cyrus v2.2.12) with LMTPA; +Thu, 03 Jan 2008 16:34:40 -0500 +Received: from icestorm.mr.itd.umich.edu (icestorm.mr.itd.umich.edu [141.211.93.149]) +by chaos.mail.umich.edu () with ESMTP id m03LYdY1029538; +Thu, 3 Jan 2008 16:34:39 -0500 +Received: FROM paploo.uhi.ac.uk (app1.prod.collab.uhi.ac.uk [194.35.219.184]) +BY icestorm.mr.itd.umich.edu ID 477D54EA.13F34.26602 ; +3 Jan 2008 16:34:36 -0500 +Received: from paploo.uhi.ac.uk (localhost [127.0.0.1]) +by paploo.uhi.ac.uk (Postfix) with ESMTP id CC710ADC79; +Thu, 3 Jan 2008 21:34:29 +0000 (GMT) +Message-ID: <200801032133.m03LX3gG005191@nakamura.uits.iupui.edu> +Mime-Version: 1.0 +Content-Transfer-Encoding: 7bit +Received: from prod.collab.uhi.ac.uk ([194.35.219.182]) +by paploo.uhi.ac.uk (JAMES SMTP Server 2.1.3) with SMTP ID 611 +for ; +Thu, 3 Jan 2008 21:34:08 +0000 (GMT) +Received: from nakamura.uits.iupui.edu (nakamura.uits.iupui.edu [134.68.220.122]) +by shmi.uhi.ac.uk (Postfix) with ESMTP id 43C4242B55 +for ; Thu, 3 Jan 2008 21:34:12 +0000 (GMT) +Received: from nakamura.uits.iupui.edu (localhost [127.0.0.1]) +by nakamura.uits.iupui.edu (8.12.11.20060308/8.12.11) with ESMTP id m03LX3Vb005193 +for ; Thu, 3 Jan 2008 16:33:03 -0500 +Received: (from apache@localhost) +by nakamura.uits.iupui.edu (8.12.11.20060308/8.12.11/Submit) id m03LX3gG005191 +for source@collab.sakaiproject.org; Thu, 3 Jan 2008 16:33:03 -0500 +Date: Thu, 3 Jan 2008 16:33:03 -0500 +X-Authentication-Warning: nakamura.uits.iupui.edu: apache set sender to cwen@iupui.edu using -f +To: source@collab.sakaiproject.org +From: cwen@iupui.edu +Subject: [sakai] svn commit: r39744 - oncourse/branches/oncourse_OPC_122007 +X-Content-Type-Outer-Envelope: text/plain; charset=UTF-8 +X-Content-Type-Message-Body: text/plain; charset=UTF-8 +Content-Type: text/plain; charset=UTF-8 +X-DSPAM-Result: Innocent +X-DSPAM-Processed: Thu Jan 3 16:34:40 2008 +X-DSPAM-Confidence: 0.9846 +X-DSPAM-Probability: 0.0000 + +Details: http://source.sakaiproject.org/viewsvn/?view=rev&rev=39744 + +Author: cwen@iupui.edu +Date: 2008-01-03 16:33:02 -0500 (Thu, 03 Jan 2008) +New Revision: 39744 + +Modified: +oncourse/branches/oncourse_OPC_122007/ +oncourse/branches/oncourse_OPC_122007/.externals +Log: +update external for GB. + +---------------------- +This automatic notification message was sent by Sakai Collab (https://collab.sakaiproject.org/portal) from the Source site. +You can modify how you receive notifications at My Workspace > Preferences. + + + +From cwen@iupui.edu Thu Jan 3 16:29:07 2008 +Return-Path: +Received: from murder (mail.umich.edu [141.211.14.46]) +by frankenstein.mail.umich.edu (Cyrus v2.3.8) with LMTPA; +Thu, 03 Jan 2008 16:29:07 -0500 +X-Sieve: CMU Sieve 2.3 +Received: from murder ([unix socket]) +by mail.umich.edu (Cyrus v2.2.12) with LMTPA; +Thu, 03 Jan 2008 16:29:07 -0500 +Received: from galaxyquest.mr.itd.umich.edu (galaxyquest.mr.itd.umich.edu [141.211.93.145]) +by fan.mail.umich.edu () with ESMTP id m03LT6uw027749; +Thu, 3 Jan 2008 16:29:06 -0500 +Received: FROM paploo.uhi.ac.uk (app1.prod.collab.uhi.ac.uk [194.35.219.184]) +BY galaxyquest.mr.itd.umich.edu ID 477D5397.E161D.20326 ; +3 Jan 2008 16:28:58 -0500 +Received: from paploo.uhi.ac.uk (localhost [127.0.0.1]) +by paploo.uhi.ac.uk (Postfix) with ESMTP id DEC65ADC79; +Thu, 3 Jan 2008 21:28:52 +0000 (GMT) +Message-ID: <200801032127.m03LRUqH005177@nakamura.uits.iupui.edu> +Mime-Version: 1.0 +Content-Transfer-Encoding: 7bit +Received: from prod.collab.uhi.ac.uk ([194.35.219.182]) +by paploo.uhi.ac.uk (JAMES SMTP Server 2.1.3) with SMTP ID 917 +for ; +Thu, 3 Jan 2008 21:28:39 +0000 (GMT) +Received: from nakamura.uits.iupui.edu (nakamura.uits.iupui.edu [134.68.220.122]) +by shmi.uhi.ac.uk (Postfix) with ESMTP id 1FBB042B30 +for ; Thu, 3 Jan 2008 21:28:38 +0000 (GMT) +Received: from nakamura.uits.iupui.edu (localhost [127.0.0.1]) +by nakamura.uits.iupui.edu (8.12.11.20060308/8.12.11) with ESMTP id m03LRUk4005179 +for ; Thu, 3 Jan 2008 16:27:30 -0500 +Received: (from apache@localhost) +by nakamura.uits.iupui.edu (8.12.11.20060308/8.12.11/Submit) id m03LRUqH005177 +for source@collab.sakaiproject.org; Thu, 3 Jan 2008 16:27:30 -0500 +Date: Thu, 3 Jan 2008 16:27:30 -0500 +X-Authentication-Warning: nakamura.uits.iupui.edu: apache set sender to cwen@iupui.edu using -f +To: source@collab.sakaiproject.org +From: cwen@iupui.edu +Subject: [sakai] svn commit: r39743 - gradebook/branches/oncourse_2-4-2/app/ui/src/java/org/sakaiproject/tool/gradebook/ui +X-Content-Type-Outer-Envelope: text/plain; charset=UTF-8 +X-Content-Type-Message-Body: text/plain; charset=UTF-8 +Content-Type: text/plain; charset=UTF-8 +X-DSPAM-Result: Innocent +X-DSPAM-Processed: Thu Jan 3 16:29:07 2008 +X-DSPAM-Confidence: 0.8509 +X-DSPAM-Probability: 0.0000 + +Details: http://source.sakaiproject.org/viewsvn/?view=rev&rev=39743 + +Author: cwen@iupui.edu +Date: 2008-01-03 16:27:29 -0500 (Thu, 03 Jan 2008) +New Revision: 39743 + +Modified: +gradebook/branches/oncourse_2-4-2/app/ui/src/java/org/sakaiproject/tool/gradebook/ui/RosterBean.java +Log: +svn merge -c 39403 https://source.sakaiproject.org/svn/gradebook/trunk +U app/ui/src/java/org/sakaiproject/tool/gradebook/ui/RosterBean.java + +svn log -r 39403 https://source.sakaiproject.org/svn/gradebook/trunk +------------------------------------------------------------------------ +r39403 | wagnermr@iupui.edu | 2007-12-17 17:11:08 -0500 (Mon, 17 Dec 2007) | 3 lines + +SAK-12504 +http://jira.sakaiproject.org/jira/browse/SAK-12504 +Viewing "All Grades" page as a TA with grader permissions causes stack trace +------------------------------------------------------------------------ + + +---------------------- +This automatic notification message was sent by Sakai Collab (https://collab.sakaiproject.org/portal) from the Source site. +You can modify how you receive notifications at My Workspace > Preferences. + + + +From cwen@iupui.edu Thu Jan 3 16:23:48 2008 +Return-Path: +Received: from murder (mail.umich.edu [141.211.14.91]) +by frankenstein.mail.umich.edu (Cyrus v2.3.8) with LMTPA; +Thu, 03 Jan 2008 16:23:48 -0500 +X-Sieve: CMU Sieve 2.3 +Received: from murder ([unix socket]) +by mail.umich.edu (Cyrus v2.2.12) with LMTPA; +Thu, 03 Jan 2008 16:23:48 -0500 +Received: from salemslot.mr.itd.umich.edu (salemslot.mr.itd.umich.edu [141.211.14.58]) +by jacknife.mail.umich.edu () with ESMTP id m03LNlf0002115; +Thu, 3 Jan 2008 16:23:47 -0500 +Received: FROM paploo.uhi.ac.uk (app1.prod.collab.uhi.ac.uk [194.35.219.184]) +BY salemslot.mr.itd.umich.edu ID 477D525E.1448.30389 ; +3 Jan 2008 16:23:44 -0500 +Received: from paploo.uhi.ac.uk (localhost [127.0.0.1]) +by paploo.uhi.ac.uk (Postfix) with ESMTP id 9D005B9D06; +Thu, 3 Jan 2008 21:23:38 +0000 (GMT) +Message-ID: <200801032122.m03LMFo4005148@nakamura.uits.iupui.edu> +Mime-Version: 1.0 +Content-Transfer-Encoding: 7bit +Received: from prod.collab.uhi.ac.uk ([194.35.219.182]) +by paploo.uhi.ac.uk (JAMES SMTP Server 2.1.3) with SMTP ID 6 +for ; +Thu, 3 Jan 2008 21:23:24 +0000 (GMT) +Received: from nakamura.uits.iupui.edu (nakamura.uits.iupui.edu [134.68.220.122]) +by shmi.uhi.ac.uk (Postfix) with ESMTP id 3535542B69 +for ; Thu, 3 Jan 2008 21:23:24 +0000 (GMT) +Received: from nakamura.uits.iupui.edu (localhost [127.0.0.1]) +by nakamura.uits.iupui.edu (8.12.11.20060308/8.12.11) with ESMTP id m03LMFtT005150 +for ; Thu, 3 Jan 2008 16:22:15 -0500 +Received: (from apache@localhost) +by nakamura.uits.iupui.edu (8.12.11.20060308/8.12.11/Submit) id m03LMFo4005148 +for source@collab.sakaiproject.org; Thu, 3 Jan 2008 16:22:15 -0500 +Date: Thu, 3 Jan 2008 16:22:15 -0500 +X-Authentication-Warning: nakamura.uits.iupui.edu: apache set sender to cwen@iupui.edu using -f +To: source@collab.sakaiproject.org +From: cwen@iupui.edu +Subject: [sakai] svn commit: r39742 - gradebook/branches/oncourse_2-4-2/app/ui/src/java/org/sakaiproject/tool/gradebook/ui +X-Content-Type-Outer-Envelope: text/plain; charset=UTF-8 +X-Content-Type-Message-Body: text/plain; charset=UTF-8 +Content-Type: text/plain; charset=UTF-8 +X-DSPAM-Result: Innocent +X-DSPAM-Processed: Thu Jan 3 16:23:48 2008 +X-DSPAM-Confidence: 0.9907 +X-DSPAM-Probability: 0.0000 + +Details: http://source.sakaiproject.org/viewsvn/?view=rev&rev=39742 + +Author: cwen@iupui.edu +Date: 2008-01-03 16:22:14 -0500 (Thu, 03 Jan 2008) +New Revision: 39742 + +Modified: +gradebook/branches/oncourse_2-4-2/app/ui/src/java/org/sakaiproject/tool/gradebook/ui/RosterBean.java +Log: +svn merge -c 35014 https://source.sakaiproject.org/svn/gradebook/trunk +U app/ui/src/java/org/sakaiproject/tool/gradebook/ui/RosterBean.java + +svn log -r 35014 https://source.sakaiproject.org/svn/gradebook/trunk +------------------------------------------------------------------------ +r35014 | wagnermr@iupui.edu | 2007-09-12 16:17:59 -0400 (Wed, 12 Sep 2007) | 3 lines + +SAK-11458 +http://bugs.sakaiproject.org/jira/browse/SAK-11458 +Course grade does not appear on "All Grades" page if no categories in gb +------------------------------------------------------------------------ + + +---------------------- +This automatic notification message was sent by Sakai Collab (https://collab.sakaiproject.org/portal) from the Source site. +You can modify how you receive notifications at My Workspace > Preferences. diff --git a/Assignment 9.4/readme.md b/Assignment 9.4/readme.md new file mode 100644 index 0000000..c5238d4 --- /dev/null +++ b/Assignment 9.4/readme.md @@ -0,0 +1,6 @@ +9.4 Write a program to read through the mbox-short.txt and figure out who has sent the greatest number of mail messages. The program looks for 'From ' lines and takes the second word of those lines as the person who sent the mail. The program creates a Python dictionary that maps the sender's mail address to a count of the number of times they appear in the file. After the dictionary is produced, the program reads through the dictionary using a maximum loop to find the most prolific committer. + +Desired Output: +``` +cwen@iupui.edu 5 +``` \ No newline at end of file diff --git a/readme.md b/readme.md new file mode 100644 index 0000000..e69de29